You are on page 1of 77

Chapter-31

Mensuration II
(Surface Area and Volume)
Cuboid and Cube
Here we will introduce two rectangular solids,
namely, a cuboid and a cube. Before going into
the details of cuboid and cube, let me introduce a
term Parallelopiped to you (See the figure given
below).
A solid bounded by three pairs of parallel plane
surfaces is called a parallelopiped. The plane surfaces
are known as the faces of the parallelopiped.
Each face of a parallelopiped is a parallelogram
and opposite faces are congruent ie equal in all
respects.
Any two adjacent faces of a parallelopiped meet
along a line segment which is called an edge. A
parallelopiped has 12 edges. Point of intersection
of two edges is called a vertex or a corner. A
parallelopiped has 8 vertices or corners. Note that
at each vertex, three edges and three faces meet.

(i) Cuboid
A cuboid is a solid bounded by six rectangular plane
regions. In other words, A parallelopiped whose faces
are rectangles and adjacent faces are perpendicular is
called a rectangular parallelopiped or a cuboid.
The figure given below represents a cuboid. It
should be noted t hat it is not a plane figure,
although it is drawn on a sheet of the paper.

Note: A word cuboid is used for both, the hollow


cuboid and the solid cuboid. In any use of
the word, the context in which it is used,
will make the meaning clear.

(ii) Cube
A cuboid whose length, breadth and height are all
equal is called a cube.
As with a cuboid, the word cube is also used
for both the hollow cube and the solid cube. In
any use of the word, the context in which it is
used, will make the meaning clear. Each edge of a
cube is called its side.

Important Formulae
(i) Volume of a Cuboid and a Cube
(a) Volume of a Cuboid : The volume V of a cuboid
of length = l cm, breadth = b cm and height = h cm
is given by
V = (l b h) cm3
Note: 1. While finding the volume of a cuboid, its
lengt h, br eadt h and height must be
expressed in the same units.
2 . From the above formula, we also obtain
that
Volume
V

Breadth Height ie l = b h

Length =

Volume
V

Breadth = Length Height ie b =


l h

and
l

Volume
V

Height = Length Breadth ie h =


l b

(b) Volume of a Cube : We know that a cube is a


special type of a cuboid whose length, breadth and
height are all equal.
So, the volume V of a cube of side (or edge) l cm
is given by
V = (l l l) cm3 or V = l3 cm3
or, V = (side)3

564
(ii) Surface Area of a Cuboid and a Cube
(a) Surface Area of a Cuboid : Total surface area
of the cuboid = 2 (length breadth + breadth
height + length height) cm 2
Note:For the calculation of surface area of a cuboid,
the length, breadth and height must be
expressed in the same units.
(b) Surface Area of a Cube : Since all the six faces
of a cube are squares of the same size ie for a cube
we have l = b = h. Thus, if l cm is the length of the
edge or side of a cube, then surface area of the
cube = 2 (l l + l l + l l) = 2 3l2 = 6l2 = 6 (Edge)2.

(iii) Diagonal of a Cuboid and a Cube


(a) Diagonal of a Cuboid : If the edges of a cuboid
be l, b and h units of length, then its diagonal is
l 2 b 2 h 2 . This can be proved with the help of
Pythagoras Theorem.
(b) Diagonal of a Cube : Here the edges are all
equal. Let each edge be l units.
diagonal =

l 2 l 2 l 2 l 3 units.
(c) The length of the longest rod that can be placed
in a room whose dimensions are l, b and h =
l 2 b2 h 2 .

Standard Units of Volume

Concept of Arithmetic
Metre Cube and Cubic Metre: The solid region
formed by a cube of side 1 m is called a metre cube
and its volume is l cubic metre (lm3).
Since 1 m = 100 cm
1 m3 = (100 100 100) cm3
= 1000000 cm3
But, 1000 cm3 = 1 litre

1000000
litres
1 m3 =
1000
3
or, 1 m = 1000 litres or 1000 l.
Now, 1l = 1000 cm 3.

1
th of a litre
1000
This is generally called a millilitre (ml).
Thus 1l = 1000 ml.
Cubic Millimetre: The volume of a solid cubical
region of side 1 mm is called a cubic millimetre
(mm3).
Since 1 cm = 10 mm
1 cm3 = (10 10 10) mm3 = 1000 mm3
Note:1 m3 is also called a kilolitre. Thus,
1 kilolitre = 1 m3
= 1000000 cm3
= 1000 1000 cm3
= 1000 litres [ 1000 cm3 = 1 litre]
1 cm3 =

Conversion of Units

K KUNDAN

Since there are various units of measurement of


lengt h like metr e, decimet r e, decamet r e et c
t her ef or e ther e ar e many standar d unit s of
measurements of volume.
The volume of any solid figure is the amount
of space enclosed within its bounding faces. This
is measured by the number of cubic metres, or cubic
kilometres or cubic centimetres (or other units of
cubic measure) it contains.
Cubic Centimetre : A cubic centimetre is the
amount of space enclosed within the faces of a
cube of which each edge is 1 centimetre.
Litre or Cubic Decimetre: The volume of the
solid region formed by a cube of side 1 decimetre
(dm) is called a litre or a cubic decimetre (1 dm3).
Since 1 dm = 10 cm
1 dm3 = 1 dm 1 dm 1 dm
= (10 10 10) cm3 = 1000 cm3
or, 1 litre = 1000 cm3.

Each of the various standard units introduced above


can be converted into the others by using the
following table:
Units of Length
Millimetre (mm)
Centimetre (cm)
1 cm = 10 mm
Decimetre (dm)
1 dm = 10 cm

Metre
1m = 10 dm

Units of Volume
Cubic millimetre (mm3)
Cubic
cent imet r e
or
millilitre
1 cm3 = (10 10 10) mm3
Cubic decimetre or litre
1 dm3 ( = 1 litre)
= (10 10 10) cm3
= 1000 cm3 = 1000 ml
[ 1 cm3 = 1 ml]
Cubic metre (m3)
1 m3 = (10 10 10) dm3
= 1000 dm3
= 1000 litre

565

Mensuration II (Surface Area and Volume)

Solved Examples
Ex. 1:

Soln:

Ex. 2:
Soln:

A match box measures 4 cm by 2.5 cm


by 1.5 cm. What will be the volume of a
packet contai ning 12 such m at ch
boxes? How many such packets can be
placed in a card board box whose size
is 60 cm 30 cm 24 cm?
We have,
Volume of a match box
= (4 2.5 1.5) cm3 = 15 cm3
Volume of a packet containing 12 match
boxes
= (12 15) cm3 = 180 cm3.
Now, volume of card board box
= (60 30 24) cm2 = 43200 cm3.
Number of packets that can be put in a

43200
= 240.
card board =
180
What will happen to the volume of a
cube if its edge is doubled?
Let the edge of the cube be l cm. Then, its
volume V is given by
V = l3 cm3
....... (i)
Let V1 be the volume of the cube when its
edge is doubled.
Then, V1 = (2l) 3 cm3
[ Length of the edge of
new cube = 2l cm]
or, V1 = 8l3
or, V1 = 8V
[Using (i)]
Hence, if each edge of the cube is doubled,
then the volume becomes 8 times.
Three cubes whose edges measure 3 cm,
4 cm and 5 cm respectively to form a
single cube. If there be no loss of metal
in the process find its edge. Also, find
the surface area of the new cube.
Let x cm be the edge of the new cube.
Th en ,
Volume of the new cube = Sum of the
volumes of three cubes
The underlying concept for these type of
questions is that the total volume of a
solid does not change even when its
shape changes.
or, x3 = 33 + 43 + 53 =27 + 64 + 125
or, x3 = 216
or, x3 = 63
x = 6 cm
Edge of the new cube is 6 cm long
surface area of the new cube
= 6x2 = 6 (6)2 cm2 = 216 cm2
A rectangular block of ice measures 40
cm by 25 cm by 15 cm. Calculate its

Soln:

weight of the same volume of water and


1 cm3 of water weighs 1 gm.
We have,
Volume of rectangular block of ice
= 40 25 15 cm3 = 15000 cm3
Now, weight of 1 cm3 of water = 1 gm

9
th of the
and weight of 1 cm3 of ice =
10
weight of 1 cm3 of water
9
gm.
Weight of 1 cm3 of ice =
10
Weight of the rectangular block of ice

Ex. 5:

Soln:

15000 gm = 13500 gm = 13.5 kg


=
10

Eight identical cuboidal wooden blocks


are stacked one on top of the other.
The total volume of the solid so formed
is 128 cm3. If the height of each block
is 1 cm and base is a square, find the
dimensions of each block.
Let the length and breadth of the base of
each wooden block be x cm each. Since
eight identical blocks are stacked one on
top of the other and the height of each
block is 1 cm.
So, height of the solid formed = 8 cm.
volume of the solid formed
= (x x 8) cm3 = 8x 2 cm3.
But the volume of solid formed is 128 cm3.
8x2 = 128

K KUNDAN

Ex. 3:

Soln:

Ex. 4:

weight in kg, if ice weighs

9
10

of the

128
8
or, x2 = 16
or, x2 = 42
x = 4 cm.
Hence, each wooden block is of dimension
4 cm 4 cm 1 cm.
A rectangular water reservoir contains
42000 litres of water. Find the depth
of the water in the reservoir if its base
measures 6 m by 3.5 m.
We have,
Volume of the reservoir = 42000 litres
or, x2 =

Ex. 6:

Soln:

42000 3
m [ 1 m3 = 1000 litres]
1000
= 42 m3.
Length of the reservoir = 6 m
Breadth of the reservoir = 3.5 m
Area of the base of the reservoir
= (6 3.5) m2 = 21 m2.
But, (Area of the base Height)
= Volume of the reservoir
=

566

Concept of Arithmetic
Height or Depth of the reservoir

Volume
42
m = 2m.
=
Area of the base
21
Hence, the depth of the reservoir is 2 m.
How many bricks of size 22 cm 10
cm 7 cm are required to construct a
wall 33 m long, 3.5 m high and 40 cm
thick, if cement and sand used in the
=

Ex. 7:

construction occupy
Soln:

1
th part of the
10

wall?
We have,
Length of the wall = 33 m
Height of the wall = 3.5 m
Thickness of the wall = 40 cm

40
m = 0.4 m
100
Volume of the wall
= (33 3.5 0.4) m3 = 46.2 m3
Volume of the space occupied by the
cement and sand
=

1
th of the volume of the wall
10

46.2
=
m2 = 4.62 m2
10
Volume of the bricks
= (46.2 4.62) m3
= 41.58 m3
= 41.58 1000000 cm 3
= 41580000 cm 3
Now,
Volume of a brick = (22 10 7) cm3
= 1540 cm3
Number of bricks

But the surface areas is given as 54 cm2.


6l2 = 54

54
= 9
6
2
2
or, l = 3
l = 3 cm
Now, volume of the cube = l3 cm3
= 33 cm3 = 27 cm3.
Ex. 10: A cuboidal oil tin is 30 cm by 40 cm by
50 cm. Find the cost of the tin required
for making 20 such tins if the cost of
tin sheet is Rs 20 per square metre.
Soln:
The cost of tins depends upon their total
surface area.
It is given that a tin is in the shape of a
cuboid such that
l = 30 cm, b = 40 cm and h = 50 cm
Surface area of one tin
= 2 (lb + bh + lh)
= 2(30 40 + 40 50 + 30 50) cm2
= 2 (1200 + 2000 + 1500) cm2
= (2 4700) cm2 = 9400 cm2
Surface area of 20 such tins
= (20 9400) cm2
= 188000 cm 2
or, l2 =

188000
m2 [ 10000 cm2 = 1 m2]
10000
= 18.8 m2
Now, cost of 1 square metre of tin sheet
= Rs 20.
Cost of 18.8 m2 of tin sheet
= Rs (20 18.8) = Rs 376
Hence, the cost of making 20 tins
= Rs 376.
Ex. 11: The pai nt i n a certain container is
sufficient to paint an area equal to
9.375 m2. How many bricks measuring
22.5 cm by 10 cm by 7.5 cm can be
painted out of this container?
Soln:
We have,
l = length of a brick = 22.5 cm
b = breadth of a brick = 10 cm
h = height of a brick = 7.5 cm
Surface area of a brick
= 2 (lb + bh + lh)
= 2( 22.510+107.5 + 7.522.5) cm 2
= 2(225 + 75 + 168.75) cm2
= 937.5 cm 2.
The paint in the container is sufficient to
paint area = 9.375 m2
= 9.375 10000 cm 2
[ 1 m2 = 10000 cm2]
= 93750 cm 2.
Number of bricks that can be painted
=

K KUNDAN
=

Ex. 8:
Soln:

Ex. 9:
Soln:

Volume of the bricks


Volume of a brick

41580000
= 27000
=
1540

Find the surface area of a cube whose


volume is 512 m3.
Let the length of an edge of the cube be
l m.
Then, its volume is l3.
But, the volume is given as 512 m3.
or, l3 = 512
or, l3 = 83
l = 8 m
Now, surface area = 6l2 = 6 (8)2 m2
= 6 64 m2 = 384 m2.
Find the volume of a cube whose surface
area is 54 cm 2 .
Let the length of an edge of the cube be l
cm. Then, its surface area is 6l2 cm2.

93750
937.5
= 100.

out of the paint in the container =

567

Mensuration II (Surface Area and Volume)


Ex. 12: A water tank built by a municipality of
a town to supply water to its 25000
inhabitants at 125 litres per day per
person is 40 m long and 31.25 m broad.
The tank, when it is full, can supply
water for two days to the inhabitants
of the town. Find the depth of the tank.
Soln:
Water consumed by the inhabitants in one
day
= Number of inhabitants Water consumed
by an inhabitant in one day
= (25000 125) litres
= 3125000 litres
Water consumed by the inhabitants in
two days = (3125000 2) litres
= 6250000 litres
6250000
=
m3 [ 1000 litres = 1 m3]
1000
3
= 6250 m
Volume of the tank = 6250 m3
Depth of the tank
=

Volume
6250

m = 5m
Length Breadth 1250

Ex. 13: A rectangular field is 154 m long and


121 m broad. A well of 14 m length and
11 m breadth is dug inside the field
and mud taken out is spread evenly
over the remaining part of the field to
a thickness of 25 cm. Find the depth
of the well.
Soln:
We have, length of the well = 14 m
Breadth of the well = 11 m
Area of the base of the well
= (14 11) m2 = 154 m2
Also, area of the field
= (154 121) m2 = 18634 m2
Area of the field in which mud is spread
= (18634 - 154) m2 = 18480 m2
Thickness of the mud
25
1
m m.
= 25 cm =
100
4
Volume of the mud

l + b + h = 19
Diagonal = 11 cm

.... (i)

or,

l 2 b 2 h 2 11
or, l + b2 + h2 = 121
.... (ii)
Now, l + b + h = 19
or, (l + b + h)2 = 192
or, l2 + b2 + h2 + 2 (lb + bh + lh) = 361
or, 121 + 2(lb + bh + lh) = 361
[Using (ii)]
or, 2 (lb + bh + lh) = 240
Hence, the surface area of the cuboid is
240 cm 2.
Ex. 15: A plot of land in the form of a rectangle
has a dimension 240 m 180 m. A
drainlet 10 m wide is dug all around it
(on the outside) and the earth dug out
i s ev enl y spr ead ov er t he plot ,
increasing its surface level by 25 cm.
Find the depth of the drainlet.
Soln:
Let the depth of the drainlet be x metres.
Width of the drainlet = 10 m.
Volume of the drainlet
= (260 10 x + 260 10 x + 180
10 x + 180 10 x) m3
= (5200x + 3600x) m3 = 8800x m 3
When earth dug out is evenly spread over
the plot, we get a cuboid whose base area
is 240 180 m2 and height = 25 cm
= 0.25 m.
2

K KUNDAN
1 3

3
= 18480 m 4620 m
4

Depth of the well


Volume of the mud taken out of the well
=
Area of the base of the well

4620
m = 30 m
154
Ex. 14: The sum of length, breadth and depth
of a cuboid is 19 cm and the length of
its diagonal is 11 cm. Find the surface
area of the cuboid.
Soln:
Let the length, breadth and height of the
cube be l cm, b cm and h cm respectively.
Th en ,
=

Volume of earth spread over the plot


= (240 180 0.25) m3
= 10800 m3
Clearly, volume of earth spread over the
plot = Volume of the drainlet
10800 = 8800x
10800
or, x =
= 1.227 m
8800
Ex. 16: Three cubes each of side 5 cm are joined
end to end. Find the surface area of
the resulting cuboid.
Soln:
The dimensions of the cuboid so formed
are as under:

l = Length = 15 cm,
b = Breadth = 5 cm,
h = Height = 5 cm.

568

Concept of Arithmetic

So, the surface area of the cuboid


= 2 (15 5 + 5 5 + 15 5) cm2
= 2 (75 + 25 + 75) cm2 = 350 cm2
Ex. 17: In the given figure, the shape of a solid
copper piece (made of two pieces with
dimensions as shown in the figure) is
shown. The face ABCDEFA i s t he
uniform cross-section. Assume that the
angles at A, B, C, D, E and F are right
angles. Calculate the volume of the
piece.

Ex. 19: An agricultural field is in the form of a


rectangle of length 20 m and width 14
m. A pit 6 m long, 3 m wide and 2.5 m
deep is dug in a corner of the field and
the earth taken out of the pit is spread
uniformly over the remaining area of
the field. Find the extent to which the
level of the field has been raised.
Soln:
Let ABCD be the field and let AB1C1D1 be
the part of the field where a pit is to be
dug.

Soln:

For the horizontal piece, we have


Length = 8 cm, Breadth = 22 cm, Height
= 3 cm
Volume of the horizontal piece
= (8 22 3) cm3 = 528 cm3
For the vertical piece, we have
Length = 22 cm, Breadth = 2 cm,
Height = (5 + 3) cm = 8 cm
Volume of the vertical piece
= (22 2 8) cm3 = 352 cm3
Hence, volume of the whole piece
= (528 + 352) cm3 = 880 cm3
Ex. 18: A metallic sheet is of the rectangular
shape with dimensions 48 cm 36 cm.
From each one of its corners, a square
of 8 cm is cutoff. An open box is made of
the remaining sheet. Find the volume of
the box.
Soln:
In order to make an open box, a square of
side 8 cm is cut off from each of the four
corners and the flaps are folded up.

Volume of the earth dugout


= (6 3 2.5) m3 = 45 m3
... (i)
Area of the remaining part of the field
= Area of the field Area of pit
= (20 14 6 3) m2 = 262 m2
The earth taken out of the pit is spread
uniformly over the remaining area of the
field. Let h metres be the level raised over
the f ield uniformly. Clearly, the earth
taken out forms a cuboid of base area 262
m2 and height h.
Volume of the earth dugout
= (262 h) m3
... (ii)
From (i) and (ii), we get
262h = 45

K KUNDAN
45
0.1718m = 17.18 cm
or, h =
262
Hence, the level is raised by 17.18 cm.
Ex. 20: A solid cube is cut into two cuboids of
equal volumes. Find the ratio of the
total surface area of the given cube and
that of one of the cuboids.
Soln:
Let the edge of the solid cube be a units.
Since the cube is cut into two cuboids of
equal volumes. Therefore, the dimensions
of each of the cuboid are: length = a units,
breadth = a units and height =
Thus, t he box will have the following
dimensions:
Length = (48 8 8) cm = 32 cm
Breadth = (36 8 8) cm = 20 cm,
Height = 8 cm
Volume of the box formed
= (32 20 8) cm3 = 5120 cm3

a
units.
2

Now,
S = Total surface area of cube = 6a2 sq
units.
S1 = Total surface area of one cuboid

a a

= 2 a a a a
2 2

= 4a2 sq units
S : S1 = 6a2 : 4a2 = 3 : 2

569

Mensuration II (Surface Area and Volume)


Ex. 21: Water in a canal, 30 dm wide and 12
dm deep, is flowing with a velocity of
20 km per hour. How much area will it
i rr i gat e in 30 m i nut es i f 9 cm of
standing water is desired?
Soln:
Water in the canal forms a cuboid whose
breadth = 12 dm =

12
m = 1.2 m,
10

Ex. 23: A rectangular tank is 225 m by 162 m


at the base. With what speed must
water flow into it through an aperture
60 cm by 45 cm that the level may be
raised 20 cm in 5 hours?
Soln:
Since the level of water rises by 20 cm in
5 hours. Therefore, the volume of the water
flowed in the tank in 5 hours

20

m3
= 225 162
100

So, the volume of the water flowed in the


tank in one hour

30
height = 30 dm =
m = 3 m and
10
length = distance covered by water in 30
minutes = Velocity of water time

30
= 20000
m = 10000 m
60
Volume of water flown in 30 minutes
= lbh
= (1.2 3 10000) m3 = 36000 m3
Suppose area irrigated be A m2. Then,
A

= 1458 m3
... (i)
Area of the cross-section of aperture

45 2
27
60

m2
m
=
100
100
100

Let the speed of the water be x metres per


hour. Then,
volume of the water flowed in the tank in
one hour
= (Area of cross-section of aperture)
(speed in metre per hour)

9
= Volume flown in 30 minutes
100

9
= 36000
100
A = 400000 m2
Hence, area irrigated = 400000 m 2.
Ex. 22: A rectangular reservoir is 120 m long
and 75 m wide. At what speed per hour
m ust wat er fl ow i nt o it t hr ough a
square pipe of 20 cm wide so that the
water rises by 2.4 m in 18 hours.
Soln:
Volume of wat er accumulat ed in t he
reservoir in 18 hours
= (120 75 2.4) m3
Volume of water accumulated in one

1
20
225 162
m3
5
100

or, A

27

x
=
100

From (i) and (ii), we get

K KUNDAN
120 75 2.4
m3
hour =
18

Now, cross-sectional area of the square


pipe

27
x 1458
100

1458 100
m/hr
27
or, x = 5400 m/hr
Ex. 24: The external length, breadth and height
of a closed rectangular wooden box are
18 cm, 10 cm and 6 cm respectively and
or, x =

1
cm. When the
2
box is empty, it weighs 15 kg and when
filled with sand it weighs 100 kg. Find
the weight of the cubic cm of wood and
cubic cm of sand.
thickness of wood is

20
20

m2
=
100
100

1
m2 = 0.4 m2
25
Speed of water
=

120 75 2.4
m/hr
=
18 0.4

120 75 2.4
km/hr
=
18 0.4 1000
= 30 km/hr
Hence, water must flow at the speed of
30 km/hour into the reservoir.

... (ii)

Soln:

1
cm
2
Internal length of wooden box
Thickness of wood =

1 1
= 18 17 cm
2 2

Internal breadth of wooden box

1 1
= 10
2 2

9 cm

570

Concept of Arithmetic
Internal depth of wooden box

1 1
= 6 5 cm
2 2

Internal volume of wooden box


= (17 9 5) cm3 = 765 cm3
External volume of wooden box
= (18 10 6) cm3 = 1080 cm3
Volume of wood
= External volume Internal volume
= (1080 765) cm3 = 315 cm3
Weight of empty box = 15 kg
or, Weight of 315 cm3 wood is 15 kg
Weight of 1 cm3 of wood
1
15
kg =
=
kg.
315
21
Now, volume of sand = Internal volume of
box = 765 cm3
Weight of sand = Weight of box filled with
sand Weight of empty box
= (100 15) kg = 85 kg
Volume of sand = 765 cm3
Weight of 1 cm3 of sand
1
85
kg =
=
kg.
765
9
Ex. 25: The out er di m ensi ons of a cl osed
wooden box are 10 cm by 8 cm by 7 cm.
Thickness of the wood is 1 cm. Find
the total cost of wood required to make
box if 1 cm3 of wood cost Rs 2.00.
Soln:
The external dimension of the box are:
Length = 10 cm, Breadth = 8 cm, Height
= 7 cm
External volume of the box
= (10 8 7) cm3 = 560 cm3
Thickness of the wood = 1 cm
Internal length = (10 2 =) 8 cm
Internal breadth = (8 2 =) 6 cm
Internal height = (7 2 =) 5 cm
Internal volume = (8 6 5) cm3
= 240 cm3
Volume of the wood
= External volume Internal volume
= (560 240) cm3 = 320 cm3
Hence, total cost of wood required to make
box
= Rs (320 2) = Rs 640.

Suppose we have to find the volume of water in a


swimming pool. The heights of the water level are
h1 and h2, l is the length of the swimming pool,
and w is the width of the swimming pool.
Firstly, we have to note in the given figure
which cross-sectional area is the same throughout.
Face-vertical cross-sectional area, ie ABCD, is
the same throughout. ABCD is a trapezium.
Volume of water in the swimming pool
= (Ar ea of cr oss-sect ion ABCD)
Corresponding height
= {Area of Trapezium ABCD} w

= h1 h 2 l w
2

Solved Examples
Ex. 26: See the figure of a roof-top house given
below.

K KUNDAN

Swimming Pool

Soln:

Find the volume of the house.


In this case, we take side-vertical cross
section.
(see the Note of Ex. 27).
Volume of the given house

1
Area of the (ABCD) length of
2
the house (CH)
[ Side-vertical cross-section of the rooftop house gives a trapezium ABCD having
par allel sides as AB and CD and the
height as BC]
=

= 9 5 6 20
2

= 840 m3
Ex. 27: The l engt h and t he wi dt h of a
swimming pool are 50 metres and 15
metres respectively. If the depth of the
swimming pool at one end is 10 metres
and at the other 20 metres, then find
the volume of water in the swimming
pool.

571

Mensuration II (Surface Area and Volume)


Soln:

Area of the four walls of the room


= Area of the rectangle
= Length of the rectangle Height of the
rectangle
= 2 (l + b) h
= 2(Length + Breadth) Height
= (Perimeter of the floor) height
[ 2(l + b) = Perimeter of the floor]
1.
If we take the face-vertical cross-section
of the swimming pool, we will have a
trapezium having parallel sides 10 m and
20 m and the height 50 m. (see Note)
Volume of water in the swimming pool
= Area of the vertical cross-section of the
swimming pool width of the swimming
pool

= 10 20 50 15
2

[ Area of the trapezium =

1
(sum of
2

the parallel sides) height]


= 11250 m3
Note: We will have two options of taking
ver tical cr oss-sect ion: (i) Face-Vertical
Cross-Section and (ii) Side-Vertical CrossSection. We take that vertical cross-section
in which area remains the same for every
cross-section. For example, if we take sidevertical cross-section in the above question,
area will be gradually increasing for every
cross-section. Also see Ex. 26.

Area of four walls


= 2(Length + Breadth) Height

2.

Area of four walls


Height = 2(Length Breadth)

3.

Length + Breadth =

4.

Total cost
Area of four walls = Cost of 1 sq unit

Area of four walls


2 Height

Solved Examples
Ex. 28: Find the area of the four walls of a
room whose length is 6 m, breadth 5
m and height 4 m. Also find the cost of
white-washing the walls, if the rate of
whi t e washi ng i s Rs 5 per squar e
m et r e. ( Door s, wi ndows and ot her
openings ignored).
Soln:
Here, l = 6 m, b = 5 m and h = 4 m.
Area of the walls = 2h (l + b)

K KUNDAN
Surface Area of the Walls of a Room
In the previous section, we have learnt the formula
for the surface area of a cuboid and a cube. In this
section, we will obtain a formula for the surface
area of the walls of a room. For this, let us consider
the following.
Make a card-board model of a room. Cut it along
its height through corner and spread it out as
shown in the figure given below, we see that it
has taken the form of a rectangle.
The breadth of this rectangle
= The height (H) of the room

The length of this rectangle


= Length + Breadth + Length + Breadth
of the room
= 2 (Length + Breadth) = 2 (l + b)

= [2 4 (6 + 5)] m2

= (8 11) m2 = 88m2
Cost of white-washing of 1 square metre
= Rs 5.
Cost of white washing the walls
= Rs (5 88) = Rs 440.
Ex. 29: A hall is 21 m long, 14 m broad and
8 m hi gh. It has t wo door s each
2.5 m 2 m and four windows each
1.5 m 1 m, find the cost of colouring
the walls at Rs 15 per sq. m.
Soln:
Area of four walls = 2 (l + b) h
= 2 (21 + 14) 8 sq m
= 2 35 8 sq m = 560 sq m.
Area of two doors = (2.5 2) 2 sq m.
= 10 sq m.
Area of four windows
= (1.5 1) 4 sq m = 6 sq m.
Area of the walls to be coloured
= (560 10 6) sq m
= 544 sq m.
Cost of colouring the walls at Rs 15 per
sq m.
= Rs (544 15) = Rs 8160.

572

Concept of Arithmetic

Ex. 30: A room is 7 metres long and 5 metres


broad, the door and windows occupy 5
sq metres and the cost of papering the
remaining part of the surface of the
walls with paper 75 cm wide, at Rs
46.20 per piece of 13 m is Rs 431.20.
Find the height of the room.
431 .20

Soln:
Number of pieces of paper =
46 .20
Length of the paper

Total area to be cemented


= [280 + 300] m2 = 580 m2
Cost of cementing of 1 square metre
= Rs 12.
Cost of cementing the floor and the walls
= Rs [12 580] = Rs 6960.
Ex. 33: Length of a classroom is two times its

431 .20
364
13 m =
m
=
46 .20
3
364 75

91 sq m
Area of paper =
100
3
Area of walls = (91 + 5 =) 96 sq m
Now, area of walls = 2 (7 + 5) Height
= (24 Height) sq m
According to the question,
24 Height = 96
96
Height =
= 4 metres
24
Ex. 31: A hall, whose length is 16 metres and
breadth twice its height, is needed 168
metres of paper 2 metres wide for its
four walls. Find the area of the floor.
Soln:
Let the breadth = 2h metres, then height
= h metres.
Area of walls = 2 (16 + 2h)h sq metres
Area of paper = (168 2) sq metres
Now, according to the question,
2 (16 + 2h)h = 168 2
(8 + h)h = 84
or, h2 + 8h 84 = 0
or, h2 + 14h 6h 84 = 0
or, h (h + 14) 6 (h + 14) = 0
or, h = 6 and 14
Negative value of h should be omitted.
h = 6
Height = 6 m and
breadth = (6 2 =) 12 m
Area of the floor
= (16 12 =) 192 sq metres
Ex. 32: A swimming pool is 20 m in length, 15
m in breadth, and 4 m in depth. Find
the cost of cementing its floor and walls
at the rate of Rs 12 per square metre.
Soln:
We have,
l = length of the swimming pool = 20m
b = breadth of the swimming pool=15m
h = height of the swimming pool = 4m.
Area of the four walls of the swimming
pool
= 2h(l + b)
= [2 4 (20 + 15)] m2
= 8 35 m2 = 280 m2
Area of the floor of the swimming pool
= l b = 20 15 m2 = 300 m2.

1
times its
2
height. The cost of white-washing the
walls at the rate of Rs 1.60 per sq m is
Rs 179.20. Find the cost of tilling the
floor at the rate of Rs 6.75 per sq m.
Let the height of the classroom be h
metres. Then,
Length = 2h metres and, Breadth
height and its breadth is 1

Soln:

3
h metres.
2

Area of the four walls


= 2 Height (Length + Breadth)

= 2 h 2h h sq m
2

4h 3h
= 2 h
2

sq m

K KUNDAN
7h

= 2 h
sq m = 7h2 sq m.
2

Cost of white-washing of 1 sq m = Rs 1.60


Cost of white-washing of the four walls
= Rs (1.60 7h2) = Rs 11.20h2
But the cost of white washing is given as
Rs 179.20.
11.20h2 = 179.20

179.20
= 16
11.20
or, h2 = 42
or, h = 4
Length of the classroom = (2 4)m = 8m
Breadth of the classroom
or, h2 =

3
= 4 m = 6m
2

Area of the floor of the room


= (8 6) m2 = 48 m2.
Cost of tilling of 1 sq m of the floor
= Rs 6.75
Cost of tilling the floor
= Rs (6.75 48) = Rs 324.
Ex. 34: The cost of papering the walls of a hall
at 35 pai se per square metre is Rs
157.50. The height of the hall is 5 m.
Find its length and breadth, if they are
in the ratio 3 : 2.

573

Mensuration II (Surface Area and Volume)


Soln:

Rate of papering = 35 paise per sq m.


Cost of papering the four walls
= Rs 157.50 = 15750 paise

Let the height of the room be h metres.


Th en ,
Area of four walls
= 2(length + breadth) height
= 2(6 + 9) h m2 = 30 h sq m
Area of 1 door and 2 windows = 8 sq m
Area to be papered
= Ar ea of 4 walls Area of 1 door
and 2 windows
= (30h 8) sq m
Cost of papering walls at Rs 1.40 per
m2 = Rs (30h 8) 1.40
or, (30h 8) 1.40 = 240.80
[ Cost = Rs 240.80 (given)]

15750
sq m
35
= 450 sq m.
or, 2 h (l + b) = 450
or, 2 5 (l + b) = 450
Area of four walls =

450
= 45
10
But l : b = 3 : 2
Let length (l) be 3x m and Breadth (b) be
2x m.
l + b = 45
3x + 2x = 45
or, 5x = 45
or, l + b =

or, (30h 8)
or, 30h 8

45
or, x =
= 9
5
Hence length = (3 9 =) 27 m
and breadth = (2 9 =) 18 m.
Ex. 35: A room is half as long again as it is
broad. The cost of carpeting the room
at Rs 3.25 per sq m is Rs 175.50 and
the cost of papering the walls at Rs
1.40 per sq m is Rs 240.80. If 1 door
and 2 windows occupy 8 sq m, find the
dimensions of the room.
Soln:
Let the breadth of the room be x m. Then,
length

240.80
1.40

24080
140

2408
14
or, 30h 8 = 172
or, 30h = 180
h = 6
Height = 6m
Hence, the dimensions of the room are:
length = 9 m, breadth = 6 m and
height = 6 m
or, 30h 8

K KUNDAN
x

3x
m
= x m
2

2
Area of the room

3x 2
3x

m2 =
= x
sq m
2

Prism

A prism is a solid, whose side faces are parallelograms


and whose ends (or bases) are congruent parallel
rectilinear figures.
In the figure given below, there is a prism whose
ends are rectilinear figures ABCDE and ABCDE .

or, cost of carpeting the room at the rate


of Rs 3.25 per sq m

3x 2

= Rs 2 3.25

or,

3x 2
3.25 = 175.50
2

[ Cost of carpeting = Rs 175.50 (given)]


or, x2 =

175.50 2
351

3 3.25
3 3.25
351 3510

= 36
6.5
65

x = 6
Thus, breadth = 6m and length

3 6
m = 9 m
=
2

Important Terms Related to Prism


(a) Base of a Prism: The end on which a prism
may be supposed to stand is called the base of the
prism.

574

Concept of Arithmetic

In the above figure ABCDE and ABCDE are


the bases of the prism. Every prism has two bases.
( b) Height of a Prism: The per pendicular
distance between the ends of a prism is called the
height of the prism.
In the above figure BF is the perpendicular
distance between the ends ABCDE and ABCDE .
So, it is the height of the prism shown in the
above figure.
(c) Axis of a Prism: The straight line joining
the centres of the ends of a prism is called the
axis of the prism.
In the abov e figure, a straight line passing
through O and O is the axis of the prism.
(d) Lateral Faces: All faces other than the bases
of a prism are known as its lateral faces.
In the above figure ABBA , BC CB, CD DC

Triangular Prism and Right Triangular


Prism
Triangular Prism: A prism is called a triangular
prism if its ends are triangles.
Right Triangular Prism: A right prism is called
a right triangular prism if its ends are triangles.
In other words, a triangular prism is called a
right t riangular prism if its lateral edges are
perpendicular to its ends.
The prism shown in figure (III) is a triangular
prism whereas the prism shown in figure (IV) is a
right triangular prism.

etc are lateral faces.


(e) Lateral Edges: The lines of intersection of
the lateral faces of a prism are called the lateral
edges of the prism.
In the above figure AA , BB, CC, DD and EE
are the lateral edges of the prism.

Regular Prism and Right Prism


Regular Prism: A prism is called a regular prism
if ends are regular figures.
Right Prism: A prism is called a right prism if
its lateral edges are perpendicular to its ends
(bases). Otherwise it is said to be an oblique prism.
The prism shown in figure (I) is an oblique prism
whereas the prism shown in figure (II) is a right
prism.

III

IV

K KUNDAN
A prism is said to be a quadrilateral prism or a
pent agonal pr ism or a hexagonal prism et c
according as the number of sides in the rectilinear
figure forming the ends (base) is four or five or six
etc.
If t he ends of a quadrilater al pr ism ar e
parallelograms, t hen it is also known as a
parallelopiped.
A quadrilateral prism with its ends as squares
is called a rectangular solid or a cuboid.
Figure (V) shows a right pentagonal prism and
figure (VI) shows a rectangular solid.

II
In a right prism, length of the prism is same as
its height. Also, all lateral edges are of the same
length equal to the height of the prism. It is also
evident from the definition of a right prism that
its all lateral faces are rectangles. The number of
lateral edges and lateral faces of a prism is same
as the number of sides in the base of the prism.

VI

575

Mensuration II (Surface Area and Volume)


Volume and Surface Area of a Right Prism
( i ) Volume of a right prism
= Area of the Base Height
( i i) Lateral Surface area of a right prism
= Perimeter of the Base Height
( i ii ) Total Surface area of a right prism
= Lateral Surface area + Area of Ends
= Lateral Surface area + 2 (Area of the
Base)
( iv) A Particular Case : If the base of a right
prism is an equilateral triangle of side a
and height h, then
Lateral surface area = 3a h
Total surface area = 3a h

Volume =

3 2
a
2

3 2
a h
4

Total surface area


= Lateral surface area + Area of ends

= 576 2 4 8 cm2

= 576 32 3

cm2

= 631.424 cm 2
Ex. 38: The perimeter of the base of a right
pri sm st andi ng on an equi lat eral
triangle is 12 cm. If the length of the
prism be 60 cm, find its volume and
surface area.
Soln:
Let t he lengt h of each side of t he
triangular base be a cm. Then,
Perimeter = 12 cm
or, 3a = 12 cm
a = 4 cm
Area of the base =

Solved Examples
Ex. 36: Find the area of the base of a right
triangular prism having volume of 672
cm3 and height 8 cm.
Soln:
We have,
Volume = Area of the base Height

3
a2
4

= 4 4 cm2

[ 3 = 1.732]
Volume = Area of the base Height
= 6.928 60 cm3
[ Height = Length = 60 cm]
= 415.68 cm3
Lateral surface area
= Perimeter of the base Height
12 60 cm2 = 720 cm2
Total surface area
= Lat eral sur f ace ar ea + 2(Ar ea of
the base)
= (720 + 2 6.928) cm2
= 733.856 cm 2
Ex. 39: A right prism of height 20 cm stands
on a triangular base whose sides are
13 cm, 14 cm and 15 cm, find its lateral
surface area, total surface area and
volume.
Soln:
If a, b, c are the lengths of the sides of a
triangle and s is the semi-perimeter, then
its area A is given by
= 6.928 cm2

K KUNDAN
or, Area of the base =

Volume
Height

672
cm2 = 84 cm2
8
Ex. 37: The base of a ri ght pr i sm i s an
equilateral triangle with a side 8 cm
and its height is 24 cm. Find its volume,
lateral surface area and total surface
ar ea.
Soln:
We know that the volume V of a right
triangular prism is given by V = Area of
the base Height.
Since base is an equilateral triangle with
a side 8 cm. Therefore,
or, Area of the base =

Area of the base =

3
(side )2
4

= 4 8 cm2 = 16 3 cm2

= 27.712 cm 2

V = 16 3 24 cm3
= 384 3 cm3 = 665.088 cm3
Lateral surface area
= Perimeter of the base Height
= (8 + 8 + 8) 24 cm2 = 576 cm2

A =

s (s a ) (s b ) (s c )

1
(a b c )
2
Here, a = 13 cm, b = 14 cm and
c = 15 cm.
where s

s =

1
13 14 15 = 21 cm.
2

Thus,
Perimeter of the base = 2s = 42 cm.

576

Concept of Arithmetic
Area of the base
=

s (s a ) (s b ) (s c )

21(21 13) (21 14) (21 15 )

21 8 7 6

738732

7 2 32 42
= 7 3 4 cm2 = 84 cm2
Lateral surface
= Perimeter of the base Height
= (42 20) cm2 = 840 cm2
Total surface area
= Lat eral sur f ace ar ea + 2(Ar ea of
the base)
= (840 + 2 84) cm2 = 1008 cm2
Volume = Area of the base Height
= (84 20) cm3 = 1680 cm3
Ex. 40: A right prism stands on a triangular
base. The volume of the prism is 630
cm 3 and the sides of the base are 5
cm, 5 cm and 8 cm. Find the height of
the prism.
Soln:
If a, b, c denote the lengths of the sides of
a triangle, then its area A is given by
A =

s (s a ) (s b ) (s c )

Now, Area of the base =

3
side 2
4

52 cm2 25 3 cm 2
4

Volume of the prism


= Area of the base Height

25 3

= 4 50 cm3

= 541.25 cm 3
Ex. 42: A right triangular prism of height 18
cm and of base sides 5 cm, 12 cm and
13 cm is transformed into another right
triangular prism on a base of sides 9
cm, 12 cm and 15 cm. Find the height
of new prism and the change in the
whole surface area.
Soln:
We have,
2s = Perimeter of the base
= (5 + 12 + 13) cm = 30 cm
s = 15 cm.
Here, a = 5 cm, b = 12 cm and
c = 13 cm
Area of the base
=

s (s a ) (s b ) (s c )

K KUNDAN
1
(a b c )
2
Here, a = 5 cm, b = 5 cm and c = 8 cm.
where s

s =

or, A =

1
(5 5 8) = 9 cm
2

9 (9 5) (9 5) (9 8 )

= 9 4 4 1 = 12 cm2
Now,
Volume = Area of the base Height
or, Height =

Volume
Area of the base

630
cm 52.5 cm
12
Ex. 41: The base of a ri ght pr i sm i s an
equilateral triangle of side 5 cm. If the
lateral surface area of the prism is 750
cm2, find its volume.
Soln:
We have,
Lateral surface area
= Perimeter of the base Height
or, 750 = (5 + 5 + 5) Height
Height =

or, Height =

750
cm 50 cm
15

2
2
15 10 3 2 cm = 30 cm
Let V1 be the volume of the prism. Then,
V1 = Area of the base Height
or, V1 = (30 18) cm3 = 540 cm3
Let S be the total surface area of the prism.
Th en ,
S = Lat eral sur f ace ar ea + 2(Ar ea of
the base)
= Perimeter of t he base height +
2(Area of the base)
= (30 18 + 2 30) cm2 = 600 cm2
Let h be the height of the new prism and
2s 1 be the perimeter of its base.
Th en ,
2s 1 = 9 + 12 + 15 = 36
or, s 1 = 18
The sides of the base are a = 9 cm,
b = 12 cm and c = 15 cm.
Area of the base

s1 (s1 a ) (s1 b )(s1 c )

= 18 9 6 3 cm2 = 54 cm2
Volume = Area of the base Height
or, 540 = 54 h
[Volume of the two prisms are same]
or, h = 10 cm
Thus, the new prism is of height 10 cm.
Let S1 be the total surface area of the new
prism. Then,

577

Mensuration II (Surface Area and Volume)


S 1 = Perimeter of the base height +
2(Area of the base)
= (36 10 + 2 54) cm2 = 468 cm2
Change in the whole surface area
= S S1
= (600 468) cm2 = 132 cm2
Ex. 43: The base of a right triangular prism
is an equilateral triangle. If its height
is halved and each side of the base is
doubled, find the ratio of the volumes
of the two prisms.
Soln:
Let a be the length of each side of the
base of the giv en prism and h be it s
height. Then, its volume V 1 is given by

3 2

V1 = 4 a h

h
2
If V2 is the volume of the new prism, then,
a1 = 2a

V2 =

and h1 =

3 2
3
h
a1 h1
2a 2
4
4
2

2
30 20 6 4 cm

5 6 5 4 6 4 cm2

= 5 6 4 cm2 = 120 cm2


Volume of the prism
= Area of the base Height
= 120 50 cm3 = 6000 cm3
Total surface area = Perimeter of the base
height + 2(Area of the base)
= (60 50 + 2 120) cm2
= 3240 cm2
Ex. 45: The t ot al sur face ar ea of a r ight
triangular prism of the height 4 cm is

Soln:

Let a1 be the length of each side of the


base of the new prism and h1 be its height.
Th en ,

2
72 3 cm . If the base of the prism is
an equilateral triangle, find its volume.
Let each side of the base of the prism be a
cm. Then,

total surface area = 72 3 cm2


or, Perimeter of the base height + 2(Area
of the base) = 72 3

3 2

or, 3a 4 2 4 a 72 3

3a 2 24a 144 3 0

or,

K KUNDAN
3 2

= 2 a h

or, a 2 8 3a 144 0

or, a 2 12 3a 4 3a 144 0

3 2
a h
V1
1
4

V2
2
3 2
a h
2

or, a 4 3 a 12

or, V1 : V1 = 1 : 2
Ex. 44: The perimeter of the base of a right
triangular prism is 60 cm and sides of
the base are in the ratio 5:12:13. Find
its volume and total surface area, if
its height is 50 cm.
Soln:
Let a, b, c be the lengths of the sides of
the base of the prism.
Th en ,
a : b : c = 5 : 12 : 13
or, a = 5x, b = 12x and c = 13x
or, a + b + c = 30x
or, 60 = 30x
[ a + b + c = 60 cm (given)]
x = 2
a = 10, b = 24 and c = 26.
We have, 2s = 60.
x = 30.
Area of the base =

3 0

or, a a 12 3 4 3 a 12 3 0

s (s a ) (s b ) (s c )

30 (30 10 ) (30 24 ) (30 26)

or, a 4 3 0

[ a 12 3 0 as a 0]
or, a 4 3
Volume of the prism
= Area of the base Height
=

4 cm

3
4 3
4

= 48 3 cm3

Right Circular Cylinder


A right circular cylinder is a solid described by
the revolution of a rectangle about one of its sides
which remains fixed.
Thus if the rectangle ABCD revolves about the
side AB, it describes the cylinder shown in the
figure. AB is called the axis of the cylinder. The
circles described by AD and BC are called the ends.
Either end on which the cylinder may be supposed
to stand, is called base. The height of a cylinder

578
is the length of axis AB. The radius of the circular
base is called the radius of the cylinder.

Concept of Arithmetic
Total surface area of a cylinder
= Curved surface + Area of two ends
= 2rh + 2r2 = 2r(h + r)

(iii) Volume and Surface Area of a Hollow


Cylinder

Curved or Lateral Surface

A solid bounded by two coaxial cylinders of the same


height and different radii is called a hollow cylinder.
Let R and r be the external and internal radii of
a hollow cylinder and h be its height as shown in
the figure given below. Then we have the following
results:

The curved surface joining the two bases of a right


circular cylinder is called its lateral or curved
surface.

Volume and Curved Surface (Area) of a


Cylinder
(i) Volulme of a Cylinder

(a) Each base surface areas = (R2 r2) sq units


(b) Curved (lateral) surface area
= (Exter nal sur f ace area) + (Int ernal
surface area)
= 2Rh + 2rh = 2h(R + r) sq units
(c) Total surface area

K KUNDAN

If we take a right prism with a regular polygon for


base, and imagine the number of sides of the
polygon to increase indefinitely, then the polygon
ultimately becomes a circle and t he pr ism a
cylinder.
Hence, volume of a cylinder
= (area of base) height
= 2rh; where r is the radius of the circular
base and h is the height of the
cylinder.

(ii) Curved or Lateral Surface Area of a


Cylinder

Imagine a hollow cylinder t o be made of thin


cardboard, and cut along any straight line on its
surface parallel to its axis, then it can be spread
out into a rectangle whose sides are respectively,
the circumference and height of the cylinder.
Thus curved surface area
= circumference height
= 2rh

= 2Rh 2rh 2 (R 2 r 2 )

= 2h (R r ) 2(R r ) (R r )

= 2 (R r ) (h R r ) sq units
(d) Volume of the material
= Exterior volume - Interior volume
= R 2h r 2h = h (R 2 r 2 )

Solved Examples
Ex. 46: The ci rcum fer ence of the base of a
cylinder is 132 cm and its height is
25 cm. Find the volume of the cylinder.
Soln:
Let r cm be the radius of the cylinder.
Th en ,
Circumference = 132 cm
or, 2r = 132 cm
or, 2

22
r 312
7

132 7
cm
2 22
r = 21 cm.
or, r =

579

Mensuration II (Surface Area and Volume)


We have, h = height of the cylinder
= 25 cm.
Volume of the cylinder = r 2h

22

21 21 25 cm3
=
7

= 34650 cm3
Ex. 47: The diameter of a garden roller is 1.4
m and it is 2 m long. How much area
will it cover in 5 revolutions?
(Use = 22/7)
Soln:
Clearly,
Area covered = Curved surface
Number of revolutions.

1 .4
m = 0.7 m and h = 2 m.
2
Curved surface = 2rh sq m
Here, r =

22
0.7 2 sq m
7
= 8.8 sq m
Hence, area covered
= Cur ved sur f ace ar ea Number of
revolutions
= (8.8 5) m2 = 44 sq m
Ex. 48: A rectangular sheet of paper 44 cm
18 cm is rolled along its length and a
cylinder is formed. Find the volume of
the cylinder.
(Use = 22/7)
Soln:
When the rectangular sheet is rolled along
its length, we find that the length of the
sheet forms the circumference of its base
and breadth of the sheet becomes the
height of the cylinder.
Let r cm be the radius of the base and h
cm be the height. Then, h = 18 cm.
Now, circumference of the base
= length of the sheet
Circumference = 44 cm
= 2

Ex. 49: The volume of a cylinder is 448 cm 3


and hei ght 7 cm . Fi nd i t s l at er al
surface area and total surface area.
Soln:
Let the radius of the base and height of
the cylinder be r cm and h cm respectively.
Then, h = 7 cm (given).
Now, Volume = 448 cm 3
or, r 2h 448
[ h = 7 cm]

or, r 2 7 448

448
64
7
or, r = 8 cm
Lateral surface area = 2rh cm2
2
or, r

= 2

22
8 7 cm2 = 352 cm2
7

Total surface area = 2rh 2r 2

cm2

= 2r h r cm2
= 2

22
87 8 cm2
7

5280
cm2 = 754.28 cm2
7
Ex. 50: If the radius of the base of a right
circular cylinder is halved, keeping the
height same, what is the ratio of the
volume of the reduced cylinder to that
of the original.
Soln:
Let r be the radius of the base and h be
the height of the given cylinder.
Then, radius of the base and the height
=

K KUNDAN
of the reduced cylinder ar e

r
and h
2

respectively.
Let V1 and V2 be the volumes of the given
cylinder
and
reduced
cylinder
respectively. Then,
V1 = r 2h cubic units, and,
2

2
r
V2 = h r h cubic units
4
2
or, 2r = 44

V1
r 2h
4
V
(r 2 / 4)h
2

22
r 44
or, 2
7
r = 7 cm
Volume of the cylinder = r 2h cm

22
(7)2 18 cm3
7
= 2772 cm3
=

V2
1
or, V 4
1

V2 : V1 = 1 : 4.
Ex. 51: Find the number of coins, 1.5 cm in
diameter and 0.2 cm thick, to be melted
to form a right circular cylinder of
height 10 cm and diameter = 4.5 cm.

580
Soln:

Concept of Arithmetic
Clearly, each coin is a cylinder of radius.
r = 0.75 cm height h = 0.2 cm.
Therefore,

Volume of a coin = (0.75) 0.2 cm


= ( (2.25)2 10) cm3
Number of coins

Volume of the cylinder


Volume of a coin

2.25 2 10
0.75 2 0.2

= (3r )2 4h 36 r 2h cubic units

Volume of the cylinder

V2 = Volume of the cylinder of height 4h


and radius 3r

2.25 2.25 10
0.75 0.75 0.2

= 3 3 50 = 450
Ex. 52: A glass cylinder with diameter 20 cm
has water to a height of 9 cm. A metal
cube of 8 cm edge is immersed in it
compl etely. C alculate the height by
which water will rise in the cylinder.
(Take = 3.142)
Soln:
Suppose the water rises by h cm. Clearly,
water in the cylinder forms a cylinder of
height h cm and radius 10 cm.
Volume of the water displaced
= Volume of the cube of edge 8 cm
or, r 2h 83
or, 3.142 102 h = 8 8 8
[ r = 10 cm]

V1
20 r 2h
5

V
9
36 r 2h
2

V1 : V2 = 5 : 9
Ex. 54: 2.2 cubic dm of brass is to be drawn
i nto a cyl indr ical wi r e 0.50 cm i n
diameter. Find the length of the wire.
Soln:
Let the length of the wire be l cm.
Clearly, wire forms a cylinder of radius
0.25 cm and height l cm.
Volume of the wire

22

(0.25 )2 l cm3
=
7

But, volume given


= 2.2 dm3
= (2.2 103) cm3
[ 1 dm = 10 cm]
[ 1 dm3 = (10 10 10) cm3 = 103 cm3]
= (2.2 1000) cm3 = 2200 cm3

22
0.25 0.25 l 2200
7

or,

22 25
25

l 2200
7 100 100

or,

22 1 1
l 2200
7 4 4

K KUNDAN

888
or, h =
= 1.6
3.142 10 10
Hence, the required height = 1.6 cm
Ex. 53: The radii of two right circular cylinders
are in the ratio 2 : 3 and their heights
are in the ratio 5 : 4. Calculate the ratio
of their curved surface areas and also
the ratio of their volumes.
Soln:
Let the radii of two cylinders be 2r and 3r
respectively and their heights be 5h and
4h respectively. Let S1 and S2 be the curved
surface areas of the two cylinders and V1
and V2 be their volumes.
Th en ,
S1 = Curved surface area of the cylinder of
height 5h and radius 2r
= 2 2r 5h 20rh sq units
S2 = Curved surface area of the cylinder of
height 4h and radius 3r
= 2 3r 4h 24 rh sq units
S1
20 rh
5
S 24 rh 6
2
S1 : S2 = 5 : 6
V1 = Volume of the cylinder of height 5h
and radius 2r

= (2r )2 5h 20 r 2h cubic units

2200 4 4 7
cm
22
= 11200 cm = 112 m.
Hence, the length of the wire is 112 m.
Ex. 55: A solid cylinder has total surface area
of 462 square cm. Its curved surface
area is one-third of its total surface
area. Find the volume of the cylinder.
(Take = 22/7)
Soln:
Let r be the radius of the base and h be the
height of the cylinder.
Th en ,
l =

Total surface area = 2r (h r ) cm2


Curved surface area = 2rh cm2
Now, curved surface area
=

1
(Total surface area )
3

or, 2rh

1
2r (h r )
3

or, 6rh 2rh 2r 2


or, 4rh 2r 2
or, 2h = r

581

Mensuration II (Surface Area and Volume)


Total surface area = 462 cm2
2r (h r ) 462

or, 2r r 462
2

2h r h 2

or, 2r

3r
462
2

22 3 2
r 462
7 2
or, r2 = 49
r = 7 cm.
or, 2

r
7

cm
2 2
Hence, volume of the cylinder
Now, 2h r h

7
22
2
72 cm3
= r h
2
7
= 539 cm3.
Ex. 56: A well with 10 m inside diameter is
dug 14 m deep. Earth taken out of it is
spread all around to a width of 5 m to
form an embankment. Find the height
of embankment.

Ex. 57: A cylindrical pipe has inner diameter


of 7 cm and water flows through it at
192.5 litres per minute. Find the rate
of flow in kilometres per hour.
Soln:
Volume of water that flows per hour
= (192.50 60) litres
= (192.50 60 1000) cm3 ..... (i)
Inner diameter of the pipe = 7 cm.
or, Inner radius of the pipe

7
cm = 3.5 cm.
2
Let h cm be the length of the column of
water that flows in one hour. Clearly,
water column forms a cylinder of radius
3.5 cm and length h cm.
Volume of water that flows in one hour
= Volume of the cylinder of radius 3.5 cm
and length h cm
=

22

3.5 2 h cm3
=
7

From (i) and (ii), we get

...... (ii)

22
3.5 3.5 h = 192.50 60 1000
7
192 .50 60 1000 7
cm
or, h =
22 3.5 3.5

= 300000 cm = 3 km.
Hence, the rate of flow of water is 3 km
per hour.
Ex. 58: Water is flowing at the rate of 3 km/hr
t hr ough a ci r cul ar pi pe of 20 cm
internal diameter into circular cistern
of diameter 10 m and depth 2 m. In
how much time will the cistern be filled?
Soln:
Suppose the cistern is filled in x hours.
Since water is flowing at the rate of 3
km/hr. Therefore, length of the water
column in x hours = 3x km = 3000x metres.
Clearly, the water column forms a cylinder
of radius

K KUNDAN

Soln:

Volume of the earth dugout = (r 2h ) m3


=

22
5 5 14 m3 = 1100 m3
7

Area of the embankment (shaded region)


= (R r 2 )
= (102 562 ) m2

22
75 m2
=
7
Height of the embankment
=

Volume of the earth dugout


Area of the embankment
1100
7 1100

= 4.66 m.
22
22 75
75
7

20
1
cm = 10 cm =
m
2
10
and h = height (length) = 3000x metres.
Volume of the water that flows in the
r =

cistern in x hours = r 2h

1
22 1

3000 x m3
=
7 10 10

Also, volume of the cistern


22

5 5 2 m3
=
7

[ r = 5 m, h = 2 m]
Since the cistern is filled in x hours.

582

Concept of Arithmetic
Volume of the water that flows in the
cistern in x hours is equal to the volume
of the cistern.

22 1
1
22

3000x
552
7 10 10
7

or,

5 5 2 10 10
hours
or, x =
3000

5
=
hours = 1 hours 40 minutes.
3
Ex. 59: Water is flowing at the rate of 7 metres
per second through a cir cul ar pi pe
whose internal diameter is 2 cm into a
cylindrical tank the radius of whose
base is 40 cm. Determine the increase
in the water level in half an hour.
Soln:
Rate of flow of water
= 7m/sec = 700 cm/sec.
Length of the water column in

1
hours
2

= (700 30 60) cm
Internal radius of circular pipe = 1 cm.
Clearly, water column forms a cylinder of
radius 1 cm and length (700 30 60)
cm.
Volume of the water that flows in the

Soln:

on an average. How many words would


use up a bottle of ink containing one
fifth of a litre?
Volume of barrel

22

=
0.25 0.25 7 cm 3 = 1.375 cm3
7

Volume of ink in the bottle

1
1000
litre =
cm3 = 200 cm3
5
5
Total number of barrels that can be filled
from the given volume of ink
=

200
.
1.375
So, required number of words
=

200
330 = 48000.
1.375
Ex. 61: The volume of a metallic cylindrical pipe
is 748 cm3. Its length is 14 cm and its
ext er nal radi us i s 9 cm . Fi nd i t s
thickness.
Soln:
We have,
R = external radius of pipe = 9 cm
h = length of the pipe = 14 cm
V = volume of the pipe = 748 cm3
Let r be the internal radius in centimetres.
Th en ,
Volume = 748 cm3
=

K KUNDAN
tank in

1
hour
2

or, (R 2 r 2 )h 748 cm3

22

1 700 30 60 cm3
=
7

.... (i)

[Using V = r 2h ; where r = 1 cm,


h = 700 30 60 cm]
Let h cm be the rise in the level of water
in the tank. Then,
volume of the water in the tank
=

22
40 40 h cm3 .... (ii)
7
[ r = 40 cm]

From (i) and (ii), we get

22
22
40 40 h
1 700 30 60
7
7
700 30 60
cm = 787.5 cm
or, h =
40 40

Hence, the rise in the level of water in

1
hour is 787.5 cm.
2
Ex. 60: The barrel of a fountain-pen, cylindrical
in shape, is 7 cm long and 5 mm in
diameter. A full barrel of ink in the pen
will be used up on writing 330 words
the tank in

or,

22 2
(9 r 2 ) 14 748
7

748
44
or, 81 r2 = 17
or, r2 = 64
r = 8 cm
Hence, thickness of the pipe
= (R r) cm = (9 8) cm = 1 cm.
Ex. 62: The difference between outside and
inside surfaces of a cylindrical metallic
pipe 14 cm long is 44 cm2. If the pipe is
made of 99 cu centimetres of metal, find
the outer and inner radii of the pipe.
Soln:
Let R cm and r cm be the external and
inter nal r adii of t he met allic pipe
respectively.
We have h = length of the pipe = 14 cm.
Now, it is given that outside surface area
- Inside surface area = 44 cm2
or, 2Rh 2rh 44
2
or, 81 r

or, 2(R r )h 44
or, 2

22
(R r ) 14 44
7

583

Mensuration II (Surface Area and Volume)


1
...... (i)
2
It is given that the volume of the metal
used = 99 cubic centimetres.
External volume Internal volume
= 99 cubic centimetres
or, R r =

or, R h r h 99

or,

22
R r R r 14 99
7
22
1
(R r ) 14 99
7
2

or, R + r =

Let r dm be the radius of the base and h


dm be the height of the cylindrical tank.
Then, h = 6r (given)
Total surface area
= 2r (r h ) 2r (r 6r ) 14r 2
Cost of painting

60
42 2
r
= Rs
100
5
It is given that the cost of painting is Rs
237.60.
2
= Rs (14r )

or, (R 2 r 2 )h 99
or,

Soln:

[Using (i)]

99
22

9
..... (ii)
2
Solving equations (i) and (ii), we get,
R = 2.5 and r = 2.
Hence, outer radius = 2.5 cm and inner
radius = 2 cm.
Ex. 63: An iron pipe 20 cm long has exterior
diam eter
equal t o 25 cm . I f the
thickness of the pipe is 1 cm, find the
whole surface of the pipe.
Soln:
We have
R = external radius = 12.5 cm
r = internal radius
= (external radius thickness)
= (12.5 1) cm = 11.5 cm
h = length of the pipe = 20 cm
Total surface area of the pipe = (External
curved surface) + (Internal curved surface)
+ 2(Area of the base of the ring)
R + r=

or,

42 2
r 237.60
5
42 22 2

r = 237.60
5
7

or, r2 = 237.60

5
7

= 9
42 22

r = 3 dm
h = 6r = 18 dm.
Hence, volume of the cylinder = r 2h
= ( 3 3 18) dm3

22

9 18 dm3
=
7

= 509.14 dm 3
Ex. 65: A lead pencil consists of a cylinder of
wood with a solid cylinder of graphite
filled into it. The diameter of the pencil
is 7 mm, the diameter of the graphite
is 1 mm and the length of the pencil is
10 cm. Calculate the weight of t he
whole pencil, if the specific gravity of
the wood is 0.7 gm/cm3 and that of the
graphite is 2.1 gm/cm3 .
Soln:
Diameter of the graphite cylinder

K KUNDAN
= 2Rh 2rh 2 (rR 2 r 2 )
= 2(R r )h 2(R2 r 2 )
= 2(R r )h 2(R r )(R r )
= 2(R r )(h R r )
= 2

22
(12.5 11.5 ) (20 12.5 11.5 ) cm 2
7

22
24 21 cm2
= 2
7
= 3168 cm2
Ex. 64: The cost of painting the total outside
surface of a closed cylindrical oil tank
at 60 paise per sq dm is Rs 237.60.
The height of the tank is 6 times the
radius of the base of the tank. Find
i ts v ol um e corr ect t o t wo decim al
places.

= 1 mm =

1
cm.
10

1
cm.
20
Length of the graphite cylinder = 10 cm
Volume of the graphite cylinder
Radius =

1
22 1

10 cm3
=
7 20 20

Weight of graphite
= Volume Specific gravity
1
22 1

10 2.1 gm
=
7 20 20

1
21
22 1

10
gm
=
10
7 20 20
= 0.165 gm.

584

Concept of Arithmetic

Diameter of pencil = 7 mm =

7
cm.
10

7
cm
20
and, length of pencil = 10 cm.
Volume of pencil
Radius of pencil =

7
22 7

10 cm3
=
7 20 20

Volume of wood
7
22 1
1
1
22 7
3

10

cm
=
7 20 20 10
7 20 20

22 1
1

10(7 7 1) cm3
7 20 20

11 1

48 cm3
7 20
Weight of wood
=

= 11 1 48 0.7 gm
7 20

Volume of wire
2
2

1200 cm 3
=
10 10

= 482 cm3
So, weight of wire = (482 8.88) gm
= 426.242 gm.

Pyramid
A pyramid is a solid whose base is a plane rectilinear
figure and whose side-faces are triangles having a
common vertex outside the plane of the base.
Figure given below shows a pyramid VABCDE.
The base of this pyramid is the pentagon ABCDE
and triangles VAB, VBC, VCD, VDE and VEA are
five faces.
If t he base of a pyr amid is a t r iangle, a
quadr ilat er al and a squar e, t hen it is called
triangular pyramid, quadrilateral pyramid and
square pyramid respectively. Similarly, a pyramid
is called a pentagonal, hexagonal, septagonal and
octagonal according as the number of sides of the
base is 5, 6, 7 or 8.

11 1
7
=

48
gm
10
7 20
= 2.64 gm
Total weight = (2.64 + 0.165) gm
= 2.805 gm.
Ex. 66: A copper wire 4 mm in diameter is evenly
wound about a cylinder whose length
is 24 cm and diameter 20 cm so as to
cover the whole surface. Find the length
and weight of the wire assuming the
specific gravity to be 8.88 gm/cm3 .
Soln:
Clearly, one round of wire covers 4 mm

K KUNDAN
4
cm in thickness of the surface of
10
the cylinder and length of the cylinder is
24 cm.
Number of rounds to cover 24 cm
=

24
24 10
= 4 /10 =
= 60.
4

Diameter of the cylinder = 20 cm.


Radius of the cylinder = 10 cm.
Lengt h of t he wir e in complet ing one
round
= 2r 2 10 cm = 20 cm.
Lengt h of t he wir e in covering the
whole surface = Length of the wire in
completing 60 rounds
= (20 60) cm = 1200 cm.
Radius of copper wire = 2 mm =

2
cm.
10

Figure I

Important Terms Related to Pyramid


(i) Vertex: The common vertex of the triangular
faces of a pyramid is called the vertex of the
pyramid.
In the above figure V is the v ert ex of t he
pyramid VABCDE.
(ii) Height: The height of a pyramid is the length
of the perpendicular from the vertex to the base.
In the above figure VP is the height of the
pyramid VABCDE.
(iii) Axis: The axis of a pyramid is the straight
line joining the vertex to the central point of the
base.
In the above figure VO is the axis of the pyramid
VABCDE.
(iv) Lateral Edges: The edges through the
vertex of a pyramid are known as its lateral edges.
(v) Slant Height: The slant height of a regular
right-pyramid is the line segment joining the vertex

585

Mensuration II (Surface Area and Volume)


to the mid-point of anyone of the sides of the base.
The figure given below shows a right regular
pyramid, in which O is the centre of the base and
VM is the slant height.

A tetrahedron whose all the edges are of equal


length is called a regular tetrahedron. In a regular
t et r ahedr on all t he f our faces ar e congr uent
equilateral triangles.
1. When the length of each edge of a regular
tetrahedron is given, we have the following
results:
( i ) Height of the regular tetrahedron
2
(edge).
3
( i i) Slant height of the regular tetrahedron

3
(edge).
2
( i ii ) Volume of the regular tetrahedron

2
(edge)3.
12
( iv) Later al surf ace ar ea of
tetrahedron

Figure II
Also, in right angled triangle VOM, we have
VM2 = OM2 + VO2 [By Pythagoras theorem]
VM

VO2 OM2

Right Pyramid and Regular Pyramid


(i) Right Pyramid: A pyramid is said to be right
pyramid if the perpendicular dropped from the
vertex on the base meets the base at its central
point ie the centre of the inscribed or circumscribed
circle. In other words, the vertex of the pyramid
lies on t he perpendicular t o t he base drawn
thr ough its centre. Other wise, the pyramid is
called an oblique pyramid.
The pyramid shown in the above figure I is an
oblique pyramid whereas figure II given above
shows a right pyramid.
(ii) Regular Pyramid: A pyramid is said to be a
regular if its base is a regular figure ie all sides of
its base are equal.
In case of a right regular pyramid the lateral
edges are equal and the lateral faces are congruent
triangles.

t he

r egular

3 3
(edge)2.
4

(v) Total surface area of the regular tetrahedron


2

= 3 (edge) .
2 . For a right pyramid with an equilateral
triangle of side a as base and height h, we
have the following results:

K KUNDAN

Volume and Surface area of a Pyramid

( i ) Lateral edge or Lateral height =

( i i) Slant height =

Tetrahedron and Regular Tetrahedron


A tetrahedron is a pyramid whose base is a triangle.
It has six edges and four triangular faces.

a2
3

a2
12

( i ii ) Lateral surface area

1
(Perimeter of the base slant height)
2
( iv) Total surface area
=

1
(Perimeter of the base slant height)
2

( i ) Volume of a pyramid

1
=
Area of the Base Height
3
( i i) Lateral surface area of a pyramid
= Sum of ar eas of all t he lat er al
triangular faces.
( i ii ) Total surface area of a pyramid
= Sum of areas of all lateral faces + Area
of the base.

h2

h2

3 2
a .
4

1
3
3 2

a2 h2
a h .
3
4
12
( vi) Area of lateral face

(v) Volume =

1
(Length of an edge of the base Slant
2
height)

586

Concept of Arithmetic

Solved Examples
Ex. 67: Find the volume of the right pyramid
the area of whose base is 60 cm 2 and
height 10 cm.
Soln:
We know that the volume of a right pyramid

1
(Area of the base) (Height)
3
Volume of the given pyramid
=

= 60 10 cm3 = 200 cm3


3

Ex. 68: Find the height of the right pyramid


whose volume is 750 cm 3 and area of
whose base is 150 cm 2 .
Soln:
We know that the volume of a right pyramid

1
(Area of the base Height)
3
Let h be the height of the giv en right
pyramid. Then,
1
750 150 h
3

Ex. 71: Find the lateral surface area and whole


surface area of a ri ght pyr ami d in
whi ch t he base i s an equi lat eral

Soln:

triangle of area 16 3 cm2 and length


of each lateral edge is 5 cm.
Let the length of each side of the base be
a and h be the height of the pyramid.
Th en ,
Area of the base = 16 3 cm2
3 2
a 16 3
4
or, a2 = 64
a = 8 cm
Now, Lateral edge = 5 cm

or

or,

h2

a2
5
3

or,

h2

64
5
3

750 3
cm = 15
150
Ex. 69: A right pyramid has
equilateral triangle of
its height is 30 3 cm.
of the pyramid.
Soln:
We have,
or, h =

cm
its base as an
side 40 cm and
Find the volume

2
or, h

64
25
3

2
or, h 25
2
or, h

64
3

75 64
3

K KUNDAN
3 2

Volume of the pyramid = 12 a h

Here, a = 40 and h = 30 3 .
Volume of the pyramid

= 12 40 30 3 cm3

= 12000 cm 3
Ex. 70: Find the vol ume of a regular
t et r ahedr on whose each edge i s of
6 2 cm.
Soln:
We know that
Volume of a regular tetrahedron
2
(edge )3
=
12
Volume of the given tetrahedron

2
6 2
12

cm 3

2
216 2 2 cm 3
12
= 72 cm3

11
cm
3
Slant height

h2

a2

12

11 64

9 3 cm
3 12

Now, Lateral surface area


=

1
(Per imeter of the base Slant
2
height)

1
8 8 8 3 cm2 = 36 cm2
2
Whole surface area
= Ar ea of t he base + Lat eral sur f ace
area
=

= 16 3 36 cm2
Ex. 72: If p be the length of the perpendicular
drawn fr om a ver t ex of a regular
tetrahedron to its opposite face and
each edge of length 2a, show that 3p2
= 8a2 .
Soln:
Clearly, p = height of the tetrahedron
=

2
(length of an edge )
3

587

Mensuration II (Surface Area and Volume)

Ex. 75: A r i ght pyr am id stands on an


equil ateral t riangular base of ar ea

2
2a
3

or, p
2
or, p

8a
3

Soln:
3 p 2 8a 2
Ex. 73: Find the volume of a tetrahedron the
sides of whose base are 9 cm, 12 cm
and 15 cm and height 20 cm.
Soln:
Let a = 9 cm, b = 12 cm, c = 15 cm. Then,
2s = a + b + c
or, 2s = 9 + 12 + 15 = 36
s = 18
Area of the base
=

Then, area of the base = 16 3 cm2


3 2
a 16 3
4
or, a2 = 64
a = 8 cm
Let h be the height of the pyramid and l
be its slant height. Then,

or,

s (s a ) (s b ) (s c )

18 (18 9) (18 12) (18 15 )

= 18 9 6 3 = 54 cm2
Volume of the tetrahedron
=

2
16 3 cm . If the area of one of its
lateral faces is 40 cm2, find the volume
of the pyramid.
Let the length of each side of the base be
a cm.

l h2

a2
12

2
2
or, l h

1
(Area of the base Height)
3

1
54 20 cm3 = 360 cm3
3
Ex. 74: Show that the surface area and volume
of a regular tetrahedron of height h
=

a2
12

l2 h2

or,

64
12

16
...(i)
3
Now, Area of one lateral surface = 40 cm2
2
2
l h

K KUNDAN
3 3 2
3 3
h
h respectively.
and
2
8
We know that if the length of each edge of a
regular tetrahedron is a units, then

are

Soln:

height of the tetrahedron =


or, h
a

2
a
3

2
a
3

1
a l 40
2
or, a l = 80
or, 8 l = 80
l = 10
Putting l = 10 in (i), we get

100 h 2

16
3

2
or, h 100

3
h
2

Surface area of the tetrahedron


=

3 (edge )2

3
3
h
2

3 3 2
h
=
2

Volume of the tetrahedron

2 3
2
edge 3 = 12 2 h
=
12

3 3 3
3 3
h
h
=
24
8

16
3

284
3
Volume of the pyramid

1
(Area of the base Height)
3

1
284
16 3
cm3
3
3

16
284 cm3
3

Ex. 76: Area of a regular hexagon is 216 3


sq units. A pyramid of the height 6
uni t s i s for m ed upon the regular

588

Soln:

Concept of Arithmetic
hexagon. Find the slant surface area
of the pyramid.
Let each side of the regular hexagon be a
units.
area of the regular hexagon =

3 3 2
a
2

3 3 2
a 216 3
2
or, a2 = 144
a = 12 units.

or,

Let O be the centre of regular hexagon.


Then, OOG 90 and
right-angled triangle.
OG

OG

is t he

OO2 OG 2

OO is given, now we have to find OG .


Six equal triangles can be drawn in a
regular hexagon and area of each triangle
is
1

= 216 3 36 3 sq units.
6

(See the above figure II)


each side of regular hexagon = 12 units
Therefore in OEF
EF OG = 36 3
or,

1
12 OG 36 3
2

or, OG = 6 3 units.
Slant height of the pyramid
=
=

OO2 OG 2
6 2 6

144

K KUNDAN
= 12 units
Now, slant surface area

I
Slant surface area

1
(perimeter of base) slant height
2
OG is the line joining the mid-point of
any side of regular hexagon to point O of
the pyramid ie slant height of pyramid.
(See the above figure.)
=

II

1
perimeter of base slant height
2

1
12 6 12 = 432 units2
2

Right Circular Cone


A right circular cone is a solid generated by the
revolution of a right-angled triangle about one of
the sides containing the right angle as axis.

Take a t riangle ABC right -angled at B and


suppose it to revolve about one side AB as fixed
axis. The hypotenuse AC will generate the curved
surface of the cone.
The circle described by the radius BC (usually
denoted by r) is the base of the cone.
The point A is called the vertex. The height of

589

Mensuration II (Surface Area and Volume)


the cone (usually denoted by h) is the length of the
Axis AB, and the slant height (usually denoted by
l) is the length of the hypotenuse AC.

Volume and Curved Surface Area of a


Cone

Frustum of a Cone
If a cone is cut by plane parallel to the base of the
cone, then the portion between the plane and base
is called the frustum of the cone.

(i) Curved Surface Area of a Cone


Imagine a hollow cone to be made of thin card
board. Cut it along its slant height. Then it can be
spread out into the sector ADC of which the radius
AC and the arc CD are respectively slant height
and circumference of the base of the cone.

(a) Volume of a Frustum of a Cone: Let R and


r be the radii of base and top of the frustum of a
cone respect iv ely. Let h be t he height of the
frustum, then volume of frustum of right circular

h 2
[R r 2 Rr ] cu units
3
(b) Lateral Surface Area of frustum of right

cone =

Thus, curved surface area or lateral surface area

circular cone = (R r )l sq units


where l2 = h2 + (R r)2.
(c) Total Surface Area of frustum of right circular
cone = Area of base + Area of top + Lateral surface
area

K KUNDAN
=

1
radius (CA) arc (CD)
2

1
l 2r
2

= R2 r 2 l (R r )

1
(circumference of base) (slant height)
2
= rl
Area of the base = r2
Now, total surface area
= curved surface + area of base
= rl + r2
= r (l + r)
where r = radius of base and l = slant height
Note: When perpendicular height is given, then
Slant height
=

=
or l =

(Perpendic ular height)2 (Radius)2


h2 r 2

(Theorem of Pythagoras)

(ii) Volume of a Cone


=

1
area of the base perpendicular height
3

1 2
r h
3
where, r = radius of the base, and
h = perpendicular height
=

= R 2 r 2 l(R r ) sq units.

Solved Examples
Ex. 77: A right triangle with its sides 5 cm, 12
cm and 13 cm is revolved about the side
12 cm. Find the volume of the solid so
form ed.
Soln:
Let ABC be a right triangle with AB = 12
cm, BC = 5 cm and AC = 13 cm.
When this triangle is revolved about AB,
it forms a right circular cone of radius
= BC = 5 cm and height AB = 12 cm.

590

Concept of Arithmetic
Volume of the solid formed
= Volume of the cone of radius 5 cm
and height 12 cm

1 22

5 5 12 cm3
=
3 7
1 2

Putting r 5, h 12 in V 3 r h

= 314.28 cm 3.
Ex. 78: The base radii of two right circular
cones of the same height are in the
rati o 3 : 5. Fi nd t he r ati o of t heir
volumes.
Soln:
Let r1 and r2 be the radii of two cones and
V1 and V 2 be their volumes. Let h be the
height of the two cones.

1 2
1 2
r1 h and V =
r2 h .
Then, V1 =
2
3
3
1 2
r1 h
V1
r2
9

3
12
1 2
V2
25
r2 h r2
3
r1
3
r2
9

(Given) 12

25
r2
r2 5
Hence, the ratio of the volumes of two
cones is 9 : 25.
Ex. 79: A right circular cone is 3.6 cm high and
radius of its base is 1.6 cm. It is melted
and recast into a right circular cone
with radius of its base as 1.2 cm. Find
its height.
Soln:
We have: First cone Second cone
Radii
r1 = 1.6 cm, r2 = 1.2 cm
Heights h1 = 3.6 cm, h2 = ?
Volumes V1 and V 2
Clearly, two cones have the same volume,
ie V1 = V2

Soln:

10 cm. Find the height to which the


water rises.
Let r1 = radius of the conical vessel
= 5 cm
h1 = height of the conical vessel
= 24 cm
and, r2 = radius of the cylindrical vessel
= 10 cm
Suppose water rises upto the height of h2
cm in the cylindrical vessel.
Clearly,
Volume of water in conical vessel = Volume
of water in cylindrical vessel
or,

1 2
r1 h1 r22h 2
3

or, r12h1 3r22h 2


or, 5 5 24 = 3 10 10 h2

5 5 24
= 2 cm
3 10 10
Hence, t he height of wat er in t he
cylindrical vessel is 2 cm.
Ex. 81: A cone and a cylinder are having the
same base. Fi nd t he r ati o of t hei r
heights if their volumes are equal.
Soln:
Let the radius of the common base be r.
Let h1 and h2 be the height of the cone
and cylinder respectively.
Now,
h2 =

K KUNDAN
1 2
1
r1 h1 r22h 2
or,
3
3
or,

r12h1

or, h 2

or, h2 =

r22h 2

r12h1
r22

1.6 1.6 3.6


cm
1.2 1.2

16 16 36
cm = 6.4 cm
12 12 10
Ex. 80: A conical vessel whose internal radius
is 5 cm and height 24 cm is full of
water . The water is em pti ed i nto a
cylindrical vessel with internal radius
=

Volume of the cone =

1 2
r h1
3

Volume of the cylinder = r 2h 2

It is given that the cone and the cylinder


are of the same volume.

1 2
r h1 r 2h 2
3

or,

1
h1 h 2
3

or,

h1
3

h2
1

h1 : h2 = 3 : 1

Hence, the ratio of the height of the cone


and cylinder is 3 : 1.
Ex. 82: From a right circular cylinder with
height 10 cm and radius of base 6 cm,
a right circular cone of the same height
and base is removed. Find the volume
of the remaining solid.
Soln:
Let V1 and V2 be the volumes of the right
circular cylinder and cone respectively.
Th en .
22

6 6 10 cm3
V1 =
7

[Using: V1 = r2h]

591

Mensuration II (Surface Area and Volume)


1 22

6 6 10 cm3
and, V2 =
3 7

1 2

Using : V2 3 r h

Volume of the remaining solid = V1 V2


1 22
22

6 6 10
6 6 10 cm3
=
3 7
7

22
1

6 6 10 1 cm3
7
3

1232 3
cm = 6cm
616
2
2
2
Now, l = r + h
or, h =

22
2
6 6 10 cm3
7
3
= 754.28 cm 3.
Ex. 83: The radius of a cone is 3 cm and vertical
height is 4 cm. Find the area of the
curved surface.
Soln:
We have, r = 3 cm and h = 4 cm

196 36 =

232 cm

= 2 58 cm
Curved surface of the cone = rl

22
14 2 58 cm2
7

=
=

r 2 h 2 (14 )2 62

or, l =

= 88 58 cm2
Ex. 85: The radius and height of a cone are in
the ratio 4 : 3. The area of the base is
154 cm 2. Find the area of the curved
surface.
Soln:
Let r be the radius and h be the height of
the cone. It is given that

r
4

h
3
Let r = 4x and h = 3x
Now, area of the base = 154 cm2
or, r2 = 154

K KUNDAN
Let l cm be the slant height of the cone.
Th en ,
l2 = r2 + h2
or, l2 = 32 + 42
or, l2 = 25
or, l =

22

4 5 cm 2 = 62.85 cm2.
=
7

Ex. 84: The volume of a right circular cone is


1232 cm 3 . If the radius of its base is
14 cm, find its curved surface.
Soln:
Let r be the radius, h be the height and l
be the slant height of the cone.
Then, r = 14 cm
[Given]
Now, Volume = 1232 cm 3

1 2
r h 1232
3

1 22

14 14 h 1232
or,
3 7
or,

22
(4x )2 154
7

or,

22
16x 2 154
7

2
or, x

or, x =

25 cm = 5 cm

Area of the curved surface = rl

or,

or,

616
h 1232
3

154 7 49

22 16 16
49
7

cm
16
4

r = 4x = 4

7
cm = 7 cm,
4

7 21

cm
4
4
Let the slant height be l cm. Then,
l2 = r2 + h2
and h = 3

or, l =

r2 h2

or, l =

21
72

cm

441
1225
35

cm.
16
16
4
Curved surface area = rl

l =

49

35
22
7
cm2 = 192.5 cm2
=
7
4

592

Concept of Arithmetic

Ex. 86: What length of cloth 3 m wide will be


required to make a conical tent whose
height is 16 m and the radius of whose
base is 12 m?
Soln:
Let the slant height of the cone be l metres.
Th en ,
l =

r2 h2

or, l =

122 162 m
[ r = 12 m and h = 16 m]

or, l =

400 m = 20 m
Curved surface area of the tent = rl
22

12 20 m2
=
7

Let the length of the cloth required to


make the tent be x metres.
Th en ,
Area of the cloth = Curved surface area of
the tent
or, 3 x =

22
12 20
7
[ Width of the cloth = 3 m]

22 12 20
m = 251.42 m
73
Hence, the length of the required cloth
= 251.42 m.
Ex. 87: A coni cal tent is 9 m hi gh and t he
radius of its base is 12 m.
(i) What is the cost of the canvas
required to make it, if a square
metre canvas costs Rs 10?
(ii) How m any per sons can be
accommodated in the tent, if each
person requires 2 square metres
on the ground and 15 m3 of space
to breathe in?
Soln:
We have,
r = radius of the base of conical tent
= 12 m
h = height of the conical tent = 9 m.
l = slant height of the conical tent
or, x =

Since each person requires 2 sq metres of


floor area.
Maximum number of persons who will
have enough space on the ground
452.16
=
= 226 (approximately)
...(i)
2
Again,
Volume of the conical tent
1
=
Area of the base height
3

1
452.16 9 m3 = 1356.48 m3
3
Air space required per person = 15 m3
Number of per sons who will hav e
enough air space to breathe in
1356.48
=
= 90
15
Between 226 and 90, the smaller number
is 90. Hence, 90 persons can be
accommodated.
Ex. 88: A conical tent is to accommodate 11
persons. Each person must have 4 sq
metres of the space on the ground and
20 cubic metres of air to breathe. Find
the height of the cone.
Soln:
Let h metres be the height, r metres be
the radius of base of the cone. Since the
tent can accommodate 11 persons and each
person requires 4 sq metres of the space
on the ground and 20 cubic metres of air.
Therefore,
Area of the base = (11 4) m2 = 44 m2
or, r2 = 44 m2
.... (i)
and
Volume of the cone = (11 20) m3
= 220 m3
=

K KUNDAN
(i)

r2 h2 =

122 92 m

225 m = 15 m

Area of lateral surface = rl

22
12 15 m2 = 565.2 m2
7
Total cost of canvas
= Rs (565.2 10) = Rs 5652
(ii) Area of the base of the conical tent
=

= r2 =

22
12 12 m2 = 452.16 m2
7

1 2
r h 220 m3
.... (ii)
3
Dividing (ii) by (i), we get
or,

1 2
r h
220
3

44
r 2

h
5
3
or, h = 15 m.
Hence, t he height of t he cone is 15
metres.
Ex. 89: Water flows at the rate of 10 metres
per minute through a cylindrical pipe 5
mm in diameter. How long would it take
to fill a conical vessel whose diameter
at the base is 40 cm and depth 24 cm?
Soln:
We have
r = radius of the base of the conical vessel
= 20 cm
or,

593

Mensuration II (Surface Area and Volume)


h = height of the conical vessel = 24 cm
Volume of the conical vessel
1 2
r h
=
3

1 22

20 20 24 cm3
=
..... (i)
3 7

Suppose the conical vessel is filled in x


minutes. Then, length of the water column
= (10 x) metres = 1000x cm.
Clearly, water column forms a cylinder of
5
mm
lengt h 1000x cm, and r adius
2

5
1
cm cm .
20
4
Volume of the water that flows in x
minutes

22 1 2

= 7 4 1000 x cm3

Ex. 91: A cone of height 24 cm has a curved


surface area 550 cm2. Find its volume.
(Take = 22/7)
Soln:
Let r cm be the radius of the base and l cm
the slant height. Then,
l2 = r2 = 242
[Using l2 = r2 = h2]
or, l2 = r2 = 576
l r 2 576
... (i)
Now, curved surface area = 550 cm2
or, rl = 550
[Using (i)]
or,

2
or, r r 576 550

7
22

or, r r 2 576 25 7
or,

..... (ii)

22
r r 2 576 550
7

r 2 (r 2 576) (25 7 )2

or, r 4 576r 2 (252 72 ) 0

From (i) and (ii), we have

or, r 2 576r 2 625 49 0

22 1 2

1000 x

7 4

or, r 4 625r 2 49r 2 625 49 0


or, r 2 (r 2 625) 49(r 2 625) 0

1 22

20 20 24 cm3

3 7

K KUNDAN
or, r 2 49 0
or, r = 7

20 20 24 16
3 1000

or, x =

or, (r 2 625)(r 2 49) 0

256
1
51
minutes
5
5
= 51 minutes 12 seconds.
Hence, the conical vessel is filled in 51
minutes 12 seconds.
Ex. 90: If h, c, V are respectively the height,
the curved surface and the volume of a
cone, prove that 3Vh3 c2h2 + 9v 2 = 0.
Soln:
Let r and l denote respectively the radius
of the base and slant height of the cone.
Th en ,
or, x =

1 2
l r 2 h 2 , V 3 r h and c rl

[ r 2 625 0]

1 2
1 22
r h =

7 7 24
3
3 7
= 1232 cm3
Ex. 92: A sem i-ci rcul ar sheet of met al of
diameter 28 cm is bent into an open
coni cal cup. Find t he dept h and
capacity of cup.
Soln:
When the semi-circular sheet is bent into
an open conical cup, the radius of the
sheet becomes the slant height of the cup
and t he circumfer ence of t he sheet
becomes the circumference of the base of
the cone.
Volume =

3Vh 3 c 2h 2 9V 2

1 2
1

r h h 3 rl 2 h 2 9 r 2h
3
3

2r 2h 4 2r 2l 2h 2 2r 4h 2

2r 2h 4 2r 2h 2 r 2 h 2 2r 4h 2

r 2 h2

2r 2h 4 2r 4h 2 2r 2h 4 2r 4h 2 0

l = slant height of the conical cup


= 14 cm
Let r cm be the radius and h cm the height
(depth) of the conical cup. Then,
Circumference of the base of the conical
cup = Circumference of the sheet

594

Concept of Arithmetic

V
or, 2r 14
or, r = 7 cm
Now,

1 2
r h
3

1 22

4 4 8 2
or, V
3 7

l2 r 2 h2
or, h l 2 r 2 = 142 72 7 3 cm
= (7 1.732) cm = 12.12 cm
Depth of the cup = 12.12 cm
Also, capacity of the cup = Volume of the
cup

1 2
= r h cm3
3

1 22

7 7 12.12 cm3
=
3 7

= 622.26 cm 3
Ex. 93: A sector of a circle of radius 12 cm
has the angle 120. It is rolled up so
that t wo boundi ng r adii ar e joined
together to form a cone. Find the volume
of the cone.
Soln:
When a sector of a circle is rolled up in a
given manner, we obtain a cone whose
slant height is equal to the radius of the
sector and the circumference of the base
of the cone is equal to the length of the
arc of the sector.
Now, length of the arc of the sector
120

2 12 cm = 8 cm
=
360

22 128 2
189 .5 cm 3
21
Ex. 94: A right triangle, whose sides are 15
cm and 20 cm, is made to revolve about
its hypotenuse. Find the volume and the
sur face ar ea of t he double cone so
form ed.
(Use = 3.14)
Soln:
Let ABC be the right triangle right angled
at A whose sides AB and AC measure 15
cm and 20 cm, respectively.

or, V

K KUNDAN
Sector angle

2r
Using : arc
360

Let r cm be the radius of the base, h cm be


the height and l cm be the slant height of
the cone. Then,
l = radius of the sector = 12 cm.
And, Circumference of the base of the cone
= Length of the arc of the sector
or, 2r 8 cm
or, r = 4 cm
Now,

The length of the side BC (hypotenuse)


=

15 2 20 2 cm = 25 cm

Here, AO (or AO ) is the radius of the


common base of the double cone formed
by revolving the right triangle about BC.
Height of the cone BA A is BO and slant
height is 15 cm.
Height of the cone CAA is CO and slant
height is 20 cm.
Now, AOB ~ CAB (AA similarity)
AO 15

Therefore,
20
25
This gives AO =

l2 r 2 h2

20 15
cm = 12 cm
25

BO 15

15
25

or, h l 2 r 2 122 42

Also,

128 8 2 cm
Let V cm3 be the volume of the cone. Then,

This gives BO =

15 15
cm = 9 cm
25

595

Mensuration II (Surface Area and Volume)


Thus, CO = 25 cm 9 cm = 16 cm
Now, volume of the double cone
1
1

3.14 122 9 3.14 122 16 cm 3


3
3

3.14
122 (9 16) cm3 = 3768 cm3
3
Surface area of the double cone
= (3.14 12 15 + 3.14 12 20) cm2
= 3.14 12 (15 + 20) cm2
=1318.8 cm 2
Ex. 95: (a) The radii of the ends of a bucket
of height 24 cm are 15 cm and 5
cm. Find its capacity.
(Use = 22/7)
(b) Also find the surface area of the
bucket.
Soln: (a) Capacity of the bucket = Volume of
frustum of a cone
h 2
R r 2 Rr
=
3

Ex. 96: The height of a cone is 30 cm. A small


cone is cut off at the top by a plane
parallel to the base. If its volume be

Soln:

1
of the volume of the given cone, at
27
what hei ght abov e t he base i s t he
disection made?
Volume of the original cone OAB
=

1
1
R 2h R 2 30
3
3

= 10R 2 cu cm.

22 24

(15)2 52 15 5 cu cm
7
3

22
8 (225 25 75) cu cm
7

K KUNDAN
Volume of small cone OCD =

1 2
r h
3

Also volume of small cone OCD =

1
27

Volume of the cone OAB

176
325 cu cm
7
= 8171.43 cu cm
(b) Again surface area of a bucket
= Later al surface area of t he f rust um
of a cone + Ar ea of t he t op of the
frustum of cone
=

= l (R r ) r 2 l (R r ) r 2
wher e
l =

1 2
1
r h
(10R 2 )
3
27

or, h

10 R 2
3
10 R
2

27
9 r
r

....(i)

From similar OQB and OPD, we get

QB OQ 30

PD
OP
h

R 30

r
h
From (i) and (ii) we get

h 2 (R r )2 (24)2 (15 5 )2

.......(ii)

10 30
10 900
2


9 h
9
h
h3 = 10 100 = 1000
h

576 100 676 = 26 cm.


Surface area of the bucket

= l(R r ) r 2

22
26(15 5 ) 52
7

22
22
(26 20 25 ) =
545
7
7
= 1712.86 sq m
=

h = 3 1000 = 10 cm.
Ex. 97: If the radii of the ends of a bucket 45
cm high are 28 cm and 7 cm, determine
its capacity and the surface area.
Soln:
Volume of bucket = Volume of the cone
OAB Volume of the cone OCD

596

Concept of Arithmetic
Sphere

OPB and OQD are similar


PO
PB

OQ QD

h1
28
4
or h 7 1
2

A tennis ball and a fully blown football are some


familiar objects which bring to our mind the concept
of a sphere. A sphere is a three dimensional
geometrical object which can be defined as follows:
The set of all points in space which are equidistant
from a fixed point, is called a sphere.
The fixed point is called the centre of the sphere
and the constant distance is called its radius.
Figure gives the outline of a sphere whose
centr e is at O and radius = OP.A spher e has
symmetry about its centre. For every point P on
the sphere, the line joining P to the centre O of
the sphere, intersects the sphere again in a point

P such

that OP = OP .

or, h1 = 4h2
.......(i)
Also OP = PQ + OQ
h1 = 45 + h2
....... (ii)
( The height of the bucket is PQ)
Solving (i) and (ii), we get
h1 = 60 and h2 = 15.
Volume of the cone OAB
=

1
(28)2 60
3

A line segment through the centre of a sphere,


and with the end-points on the sphere is called a
diameter of the sphere.
In figure given above PP is a diameter of the
sphere.
All diameters of a sphere are of constant length,
being equal to twice the radius of the sphere.
Thus, if d is the length of a diameter of a sphere
of radius r, then d = 2r.
Note:The length of a diameter is also called the
diameter of the sphere. A sphere can also
be considered as a solid obtained on rotating
a circle about its diameter.

K KUNDAN
1 22

28 28 60 cu cm
3 7
= 49280 cu cm
1
(7)2 15
Volume of the cone OCD =
3
=

1 22

7 7 15 cu cm = 770 cu cm
3 7
Hence the volume of the bucket
= 49280 cu cm 770 cu cm
= 48510 cu cm
Again slant height (l1) of the cone OAB
=

(28 )2 (60 )2 784 3600

= 4384 = 66.2 (approx.)


Slant height (l2) of the cone OCD
=

(7)2 (15 )2 49 225

= 274 = 16.6 (approx).


Hence the surface area of the bucket is
= r1l1 r2l2 r22

22
22
22
28 66 .2
7 16 .6
77
7
7
7
= 88 66.2 - 22 16.6 + 154
= 5825.6 - 365.2 + 154
= 5614.4 sq cm (approx.)
=

Section of a Sphere by a Plane


A section of a sphere by a plane is a circle. The
plane through the centre gives the largest circular
section of the sphere. The radius of this section is
the same as the radius of the sphere. Any other
plane, that is a plane not through the centre, gives
a smaller section as shown in the figure given
below

597

Mensuration II (Surface Area and Volume)


A plane through the centre of a sphere divides
the sphere into two equal parts, each of which is
called a hemisphere.

Solved Examples
Ex. 98: Find the volume of a sphere of radius
7 cm.
Soln:
We know that the volume V of a sphere of
radius r is given by

4 3
r cubic units
3
Here, r = 7 cm.
V =
A plane through the centre of a solid sphere
divides it into two equal parts, each of which is
called a solid hemisphere.

Spherical Shell
It can be considered as the difference of two
solid concentric spheres. We call it a spherical
shell as defined below.
The difference of two solid concentric spheres is
called a spherical shell.
A spherical shell has a finite thickness, which
is t he difference of t he radii of t he two solid
spheres which determine it.

Volumes of a Sphere, a Hemisphere and a


Spherical Shell
We state the following formulae without proof.
Because proof is beyond the scope of this book.
(a) The volume V of a sphere of radius r is
given by
4 3
r cubic units.
V =
3
(b) The volume V of a hemisphere of radius r
is given by
2 3
r cubic units.
V =
3
(c) The volume V of a spherical shell whose
out er and inner r adii ar e R and r
4
(R 3 r 3 )
respectively is given by V =
3
cubic units.

4 22

7 7 7 cm3 = 1437.33 cm3.


3 7
Ex. 99: Find the vol ume of hemi spher e of
radius 3.5 cm.
Soln:
We know that the volume of hemisphere
of radius r is given by
V=

2 3
r cubic units
3
Here, r = 3.5 cm
V =

V =

2 22

3.5 3.5 3.5 cm3


3 7

2 22 7 7 7
cm3.
or, V =
3 7 2 2 2

K KUNDAN

Surface Area of a Sphere, Hemisphere and


Spherical Shell
We state the following formulae without proof as
proof is beyond the scope of this book.
(a) Surface area of a sphere of radius r is given
by
S = 4r2 square units.
(b) Curved surface area of a hemisphere of
radius r is given by
S = 2r2 square units.
(c) Total surface area of a hemisphere of radius
r = 2r2 + r2 = 3r2 square units.
(d) If R and r ar e out er and inner r adii of
spherical shell, then
outer surface area = 4R2 square units.

11 49
or, V =
cm3
32
or, V = 89.83 cm3.
Ex.100: A hemispherical bowl is made of steel
sheet 0.5 cm thick. The inside radius
of the bowl is 4 cm. Find the volume of
steel used in making the bowl.
Soln:
We have,
r = Inner radius of the bowl = 4 cm.
R = Outer radius of the bowl
= (4 + 0.5) cm = 4.5 cm
Volume of the inner hemisphere

2 3
2 22

4 4 4 cm3
r =
3
3 7

Volume of the outer hemisphere


=

2
2 22

4.5 4.5 4.5 cm3


R 3 =
3
3 7

Volume of steel used


=

2 22
2 22

4.5 4.5 4.5


4 4 4 cm3
3 7
3 7

2 22

(4.5)3 (4)3 cm3


3 7

44
(91.125 64) cm3
21

44
27.125 cm3 56.83 cm3
21

598

Concept of Arithmetic

Ex.101: A solid sphere of radius 3 cm is melted


and then cast into small spherical balls
each of diam eter 0.6 cm . Fi nd t he
number of balls thus obtained.
Soln:
Let the total number of balls be x.
Volume of the solid sphere
=

4 3 4
r 33 cm3 36 cm3
3
3
0.6
cm
2
= 0.3 cm.
a spherical ball

Radius of spherical ball =


Volume of

4
(0.3)3 cm3
3

4
3
3
3
36

cm 3 =
cm 3
3
10 10 10
1000
Volume of x spherical balls
=

36
x cm3
1000
Clearly, volume of the solid sphere
= Volume of x spherical balls.
=

36
x
1000
or, x = 1000
Hence, 1000 spherical balls are obtained
by melting the given solid sphere.
Ex.102: Thr ee sol id spher es of i r on whose
diameters are 2 cm, 12 cm and 16 cm
respectively, are melted into a single
solid sphere. Find the radius of the
solid sphere.
Soln:
Let the radius of the solid sphere be r cm.
Then, volume of the solid sphere = sum
of the volumes of three solid spheres of
radii 1 cm, 6 cm and 8 cm respectively.
or
or, 36

Soln:

Radius of the sphere = 3 cm.


Volume of the sphere

4
(3)3 cm3 36 cm3
3
Radius of the cylindrical vessel = 6 cm.
Suppose water level rises by h cm in the
cylindrical vessel. Then,
volume of the cylinder of height h cm and
radius 6 cm
=

= ( 62 h ) cm3 36 h cm3
Clearly, volume of water displaced by the
spher e is equal t o the v olume of the
sphere.
36h = 36
or, h = 1 cm
Hence, water level rises by 1 cm.
Ex.104: A hem ispher i cal bowl of i nt er nal
diameter 36 cm contains a liquid. This
li quid i s t o be fill ed i n cyl indr ical
bottles of radius 3 cm and height 6 cm.
How many bottles are required to empty
the bowl?
Soln:
Radius of hemispherical bowl = 18 cm.
Volume of hemispherical bowl

2
3
= (18 ) cm3
3

Radius of a cylindrical bottle = 3 cm


Height of a cylindrical bottle = 6 cm
Volume of a cylindrical bottle

K KUNDAN

4 3 4
4
4
r (1)3 (6)3 (8)3
or,
3
3
3
3
or, r3 = 13 + 63 + 83
4

Dividing both sides by 3

or, r3 = 1 + 216 + 512 = 729


or, r3 = 93
r = 9
Hence, the radius of the solid sphere is 9
cm.
Ex.103: A sphere of diameter 6 cm is dropped
in a right circular cylindrical vessel
partly filled with water. The diameter
of the cylindrical vessel is 12 cm. If the
spher e i s com pl et el y subm er ged i n
water, by how much will the level of
water rise in the cylindrical vessel?

= ( 32 6) cm3

Suppose x bottles are required to empty


the bowl.
Volume of x cylindrical bottles
= ( 9 6 x ) cm3.
Clearly, volume of liquid in x bottles
= Volume of bowl
or, 9 6 x

2
(18 )3
3

2 18 3
or, x = 3 9 6 = 72

Hence, 72 bottles are required to empty


the bowl.
Ex.105: The largest sphere is carved out of a
cube of a side 7 cm. Find the volume of
the sphere.
Soln:
The diameter of the largest sphere which
can be carved out of a cube of side 7 cm is
7 cm.
7
Radius of the sphere = r =
cm.
2
Hence, volume of the sphere =

4 3
r
3

599

Mensuration II (Surface Area and Volume)

Ex.108: Find the volume of a sphere whose


surface area is 154 square cm.
Soln:
Let the radius of the sphere be r cm. Then,
surface area = 154 cm2
or, 4r2 = 154
22 2
r 154
or, 4
7

4 22 7

cm3
=
3 7 2

4 22 343

cm3
3 7
8
= 179.66 cm 3
Ex.106: The volumes of two spheres are in
ratio 64 : 27. Find their radii if
sum of their radii is 21 cm.
Soln:
Let the radii of two spheres be r1 cm
r2 cm respectively.
Let the volumes of two spheres be V1
V2 respectively.
Th en ,
=

or,

4

3

or, r1

and

4r2
3

2
or, r

154 7 49

4 22
4

49
7
cm
4
2
Let V be the volume of the sphere. Then,

or, r =
and

4 3
r1
64
3
or,

4 3 27
r2
3

V1
64

V2
27

r13
r23

the
the

r1
4
or, r 3
2

..... (i)

Now,

V =

4 3
r
3

4 22 7 7 7
cm3
or, V =
3 7 2 2 2
1

or, V = 11 7 7 cm3
3

or, V = 179.66 cm3


Ex.109: A sphere, a cylinder and a cone are of
the same radius and same height. Find
the ratio of their curved surfaces.
Soln:
Let r be the common radius of a sphere, a
cone and a cylinder. Then,
height of the cone = height of the cylinder
= height of the sphere = 2r
Let l be the slant height of the cone. Then,

K KUNDAN
r1 r2 2l

or,

4r2
r2 21
3

4r2

Putting r1 3

7r2
21
3
or, r2 = 9 cm
or,

r1

4r2
3

4 9
= 12 cm
3
Hence, the radii of two spheres are 12 cm
and 9 cm.
Ex.107: Find the curved surface area and total
surface area of a hemisphere of radius
21 cm.
Soln:
We know that the curved surface area S
and total surface area S1 of a hemisphere
of radius r are given by
S = 2r2 and S1 = 3r2 respectively.
Here, r = 21 cm.
r1 =

22

21 21 cm2,
S = 2
7

22

21 21 cm2
and S1 = 3
7

S = 2772 cm2 and S1 = 4158 cm2

l =

r2 h2

or, l =

r2 h2 = 5 r
Now,
S1 = Curved surface area of sphere = 4r2
S2 = Curved surface area of cylinder
= 2r 2r = 4r2and
S3 = Curved surface area of cone
= rl r 5 r =

5 r 2

S1 : S2 : S3 = 4r 2 : 4r 2 : 5 r 2
= 4: 4 : 5
Ex.110: The volume of the two spheres are in
the ratio 64 : 27. Find the difference of
their surface areas, if the sum of their
radii is 7.
Soln:
Let the radii of two spheres be r1 cm and
r2 cm respectively.
Let the volumes of two spheres be V1 and
V2 respectively.
Th en ,

V1
64

V2
27

or,

4 3
r1
64
3

4 3 27
r2
3

600

Concept of Arithmetic

or,

r13
r23

43
33

r1
4
or, r 3
2

3
r1
4
or,
3
r2

or, r1

Now, r1 r2 7

4
r2
3

.... (i)

[Given]

4
r2 r2 7
or,
3
or,

7
r2 7
3

or, r2 = 7 3 cm
7

r2 = 3 4 cm.
3

Let S1 and S2 be the surface areas of two


spheres. Then,
S1 = 4r12 4 4 4 64 cm2 and

Ex.112: The internal and external diameters of


a hollow hemispherical vessel are 24
cm and 25 cm respectively. The cost to
paint 1 cm 2 of the surface is Rs 0.05.
Find the total cost to paint the vessel
all over.
(Use = 22/7)
Soln:
Let the external and internal radii of the
hemispherical vessel be R cm and r cm
respectively. Then, R = 12.5 cm and r =
12 cm
Now, Area of outer surface = 2R2
Area of the inner surface = 2r2
Area of the ring at the top = R2 2r2
Total area to be painted

= 2R 2 2r 2 R 2 r 2

= 3R r
=

22
cm2 = 88 cm2
7
Ex.111: Show that the surface area of a sphere
i s t he sam e as t hat of t he l at er al
surface of a right circular cylinder that
just encloses the sphere.
Soln:
Let the radius of the sphere be r cm. Then,
surface area of the sphere
= 4r2cm 2
...... (i)
The radius and height of a right circular
cylinder that just encloses the sphere of
radius r are r and 2r respectively.
Surface area of the cylinder
= 2r 2r
[ h = 2r]
= 4r2 cm2
...... (ii)

22
3 (12.5 )2 (12)2
cm2
7

22 25
2
3
12 cm2
7 2

22
468.75 144 cm2
7

S2 = 4r22 4 3 3 36 cm 2
S1 S2 64 36 28 cm2

K KUNDAN
= 28

From (i) and (ii), we obtain


Surface area of the sphere is equal to the
surface ar ea of the cylinder t hat just
encloses the sphere.

22
612.75 cm2
7

13480.5
cm 2 = 1925.78 cm2
7
Cost of painting
= Rs (1925.78 0.05)
= Rs 96.28
=

Surface Area and Volume of a Combination


of Solids
Ex.113: A godown building is in the form as
shown in the figure given below. The
vertical cross-section parallel to the
width side of the building is a rectangle
7 m 3 m, mounted by a semi-circle of
radius 3.5 m. The inner measurements
of the cuboidal portion of the building
are 10 m 7 m 3 m. Find the volume
of the godown and the total interior
surface area excluding the floor (base).
(Take = 22/7)

601

Mensuration II (Surface Area and Volume)


Soln:

Since the top of the building is in the


form of half of the cylinder of radius 3.5
m, and lengt h 10 m, split along t he
diameter.
V = Volume of the godown

1
(Volume of the
2
cylinder of radius 3.5 m and length 10 m)
= Volume of the cuboid +

1 22

3.5 3.5 10 m 3
= 10 7 3
2 7

= (210 192.5) m3 = 402.5 m3.


Total interior surface areas excluding the

1
(Curved
2
surface area of the cylinder) + 2 (area of
the semi-circles)
1
22

3.5 10
= 2(10 7) 3 2
2
7

base floor = Area of four walls +

22

2
(3.5)2 m 2
7

= (102 + 110 + 38.5) m2 = 250.5 m2.


Ex.114: The length of an edge of a cube is 24
cm. It is cut by a plane into a pyramid
in such a way that its three coterminus
edges rem ain hal f of their or iginal
length. Find the volume of the pyramid.
Soln:
In the figure given below, EPQR is the
given pyr amid such that t he base is
equilateral triangle PQR.

12 2

or, 144 h 2

3
or, h2 = 144 96
2
or, h = 48

or, h = 4 3 cm
Volume of the pyramid
1
=
(Area of the base Height)
3

2
1
3

12 2 4 3 cm3 = 288 cm3


3
4
Ex.115: A cylinder is within the cube touching
all the vertical faces. A cone is inside
the cylinder. If their heights are same
with the same base, find the ratio of
their volumes.
Soln:
Let the length of each edge of the cube be
a units. Then,
V1 = Volume of the cube = a3 cubic units.

K KUNDAN
Since a cylinder is within the cube and it
touches all the vertical faces of the cube.
r = radius of the base of the cylinder
a
=
2
h = height of the cylinder = a
V2 = Volume of the cylinder = r2h
=

22 a 2

a cubic units
7
4

11 3
a cubic units
14
A cone is drawn inside the cylinder such
that it has the same base and same height.
1 2
r h
V3 = Volume of the cone =
3
=

Lateral edges EP, EQ and ER are of the


same length equal to half of the length of
an edge of the cube ie 12 cm.
Let h be the height of the pyramid EPQR.
In right triangle EPQ, we have
PQ2 = EP2 + EQ2
or, PQ2 = 122 + 122

1 22 a

a cubic units
3 7 2

11 3
a cubic units
42

or, PQ 2 122 12 2 cm.


a = length of each side of the base
= 12 2 cm.

a2
12 h
3
2

11 3 11 3
a :
a
14
42
= 42 : 33 : 11.

3
V1 : V2 : V3 = a :

602

Concept of Arithmetic

Ex.116: An iron pillar has some part in the form


of a ri ght ci rcul ar cyl i nder and
r em ai ni ng i n the for m of a r i ght
circular cone. The radius of the base
of each of cone and cylinder is 8 cm.
The cylindrical part is 240 cm high and
the conical part is 36 cm high. Find
the weight of the pillar if one cubic cm
of iron weighs 7.8 grams.
Soln:
Let r1 cm and r2 cm denote the radii of the
base of the cylinder and cone respectively.
Th en

Ex.117: A circus tent is cylindrical upto a height


of 3 m and conical above it . I f t he
diameter of the base is 105 m and the
slant height of the conical part is 53
m , fi nd t he t ot al canvas used i n
making the tent.
Soln:
Total canvas used
= Cur v ed sur f ace ar ea of cylinder +
Curved surface area of cone

22
22

52.5 3
52.5 53 m 2
= 2
7
7

r1 = r2 = 8 cm.
Let h1 and h2 cm be the heights of the
cylinder and the cone respectively. Then,
h1 = 240 cm and h2 = 36 cm.

22
52.5 (6 53) m2 = 9735 m2
7
Ex.118: A tent is of the shape of a right circular
cylinder upto a height of 3 metres and
then becomes a right circular cone with
a maxi mum hei ght of 13.5 m etr es
above the ground. Calculate the cost
of painting the inner side of the tent at
the rate of Rs 2 per square metre, if
the radius of the base is 14 metres.
Soln:
Let r metres be the radius of the base of
the cylinder and h metres be its height.
Th en
r = 14 m and h = 3 m.
=

K KUNDAN
Now, volume of the cylinder = r12h1 cm 3
= ( 8 8 240 ) cm3
= ( 64 240 ) cm3

Volume of the cone =

1 2
r2 h 2 cm3
3

= 8 8 36 cm 3
3

= 64 36 cm 3
3

Total volume of the iron


= Volume of the cylinder + Volume
of the cone
1

= 64 240 64 36 cm3
3

= 64 (240 12) cm3

22
64 252 cm3
7
= 22 64 36 cm3
Hence, total weight of the pillar
= Volume Weight per cm3
= (22 64 36) 7.8 gms
= 395366.4 gms
= 395.3664 kg
=

Curved surface area of the cylinder


= 2rh m2

22

14 3 m2 = 264 m2
= 2
7

603

Mensuration II (Surface Area and Volume)


Let r1 m be the radius of the base, h1 m be
the height and l m be the slant height of
the cone. Then,
r1 = 14 m, h1 = (13.5 - 3) m = 10.5 m
and l1 =
or, l1 =
=

r12 h12
142 (10.5)2 m

= 2r1h1 r2l 2 m2
= 2r1h1 r2l 2 m2
=

22
2 2.1 4 2.1 2.1 2 m2
7

22
2.1 8 2.1 2 m2
7

22
2.1 8 2.1 1.414 m2
7

22
2.1 8 2.9694 m2
7

196 110.25 m

= 306.25 m = 17.5 m
Curved surface area of the cone

22

14 17.5 m2
= r1l1 =
7

= 770 m2
So, total area which is to be painted
= Curved surface area of the cylinder +
Curved surface area of the cone
= (264 + 770) m2 = 1034 m2.
Hence, cost of painting
= Rs (1034 2) = Rs 2068
Ex.119: The interior of a building is in the form
of a right circular cylinder of diameter
4.2 m and height 4 m surmounted by a
cone. The vertical height of cone is 2.1
m. Fi nd the outer sur face area and
volume of the building.
(Use = 22/7)
Soln:
Let r1 be the radius of base of the cylinder
and h1 m be its height. Then, r1 = 2.1 m
and h1 = 4 m

22

2.1 10 .9694 m2
=
7

= 22 0.3 10.9694 m2
= 72.3980 m2 = 72.40 m2
Volume of the building
= Volume of the cylinder + Volume
of the cone
1
2

2
= r1 h1 r2 h 2 m3
3

[ r2 = r1]

1
2

2
= r1 h1 r1 h 2 m3
3

K KUNDAN
1

2
= r1 h1 h 2 m3
3

22
2.1 2.1 4 0.7 m3
7
= 22 0.3 2.1 4.7 m3
= 65.142 m 3
Ex.120: The interior of a building is in the form
of cyli nder of di am et er 4.3 m and
height 3.8 m, surmounted by a cone
whose vertical angle is a right angle.
Find the area of the surface and the
volume of the building.
(Take = 22/7)
Soln:
We have
=

Let r2 m be the radius of the base of the


cone, h1 m be its height and l 1 m be its
slant height. Then,
r2 = 2.1 m, h2 = 2.1 m
l 22 r22 h 22
or, l2 =
=

r22 h 22 =

2.12 2

2.12 2.12

= 2.1 2 m

Now, outer surface area of the building


= Curved surface area of cylinder +
Curved surface area of cone

22
1

2.1 2.1 4 2.1 m3


7
3

604

Concept of Arithmetic
r1 = Radius of the base of the cylinder

4. 3
m = 2.15 m
2
r2 = Radius of the base of the cone
= 2.15 m
h1 = Height of the cylinder = 3.8 m
In VOA, we have
=

sin 45 =
1

or,

or, VA

OA
VA
2.15
VA

Radius BO of the hemisphere (as well as

2 2.15 m

= (1.414 2.15) m = 3.04 m


Clearly, VOA is an isosceles triangle.
Therefore, VO = OA =2.15 m
h2 = Height of the cone = VO = 2.15 m
l2 = Slant height of the cone = VA
= 3.04 m
Now,
Surface area of the building
= Surface area of cylinder + Surface
area of cone
= 2r1h1 r2l 2 m2
= 2r1h1 r1l2 [ r1 = r2 = 2.15m]

of cone) =

1
4 cm = 2 cm.
2

Now, let right circular cylinder EFGH


circumscribe the given solid.
Radius of the base of the right circular
cylinder = HP = BO = 2 cm.
Height of the cylinder = AP = AO + OP
= 2 cm + 2 cm = 4 cm
Now, volume of the right circular cylinder
volume of the solid

1
2
2
3
3
= 2 4 2 2 cm3
3
3

= (16 8) cm3 = 8 cm3

K KUNDAN
= r12h1 l 2

= 3.14 2.15 (2 3.8 + 3.04) m2


= 3.14 2.15 10.64 m2 = 71.83 m2
Volume of the building
= Volume of t he cylinder + Volume
of the cone

1
2

2
= r1 h1 r2 h 2 m3
3

1
2

2
= r1 h1 r1 h 2 m3
3

Hence, the right circular cylinder covers

8 cm3 more space than the solid.

Ex.122: The r adi us of t he base of a r ight


circular cone is 14 cm and altitude 20
cm. What is the largest lateral surface
area possible for a cylinder inscribed
in this cone?
Soln:
Taking first of all general cases,

[ r2 = r1]

2
= r1 h1 h 2 m3
3

2.15

m3
= 3.14 2.15 2.15 3.8
3

= 3.14 2.15 2.15 (3.8 + 0.7166) m3


= 3.14 2.15 2.15 4.5166) m3
= 65.55 m 3
Ex.121: A sol i d t oy i s i n t he for m of a
hem i spher e surm ount ed by a r i ght
circular cone. Height of the cone is 2
cm and the diameter of the base is 4
cm. I f a ri ght ci rcul ar cyl i nder
circumscribes the solid, find how much
more space it will cover.
Soln:
Let BPC be the hemisphere and ABC be
the cone standing on t he base of t he
hemisphere.

Let a cylinder with base radius r and


altitude h be inscribed in the cone. From
the similarity of the triangle AOS and
BO1S, it follows that

R (H h )
....(i)
H
The lateral surface area of cylinder S
r =

= 2rh , or

605

Mensuration II (Surface Area and Volume)

S =

Volume of 10 cones with hemispherical


tops

2R
h (H h )
H

{substituting (1) for r}.


From geometric consideration, h varies in
the interval 0 < h < H.
2Rh 2
S = 2Rh
H
2

RH 2R
H

h
2
H
2
(isolating the perfect square)
S is maximum when second term is zero,
H
O
ie h
2
H
ie, h
.
2
Maximum value of lateral surface area
=

1
RH
2
Radius = 14 cm
Height = 20 cm
Maximum value of lateral surface area
=

1 22

14 20 = 440 cm2.
2 7
Ex.123: A cylindrical container of radius 6 cm
and height 15 cm is filled with icecream. The whole ice-cream has to be
di str ibuted t o 10 chil dren in equal
cones with hemispherical tops. If the
height of the conical portion is four
times the radius of its base, find the
radius of the ice-cream cone.
Soln:
Let the radius of the base of the conical
portion be r cm.
Then, height of the conical portion
= 4r cm.

= (10 2r 3 ) cm3 = 20r 3 cm3


Volume of the cylindrical container
= ( 62 15 ) cm3 = 540 cm3
Clearly, volume of 10 cones with hemispherical tops = Volume of the cylindrical
container
or, 20r 3 540
or, r3 = 27
or, r = 3 cm.
Hence, radius of the ice-cream cone is 3
cm.
Ex.124: A solid wooden toy is in the shape of a
r ight ci r cul ar cone m ount ed on a
hem i spher e. I f t he r adi us of the
hemisphere is 4.2 cm and the total
height of the toy is 10.2 cm, find the
volume of the wooden toy.
Soln:
We have VO = 10.2 cm, OA = OO
= 4.2 cm.

K KUNDAN
Let r be the radius of the hemisphere and
h be the height of the conical part of the
toy. Then,
r = OA = 4.2 cm.
h = VO = VO OO = (10.2 4.2) cm
= 6 cm.
Also, radius of the base of the cone
= OA = r = 4.2 cm.
Volume of the wooden toy = Volume of
t he conical par t + Volume of the
hemispherical part

2 3
1 2
r cm3
= r h
3
3

Volume of cone with hemispherical top


= Volume of the cone + volume of the
hemispherical top
2 3
1 2
= r 4r r cm3
3
3

6 3
= r cm3 = (2r 3 ) cm3
3

r 2
(h 2r ) cm3
3

1 22

4.2 4.2 (6 2 4.2) cm3


3 7

1 22

4.2 4.2 14.4 cm3


3 7
= 266.11 cm 3
=

606

Concept of Arithmetic

Ex.125: A vessel i s i n t he for m of a hem i spherical bowl mounted by a hollow


cylinder. The diameter of the sphere is
14 cm and t he t otal height of t he
vessel is 13 cm. Find its capacity.
(Take = 22/7)
Soln:
Let r be the radius of the hemispherical
bowl and h be the height of the cylinder.
Then, r = 7 cm and h = 6 cm.

Now, volume of the solid = Volume of the


cylinder + Volume of two hemispheres

2
2 3
= r h 2 r cm3
3

Total capacity of the bowl


= Volume of the cylinder + Volume of
the hemisphere

2 3
2
= r h r cm3
3

2
2
= r h r cm3
3

4r
2
cm3
= r h
3

22 7 2
4 7
= 7 2 12 3 2 cm3

22 7 7 50

=
cm3
7 2 2 3
= 641.66 cm 3
Ex.127: A solid is in the form of a right circular
cone mounted on a hemisphere. The
radius of the hemisphere is 3.5 cm and
the height of the cone is 4 cm. The solid
is placed in a cylindrical tub, full of
water, in such a way that the whole
soli d i s submer ged i n wat er. If t he
radius of the cylinder is 5 cm and its
height is 10.5 cm, find the volume of
water left in the cylindrical tub.
(Use = 22/7)
Soln:
We have VO = 4 cm, OA = OB = OO
= 3.5 cm.

K KUNDAN
=

22
2

72 6 7 cm3
7
3

= 22 7

32
cm3
3

4928
cm3
3
= 16.42.66 cm 3
Ex.126: A solid is in the form of a cylinder with
hemi-spherical ends. The total height
of the solid is 19 cm and the diameter
of the cylinder is 7 cm. Find the volume
of the solid.
(Use = 22/7)
Soln:
Let r cm be the radius and h cm the height
of the cylinder. Then,
=

7
cm and
2

r =

h = 19 2 cm
2

= 12 cm
Also, radius of hemisphere
=

7
cm = r cm.
2

Volume of the solid = Volume of its


conical part + Volume of its semispherical part

607

Mensuration II (Surface Area and Volume)


2 22
1 22

(3.5 )2 4
(3.5)3 cm3
=
3 7
3 7

1 22

(3.5)2 4 2 3.5 cm3


3 7

1 22 7 2

= 3 7 2 11 cm3

Clearly, when the solid is submerged in


the cylindrical tub the volume of water
that flows out of the cylinder is equal to
the volume of the solid.
Hence,
Volume of water left in the cylinder
= Volume of cylinder Volume of t he
solid
2
22

1 22 7
2
= 7 (5) 10 .5 3 7 2 11 cm3

Surface area of the other hemisphere


= 3.08 m2.
Total surface area
= (22 + 3.08 + 3.08) m2
= 28.16 m2.
Rate of painting = Rs 10 per square metre.
Cost of painting = Rs (10 28.16)
= Rs 281.60
Ex.129: A toy is in the shape of a right circular
cylinder with a hemisphere on one end
and a cone on the other. The height
and radius of the cylindrical part are
13 cm and 5 cm respectively. The radii
of the hemispherical and conical parts
are the same as that of the cylindrical
part. Calculate the surface area of the
toy if height of the conical part is 12
cm .
Soln:
Let r cm be the radius and h cm the height
of the cylindrical part. Then,
r = 5 cm and h = 13 cm.

21 1 22 7 7
22

25

11 cm3
=
2 3 7 2 2
7

1
7

= 11 25 3 11 11 cm3
3
2

= (825 141.16) cm3


= 683.83 cm 3
Ex.128: A storage tank consists of a circular
cylinder, with a hemisphere adjoined
on either end. If the external diameter
of the cylinder be 1.4 m and its length
be 5 m, what wi l l be the cost of
painting it on the outside at the rate of
Rs 10 per square metre?
Soln:
We have,

K KUNDAN
Clearly, radii of the spherical part and
base of the conical part are also r cm. Let
h1 cm be the height, l cm be the slant
height of the conical part. Then,

l 2 r 2 h12
or, l =

22
0.7 5 m2 = 22 m2
7
Again, diameter of the hemisphere
= 1.4 m
Radius of the hemisphere = 0.7 m
Surface area of a hemisphere
= 2r2
= 2

22

0.7 0.7 m2
= 2
7

= 3.08 m2

52 12 2

[ h1 = 12 cm, r = 5 cm]

Diameter of the cylinder = 1.4 m

1.4
Radius of the cylinder =
m = 0.7m
2
Length of the cylinder = 5 m
Surface area of the cylinder = 2rh

r 2 h12 =

169 = 13 cm
Now, surface area of the toy
= Curved surface areas of the cylindrical
par t + Curv ed sur face ar ea of
hemispherical part + Curved surface
area of conical part
= (2rh 2r 2 rl ) cm2
= r (2h 2r l ) cm2

22

5 (2 13 2 5 13 ) cm2
=
7

22

5 49 cm2
=
7

= 770 cm2

608

Concept of Arithmetic

Ex.130: A wooden toy is in the form of a cone


surm ount ed on a hem ispher e. The
diameter of the base of the cone is 6
cm and its height is 4 cm. Find the
cost of painting the toy at the rate of
Rs 5 per 1000 cm2 .
Soln:
We have, radius of the base of the cone
= 3 cm

Total surface area = Curved surface area


of t he cylinder + Sur f ace ar eas of
hemispherical ends
= (2rh 2 2r 2 ) cm2
= (2rh 4r 2 ) cm2
= 2r (h 2r ) cm2
= 2

22
18 108 cm2
7
= 12219.42 cm 2
Rate of polishing = 7 paise per sq cm.
= 2

Height of the cone = 4 cm


Let l be the slant height of the cone. Then,
r 2 h 2 32 42 cm = 5 cm
Lateral surface area of the cone

22
18 (72 36) cm2
7
[ r = 18 cm, h = 72 cm]

l =

Cost of polishing = Rs 12219 .42


100

= Rs 855.36.
Ex.132: A conical vessel of radius 6 cm and
height 8 cm is completely filled with
water. A sphere is lowered into the
water and its size is such that when it
touches the sides, it is just immersed.
What fraction of water overflows. (See
the figure given below).

K KUNDAN
330
22

3 5 cm2
cm2
= rl =
7
7

Surface area of the hemisphere = 2r2


22
396

3 3 cm2
cm2 .
= 2
7
7

Total surface area of the toy

330 396

cm2 = 103.71 cm2


=
7
7
Rate of painting the toy
= Rs 5 per 1000 cm2

5
per cm2
1000
Cost of painting the toy
= Rs

= Rs 103 .71
1000

= Re 0.51 = 51 paise.
Ex.131: A solid is composed of a cylinder with
hemispherical ends. If the whole length
of the solid is 108 cm and the diameter
of the hemispherical ends is 36 cm, find
the cost of polishing the surface of the
solid at the rate of 7 paise per sq cm.
(Use = 22/7)
Soln:
W e hav e, r = r adius of t he cylinder
= radius of hemispherical ends = 18 cm,
h = Height of the cylinder = 72 cm

Soln:

Here, AD = 6 cm and DC = 8 cm (given)

609

Mensuration II (Surface Area and Volume)

BC =

82 62 = 10 cm
Now consider the ACD and EOC.
Bot h of the triangles are r ight -angled
triangle and ECO of the EOC is equal
to the ACD of the ACD.
Hence, the triangles ACD and EOC are
similar.
Let the value of EO be x (ie x is the radius
of the sphere)
See the given figure,
AD = 6 cm (given) AE = 6 cm
[From the theorem, The lengths of two
tangents drawn from an external point to a
circle are equal.]
EC = 10 cm 6 cm = 4 cm
Again, since ADC and EOC are similar.

DC EC

AD OE

or,

8 4

6 x

4 6
= 3 cm
8
radius of the sphere = 3 cm
Now, volume of the cone
or, x =

1
6 6 8 cuz cm and
=
3
the volume of the sphere

Now, OP2 = OD2 + PD2


or, 8 r 2 42 r 2
or, 64 16r r 2 16 r 2
or, 16r = 64 16 = 48
or, r =

48
= 3 cm
16

Now, volume of cone =

1 2
r h
3

1 22

6 6 8 cm3
3 7

Volume of sphere =

4 3
r
3

4 22
4 22

33
27 cm3
3 7
3 7
Fraction of water which overflows
=

Volume of sphere
Volume of cone

4 22

27
3
3 7

1 22

668 8
3 7

K KUNDAN
4
3 3 3 cu cm.
3
required fraction of water
=

4
333
3
3

= 1
6 6 8 8
3

required answer =

3
.
8

Alternative Method:
AC = AD = 6 cm
[ Length of two tangents from an external
point to a circle are equal.]

OC = 8 cm
or, OA = = 10 cm
or, OD = OA AD = 10 cm 6 cm = 4 cm
or, OP = OC PC = 8 r

Conversion of Solid from One Shape to


Another

The underlying concept for these type of questions


is that the total volume of a solid does not change
even when its shape changes. See the following
examples:
Ex.133: A sol id i ron rect angular bl ock of
dimensions 4.4 m, 2.6 m and 1 m is
cast into a hollow cylindrical pipe of
internal radius 30 cm and thickness 5
cm. Find the length of the pipe.
Soln:
Let t he length of pipe be l cm. Then,
volume of iron in the pipe is equal to the
volume of iron of the block.
We have,
Volume of the block
= (4.4 2.6 1) m3
= (440 260 100) cm3
r = Internal radius of the pipe = 30 cm
R = External radius of the pipe
= (30 + 5) cm = 35 cm.
Volume of iron in the pipe
= (External volume) (Internal volume)
= R 2h r 2h
= (R2 r 2 )h
= (R r ) (R r )h
= (35 30 ) (35 30 ) h cm3
= 65 5 h cm3

610

Concept of Arithmetic
Now, volume of iron in the pipe
= Volume of iron in the block
or, 65 5 h 440 260 100

22
65 5 h = 440 260 100
7

or,

7
1 1

cm
or, h = 440 260 100
22
65
5

or, h = 11200 cm = 112 m


Hence, the length of the pipe is 112 m.
Ex.134: A solid cube of side 7 cm is melted to
make a cone of height 5 cm, find the
radius of the base of the cone.
Soln:
We have, volume of the cube
= (side)3 = 73 cm3 = 343 cm3
Let the radius of the base of the cone be r
cm. Then,

1 22 2

r 5 cm3
3 7
Since the solid cube is melted to make a
cone. Therefore,
Volume of the cube = Volume of the cone
Volume of the cone =

or, 343 =

or, r2 =

3 8 8 2
6
or, r2 = 8
r = 8 cm.
Let the slant height of the cone be l cm.
Th en ,
or, r2 =

l =

r2 h 2

or, l =

82 62 cm = 64 36 = 10 cm.
Curved surface area of the cone = rl

22
8 10 cm2 = 251.42 cm2
7
Ex.136: How many spherical bullets can be made
out of a solid cube of lead whose edge
measures 44 cm, each bullet being 4
cm in diameter.
Soln:
Let the total number of bullets be x.
=

4
cm = 2 cm
2
Now, volume of a spherical bullet
Radius of a spherical bullet =

4
4 22

8 cm3
(2)3 cm3 =
3
3 7

Volume of x spherical bullets

1 22 2

r 5
3 7

343 3 7 7203

= 65.48
22 5
110

4 22

8 x cm3
=
3 7

Volume of the solid cube = (44)3 cm3.


Clear ly, volume of x spherical bullets
= Volume of cube.

K KUNDAN
or, r =

65.48 cm = 8.09 cm
Ex.135: A solid right circular cylinder of radius
8 cm and height 2 cm is melted and
cast into a right circular cone of height
3 times that of the cylinder. Find the
curved surface of the cone.
Soln:
We have,
volume of the solid right circular cylinder
= r2h
22

8 8 2 cm3
=
7

It is given that the solid right circular


cylinder is melted and cast into a right
circular cone of height 3 times that of the
cylinder ie 6 cm. Let r cm be the radius of
the cone.
Th en ,
Volume of cone =

1 2
r h
3

1 22

r 2 6 cm3
3 7

[ h = 6 cm]

But,
Volume of the cone = Volume of the cylinder

1 22 2
22

r 6 =
882
3 7
7

or,

4 22

8 x (44)3
3 7

or,

4 22

8 x 44 44 44
3 7

44 44 44 3 7
= 2541.
4 22 8
Hence, total number of spherical bullets
= 2541.
Ex.137: How many spherical lead shots each
4.2 cm in diameter can be obtained from
a r ectangul ar sol id of l ead wit h
dimensions 66 cm, 42 cm, 21 cm.
(Use = 22/7)
Soln:
Let the number of lead shots be x.
Volume of lead in the rectangular solid
= (66 42 21) cm3
or, x =

4.2
cm = 2.1 cm
2
Volume of a spherical lead shot
Radius of a lead shot =

4 22

(2.1)3 cm3
3 7

611

Mensuration II (Surface Area and Volume)


Volume of x spherical lead shots

4 22

(2.1)3 x cm3
=
3 7

Volume of x spher ical lead shot s


= Volume of lead in rectangular solid
4 22

(2.1)3 x 66 42 21

3 7

66 42 21 3 7
or, x =

4 22 (2.1)3

66 42 21 21 1000
4 22 21 21 21
= 1500
Hence, the number of spherical lead shots
is 1500.
Ex.138: A solid lead ball of radius 7 cm was
melted and then drawn into a wire of
diameter 0.2 cm. Find the length of the
wire.
Soln:
We have,
Radius of the lead ball = 7 cm
Volume of the lead ball
=

4 22

7 7 7 cm3
=
3 7

Ex.139: A hemisphere of lead of radius 8 cm is


cast into a right circular cone of base
radius 6 cm. Determine the height of
t he cone, cor r ect to t wo places of
decimal.
Soln:
Let h cm be the height of the cone.
We have, radius of hemisphere = 8 cm,
Radius of the base of the cone = 6 cm.
Now, volume of the cone = volume of the
hemisphere

1
2
62 h (8)3
3
3
or, 36h = 2 512.
or,

1024 256

= 28.44 cm.
36
9
Ex.140: A spher i cal cannon ball , 28 cm i n
di ameter is melted and cast i nto a
right circular conical mould, the base
of which is 35 cm in diameter. Find
the height of the cone, correct to two
places of decimal.
Soln:
Let h cm be the height of the cone. We
have, diameter of spherical cannon ball =
28 cm
or, Radius of base of the spherical cannon
ball = 14 cm.
Diameter of the base of the cone = 35 cm
or, h =

K KUNDAN
4312
cm3
=
3
Clearly, the wire is a cylinder of radius
0.1 cm.
Let the length of this wire be h cm. Then,
Volume of the wire
22

0.1 0.1 h cm3


=
7

11h
cm3
=
350
But,
Volume of the wire = Volume of the lead
ball
or,

11h
4312

350
3

4312 350

cm
or, h =
11
3
4312 350
m = 457.33 m
=
3 11 100
Hence, the length of wire = 457.33 m

35
cm
2
Now, volume of t he cone = volume of
spherical cannon ball
or, Radius of base of the cone =

or,

1
4
35
3

h (14)
3
3
2
2

35
3
h 4 (14 )
or,
2
2
2

cm
or, h = 4 14 14 14
35
35

2 2

or, h = 4 2 2 14 cm
5 5

or, h =

896
cm = 35.84 cm
25

612

Concept of Arithmetic

Practice Exercise
Exercise1
(Cuboid and Cube, Swimming Pool and Area of the Four Walls)
1.

2.

3.

4.

5.

6.

If the rainfall on a certain day was 5 cm, how


many litres of water fell on 1 hectare field on
that day?
The length, breadth and height of a cuboidal
reservoir is 7 m, 6 m and 15 m respectively.
8400 litres water is pumped out from the
reservoir. Find the fall in the water-level in
the reservoir.
What is the weight of a cubical block of ice
50 cm in length, if one cubic metre of ice
weighs 900 kilograms?
A cube of 9 cm edge is immersed completely
in a rectangular vessel containing water. If
the dimensions of the base are 15 cm and 12
cm, find the rise in water level in the vessel.
The surface area of a cuboid is 22 cm 2 and
the sum of the lengths of all its edges is 24
cm. Find the length of a diagonal of the cuboid.
A rectangular tank measuring 5.5 m by 4 m
by 2.5 m is dug in the middle of a field 35
m by 15.4 m. The earth dug out is spread
evenly on the remaining portion of the field.
How much is the level of field raised?
A rectangular field is 40 m long and 28 m
wide. A pit 12 m long, 6 m wide and 5 m
deep is dug in a corner of the field and the
earth taken out of the pit is spread uniformly
over the remaining port ion of t he f ield.
Calculate by how much is the level of the
field raised.
The length of the diagonal of a cube is 17.32
cm. Find the volume of the cube.
A tank is of the shape of a cuboid whose
length is 7.2 m and breadth is 2.5 m. Water
flows into it through a pipe whose crosssection is 5 cm 3 cm at the rate of 10 m per
second. Find the height to which water level
will rise in the tank in 40 minutes?
The weight of a cubic metre of a certain metal
is 480 kg. It is melted and then rolled into a
square bar 4 m long. Now an exact cube is cut
from it. Find the weight of the cube.
A metal bar of dimensions 19 cm 4 cm 2
cm and a metal cube of side 4 cm were melted
together and re-casted into a new single cube.
Find the length of the edge of new single
cube.
A metallic cube of edge 2.5 cm is melted and
recasted into the form of a cuboid of base 1.25
cm 0.25 cm. Find the increase in the surface
area.
The surface of water in a swimming pool is

14.

15.

16.

rectangle 32 m long and 9.5 m wide and the


depth of water increases uniformly from 1.5
m at one end to 4.5 m at the other end. What
is the volume of water in the pool?
Two-thirds of a cuboidal aquarium is filled
with water and then tilted on its side until
the water level coincides with one edge on
the bottom and one edge on the top. During
the tilting process, 6 litres of water is poured
out. What is the volume of tank?
The four walls of a room can be fully covered
by 70 square wall papers of 2 m 2m size.
The length of the room is 18 m and its breadth
is twice t hat of it s height . If t he cost of
carpeting is Rs 20 per square metre, what
will be the total expenditure in carpeting the
room?
The length of a hall is 20 m and width 16 m.
The sum of the areas of the floor and the flat
roof is equal to the sum of the areas of the
four walls. Find the height and the volume of
the hall.
The internal length, breadth and height of
an open box are 85 cm, 65 cm and 72.5 cm
respectively. Find the cost of painting the
outside of the box leaving its bottom, at Rs 15
per square decimetre, it being given that wood
is 25 mm thick.

K KUNDAN

7.

8.
9.

10.

11.

12.

13.

17.

1
metre thick is
4
built surrounding the outside of a court 17
metres square. Find the cost of that wall at

18. A wall 3 metres high and

Rs 2500 per cubic metre if there be a gate 1

1
2

metres wide on each side.

1
cm
2
thick. If its external dimensions be 1 m, 85
cm and 65 cm, how many square metres of
planking are used in the construction?
2 0 . A stream which flows at a uniform rate of 2.5
km an hour, is 20 metres wide, the depth of
a certain ferry being 1.2 metres. How many
litres pass the ferry in a minute? (1 cubic
metre = 1000 litres)
21. The inside of a wat er r eser v oir wit h
19. A box with a lid is made of planking 2

rectangular base is 25

3
metres long and
5

613

Mensuration II (Surface Area and Volume)


1
metres broad. Find the diagonal of its
5
base. If it is 14 metres deep, find how many
metric tonnes of water it can contain. (One
cubic metre of water weighs 1000 kg).
2 2 . A rectangular iron cistern open at the top
measures externally 300 cm long, 208 cm
broad and 180 cm deep. If the metal is 4.0 cm
thick, find the weight of the cistern when
(i) empty
(ii) full of water
[Giv en: Density of iron = 7. 2 gm/cc and
density of water = 1 gm/cc]
19

23. Water is flowing at the rate of 5 km/hr through


a pipe of radius 7 cm into a rectangle tank
which is 100 m long and 44 m wide.
Determine the time in which the level of water
in the tank will rise 14 cm.
(Take = 22/7)
24. When water is pumped through a pipe of
diameter 6 cm for 25 minutes into a cistern
of length 5 metres, breadth 3 metres, the
height of water raised through 2.4 metres.
Find the time required to fill up a cistern of 4
met res length, 3.6 metres br eadt h and 2
metres height by pumping water through a
pipe of radius 5 cm.

Exercise2
(Prism and Cylinder)
1.

2.

3.

4.

5.

6.

7.

8.

The base of a prism is a triangle of which the


sides ar e 17 cm, 25 cm and 28 cm
respectively. The volume of the prism is 4200
cubic cm. What is the height? Find its lateral
area also.
There are two prisms, one has equilateral
triangle as a base and the other a regular
hexagon. If both of the prisms have equal
height s and volumes, t hen find the rat io
between the length of each side at their bases.
A well with 7 metres inside diameter is dug
22.5 metres deep. Earth taken out of it is
spread all round to a width of 10.5 metres to
for m an embankment. Find the height of
embankment.
(Take = 22/7)
A cylindrical tank of diameter 35 cm is full of
water. If 11 litres water is taken out from the
tank, find the drop in the water level in the
tank.
(Use = 22/7)
Water flows out through a circular pipe whose
internal diameter is 2 cm, at the rate of 7
metres per second into a cylindrical tank the
radius of whose base is 40 cm. By how much
will the level of water rise in half an hour?
Into a circular drum of radius 4.2 m and height
3.5 m, how many full bags of wheat can be
emptied if the space required for wheat in
each bag is 2.1 cubic m.
(Take = 22/7)
How many cubic metres of earth must be dugout
to sink a well 22.5 m deep and of diameter 7
m? Also, find the cost of plastering the inner
curved surface at Rs 3 per square metre.
The diameters of the internal and external
surface of a hollow spherical shell are 6 cm
and 10 cm respectively. If it is melted and
2
recast into a solid cylinder of height 2
cm
3

9.

find the diameter of the cylinder.


(Take = 22/7)
A cylindrical water tank of diameter 1.4 m
and height 2.1 m is being fed by a pipe of
diameter 3.5 cm through which water flows
at the rate of 2 m/s. Calculate in minutes the
time it takes to fill the tank.
(Use = 22/7)
Height of a solid cylinder is 10 cm and
diameter 8 cm. Two equal conical holes have
been made from its both ends. If the diameter
of the hole is 6 cm and height 4 cm, find
(i) Volume of the cylinder.
(ii) Volume of one conical hole.
(iii) Volume of the remaining solid.
A milk tanker cylindrical in shape having
diameter 2 metr es and length 4.2 metres
supplies milk to the two booths in the ratio
3 : 2. One of the milk booths has a rectangular
vessel having base areas 3.96 sq m and the
other has a cylindrical vessel having diameter
2 metres. Find the level of milk in each of
the two vessels.
Two cylindrical vessels are filled with oil.
The radius of one vessel is 15 cm and its
height is 25 cm. The radius and height of
t he other v essel ar e 10 cm and 18 cm
respectively. Find the radius of a cylindrical
v essel 30 cm in height , which will just
contain the oil of the two given vessels.
The radius of the base of a right circular cone
is 14 cm and altitude 20 cm. What is the
largest lateral surface area possible for a
cylinder inscribed in this cone?
The thickness of a metallic tube is 1 cm and
the inner diameter of the tube is 12 cm. Find
the weight of 1 m long tube, if the density of
the metal be 7.8 gm per cm3.
A cylindrical road roller made of iron is 1 m
wide. It s inner diameter is 54 cm and

K KUNDAN
10.

11.

12.

13.

14.

15.

614

Concept of Arithmetic

thickness of the iron sheet rolled into the


road roller is 9 cm. Find the weight of the
roller if 1 cc of iron weighs 8 gm.
16. The volume of a metallic cylindrical pipe is
748 cm3. Its length is 14 cm and its external
radius is 9 cm. Find its thickness.
17. Increasing the radius of the base of cylinder
by 6 units increase the volume by y cubic units.
Increasing the altitude of the cylinder by 6
units also increase the volume by y cubic units.
If the original altitude is 2 units, find the
original radius.
18. The dept h of the wat er in a r ectangular
swimming pool increases uniformly from 1
metre at the shallow end to 3.5 metres at the
deep end. The pool is 25 metres long and 12
metres wide. Calculate the volume of the water
in the pool in cubic met res. The pool is
empt ied by means of cylindr ical pipe of
internal radius 9 cm. The water flows down
the pipe at a speed of 3 metres per second.

Calculate the number of litres emptied from


the pool in 1 minute giving your answer to
the nearest 10 litres.
(1 litres = 1000 cm3).
(Take = 3.142)
19. The total surface area of a right triangular
prism of the height 4 cm is 72 3 cm2. If the
base of the prism is an equilateral triangle,
find its volume.
2 0 . The sum of radius of the base and height of a
right circular cylinder is 37 cm. If the total
surface area of the cylinder is 1628 cm2, find
its volume.
(Use = 22/7)
21. Water is flowing at the rate of 5 km/hr through
a cylindrical pipe of diameter 14 cm into a
rectangular tank which is 50 m long and 44
m wide. Determine the time in which the
level of water in the tank will raise by 7 cm.
(Use = 22/7)

Exercise3
(Pyramid and Circular Cone)
1.

Find the lateral surface area, whose surface


area and v olume of a right pyramid with
equilateral triangle as a base in which the
length of each side of the base is 4 cm and
slant height is 5 cm.
Find the volume, lateral surface area and total
surface area of a right triangular pyramid the
length of whose edge is 10 cm.
The base of a right pyramid is an equilateral
triangle of side 10 cm and its vertical height
is 5 cm find its
(i) slant height
(ii) area of one side face.
A right pyramid has an equilateral triangular
base of side 4 units. If the number of square
units of its whole surface area be three times
the number of cubic units of its volume, find
its height.
The base of a right pyramid is an equilateral
triangle each side of which is 2 m long. Every
slant edge is 3 m long. Find the lateral surface
area and the volume of the pyramid.
The base of a right pyramid is an equilateral
t riangle of side 4 cm. The height of the
pyramid is half of its slant height. Find the
volume and the length of a slant edge of the
pyramid.
The volume of a right circular cone whose
radius of the base is 21 cm is 12936 cm 3.
Find the curved surface area of the cone.
(Take = 22/7)
The radius of the base and the height of a
right circular cone are respectively 21 cm and

28 cm. Find the curved surface area, total


surface area and the volume of the cone.
(Take = 22/7)
The radius and height of a cone are in the
ratio 3 : 4. If its volume is 301.44 cm3, what
is its radius? What is its slant height?
(Take = 22/7)
The curved surface area of a cone is 4070
cm 2 and its diameter is 70 cm. What is its
slant height?
(Use = 22/7)
The radius and slant height of a cone are in
the ratio of 4 : 7. If its curved surface area is
792 cm2, find its radius.
(Use = 22/7)
The circumference of the base of a 10 m high
conical tent is 44 metres. Calculate the length
of canvas used in making the tent if width of
canvas is 2 m.
(Use = 22/7)
How many metres of cloth 5 m wide will be
required to make a conical tent, the radius of
whose base is 7 m and whose height is 24
m?
(Use = 22/7)
The volume of a right circular cone is equal
to that of a right circular cylinder 9 cm high
with diameter of the base 60 cm. The height
of the cone is 108 cm. Find the diameter of
the base of the cone.
A conical vessel, whose internal radius is 10
cm and height 48 cm, is full of water. If this
water is poured into a cylindrical vessel with

K KUNDAN

2.

3.

4.

5.

6.

7.

8.

9.

10.

11.

12.

13.

14.

15.

615

Mensuration II (Surface Area and Volume)


radius 20 cm, find the height to which the
water rises in it.
(Use = 22/7)
16. The radii of the ends of a bucket 45 cm high,
which is in the form of a frustum of a cone,
are 28 cm and 7 cm. Determine its capacity
and the surface area.
(Take = 22/7)
17. A glass piston is in the shape of a frustum of
a cone, having its top and bottom diameters
as 3 cm and 7 cm respectively. If the frustum
is 4 cm high, find its weight if 1 cm3 of glass
weighs 2.1 gm.

18. A right circular cone is cut by two planes


parallel to the base such that the height is
divided into three equal parts. Compare the
volumes of the three parts of the cone.
19. The radius of the base and height of a right
circular cone are in the ratio 5 : 12. If its
volume is 2512 cm 3, find its curved surface
area and total surface area.
2 0 . Water flows at the rate of 10 metres per minute
through a cylindrical pipe whose internal
radius is 0.5 cm. How long would it take to
fill a conical vessel whose radius at the top
is 20 cm and depth is 21 cm?

Exercise4
(Sphere, Hemisphere and Spherical Shell)
1.

2.

3.

4.

5.

Find the volume and surface area of a sphere


of radius 4.2 cm.
(Take = 22/7)
Find the volume and the total surface area of
a hemisphere of radius 3.5 cm.
(Use = 22/7)
The inter nal and exter nal diameters of a
hollow hemispherical vessel are 24 cm and
25 cm respectively. The cost to paint 1 cm 2
the surface is Re 0.05. Find the total cost to
paint the vessel all over.
(Use = 22/7)
A hollow spherical shell is made of a metal of
density 4.9 g/cm3. If its internal and external
radii are 10 cm and 12 cm respectively, find
the weight of the shell.
A sphere and a cube have the same surface.
Show that the ratio of the volume of sphere to

7.

A vessel is in the form of an inverted cone.


Its height is 8 cm and the radius is 5 cm. It is
filled with water upto the brim. When lead
shots each of which is a sphere of radius 0.5
cm are dropped into the vessel, one fourth of
the water flows out. Find the number of lead
shots dropped into the vessel.
9.
A solid is composed of a cylinder wit h
hemispherical ends. If the whole length of
the solid is 108 cm and the diameter of the
hemispherical ends is 36 cm, find the cost of
polishing the surface of the solid at the rate
of 7 paise per sq cm.
(Use = 22/7)
10. Three identical balls f it snugly into a
cylindrical can. The radius of the spheres
equal the radius of the can, and the balls just
touch the bottom and the top of the can. If
the formula for the volume of a sphere is V

K KUNDAN
that of the cube is

6.

8.

6: .
A measuring jar of internal diameter 10 cm is
partially filled with water. Four equal spherical
balls of diameter 2 cm each are dropped in it
and they sink down in the water completely.
What will be the change in the level of water
in the jar?
Metal spheres, each of radius 2 cm are packed
into a rectangular box of internal dimension
16 cm 8 cm 8 cm. When 16 spheres are
packed the box is filled with preservative
liquid. Find the volume of this liquid. Give
your answer to the nearest integer.
669

Use 213

4 3
r , what fraction of the volume of the
3
can is taken up by the balls?
11. A sphere of maximum volume is cut out from
a solid hemisphere of radius r. Find the ratio
of the volume of the hemisphere to that of the
sphere.
12. The ratio of the volumes of a right circular
cylinder and sphere is 3 : 2. If the radius of
the sphere is double the radius of the base
of the cylinder, find the ratio of the surface
areas of the cylinder and sphere.
=

616

Concept of Arithmetic
Exercise5
(Surface Area and Volume of Combination of Solids)

1.

2.

3.

4.

Find the volume of the largest right circular


cone that can be cut out of a cube whose edge
is 9 cm.
From a right circular cylinder with height 10
cm and radius of base 6 cm, a right circular
cone of the same height and base is removed.
Find the volume of the remaining solid.
A wooden toy is in t he f orm of a cone
surmounted on a hemisphere. The diameter
of the base of the cone is 6 cm and its height
is 4 cm. Find the cost of painting the toy at
the rate of Rs 7 per 100 cm2.
(Use = 22/7)
Rasheed got a playing top (lattu) as his birthday
present, which surprisingly had no colour
on it. He wanted to colour it with his crayons.
The top is shaped like a cone surmounted by
a hemisphere (see the figure given below).
The entire top is 5 cm in height and the
diameter of the top is 3.5 cm. Find the area
he has to colour.
(Take = 22/7)

8.

9.

A circus tent is cylindrical upto a height of 7 m


and conical above it. The diamet er of the
cylindrical portion is 10 m and the total height
of the tent from ground to the vertex of the
conical part is 19 m. Find the cost of canvas
required to build the tent at the rate of Rs 7.70
per square metre.
(Use = 22/7)
A solid is in t he f orm of a right circular
cylinder with hemispherical ends. The total
length of the solid is 35 cm. The diameter of

1
of its height. Find the
4
volume and total surface area of the solid.
(Use = 22/7)
10. The decorative block shown in the figure is
made of two solidsa cube and a hemisphere.
The base of the block is a cube with edge 5
cm, and the hemisphere fixed on the top has
a diameter of 4.2 cm. Find the total surface of
the block.
(Take = 22/7)
the cylinder is

K KUNDAN

5.

6.

7.

A wooden show-piece is of the shape of a


cylinder surmounted by a hemisphere. The
diameter of the cylinder is 42 cm and its
height is 40 cm. Find the cost of polishing
the surface area of the show-piece at the rate
of Rs 10 per 1000 cm2 (excluding the base of
the show-piece).
From the figure given below, find the volume
of the toy.

A sphere has a diameter of 500 3 cm. A


biggest cube is fitted in it. Now a biggest
sphere is fitted within this cube. Again a
biggest cube is fit ted within t his smaller
sphere. Determine the ratio of volume of
bigger cube to the volume of smaller cube.

11.

617

Mensuration II (Surface Area and Volume)


A wooden toy rocket is in the shape of a cone
mounted on a cylinder, as shown in the above
figure. The height of the entire rocket is 26
cm, while the height of the conical part is 6
cm. The base of the conical portion has a
diameter of 5 cm, while the base diameter of
the cylindrical portion is 3 cm. If the conical
port ion is t o be paint ed or ange and t he
cylindrical portion yellow, find the area of
the rocket painted with each of these colours.
(Take = 3.14)
12. Mayank made a bird-bath for his garden in
the shape of a cylinder with a hemisphere
depression at one end (see the figure). The
height of the cylinder is 1.45 m and its radius
is 30 cm. Find the total surface area of bird
bath.
(Take = 22/7)

the volume of air that the shed can hold.


Further, suppose the machinery in the shed
occupies a total space of 300 m 3, and there
are 20 workers, each of whom occupy about
0.08 m3 space on an average. Then how much
air is in the shed?
(Take = 22/7)

14. A juice sellor was serving his customers using


glasses as shown in the given figure. The
inner diameter of the cylindrical glass was 5
cm, but t he bot t om of t he glass had a
hemisphere raised portion which reduced the
capacity of the glass. If the height of a glass
was 10 cm, find the apparent capacity of the
glass and its actual capacity.
(Take = 3.14)

K KUNDAN

13. Shanta runs an industry in a shed which is


in the shape of a cuboid surmounted by a half
cylinder (see the figure). If the base of the
shed is of dimension 7m 15 m, and the
height of the cuboidal portion is 8 m, find

Exercise6
(Conversion of Solid From One Shape to Another)
1.

2.

3.

4.

5.

A metal (lead) cube of dimension 22 cm has


been recasted into spherical balls of diameter
2 cm. How many such balls will be formed?
The radius of a solid metallic sphere is 1.5
cm. It is melted and drawn into a wire of
diameter 0.2 cm. Find the length of the wire.
A solid sphere of radius 3 cm is melted and
then cast into small spherical balls each of
diameter 0.6 cm. Find the number of balls
thus obtained.
A solid metallic right circular cylinder of base
diameter 16 cm and height 2 cm is melted
and cast into a right circular cone of height
three times that of the cylinder. Find the
curved surface area of the cone.
(Take = 3.14)
A cone of height 24 cm and radius of base 6
cm is made up of modelling clay. A child
reshape it in the form of a sphere. Find the
radius of the sphere.

6.

7.

8.

Selvis house has an overhead tank in the


shape of a cylinder. This is filled by pumping
water from a sump (an underground tank)
which is in the shape of cuboid. The sump
has dimensions 1.57 m 1.44 m 95 cm.
The overhead tank has its radius 60 cm and
height 95 cm. Find the height of the water
left in the sump after the overhead tank has
been completely filled with water from the
sump which had been f ull. Compar e the
capacity of the tank with that of the sump.
(Take = 3.14)
A copper rod of diameter 1 cm and length 8
cm is drawn into a wire of length 18 m of
uniform thickness. Find the thickness of the
wir e.
A hemispherical tank full of water is emptied
by a pipe at the rate of 3

4
litres per second.
7

618

Concept of Arithmetic
How much time will it take to empty half the
tank, if it is 3 m in diameter?
(Take = 22/7)
The radius of the base and height of a metal
solid cylinder are r cm and 3 cm respectively.
It is melted and recast into a cone of the same
radius of base. Find the height of the cone.

9.

10. A solid metallic right circular cylinder of basediameter 16 cm and height 2 cm is melted
and recast into a right circular cone of height
three times that of the cylinder. Find the
curved surface area of the cone.
(Use = 3.14)

Answers and explanations


Exercise1
1.

We have,
Area of the field = 1 hectare = 10000 m2
[ 1 hectare = 10000 m2]
Depth of water on the field

Let the rise in water level be x cm.


Clearly, volume of the cube = volume of the
water replaced by it.
Volume of the cube = volume of a cuboid of
dimension 15 cm 12 cm x cm.
or, 729 = 15 12 x
729
or, x =
cm
15 12

5
1
m
m.
100
20
Volume of water = Area of the field depth
= 5 cm =

2.

m3 = 500 m3
of water = 10000
20

= 500 1000 litres [ 1 m3 = 1000 litres]


= 500000 litres
We have,
Volume of water pumped out from the reservoir
8400
= 8400 litres =
m 3 [ 1000l = 1 m3]
1000
= 8.4 m3
Area of the base of the reservoir
= (7 6) m2 = 42 m2.
Fall in water-level of the reservoir

81
cm = 40.5 cm
20
Let the length, breadth and height of the
cuboid be l, b and h units respectively.
According to the question,
2 (lb + bh + lh) = 22
....(i) and
4 (l + b + h) = 24
x =

5.

K KUNDAN
=

3.

24
= 6
....(ii)
4
2
2
2
2
Now, (l + b + h) = l + b + h + 2 (lb + bh + lh)
or, (6)2 = l2 + b2 + h2 + 22
or, l2 + b2 + h2 = 36 22 = 14
or, l + b + h =

Volume of water pumped out


Area of the base

Diagonal of cuboid =

8.4
m = 0.2 m
=
42
= (0.2 100) cm = 20 cm
Length of an edge of the cubical block of ice
= 50 cm =

6.

50
1
m m
100
2
2

1 3
1
Volume of the block of ice = m m .
8
2
Since one cubic metre of ice weighs 900
kilograms. Therefore,
weight of the cubical block

4.

= 900 kg = 112.5 kg
8

Edge of the given cube = 9 cm.


Volume of the cube = (93) cm3 = 729 cm3
If the cube is immersed in the vessel, then
the water level rises.

14
= 3.74 cm
Area of the field = 35 15.4 = 539 m2
Area of the rectangular tank = 5.5 4 = 22 m2
Area of the remaining field where earth is
spread out = (539 22) m2 = 517 m2
The volume of earth dug out = 5.5 4 2.5 m3
Let us assume that the level of field be raised
by h metres on spreading the earth over it.
517 h 5.5 4 2.5
5.5 4 2.5
517
or, h = 10.6 cm (approx.)
Slove as Q.No. 6.
Area of the field = (40 28 =) 1120 sq m
Area of the pit = (12 6 =) 72 sq m
Volume of the pit = (12 6 5 =) 360 m3
Ar ea of the r emaining field where soil is
spread
= Total area of the field Area of the pit
= (1120 72 =) 1048 m2
or, h =

7.

l 2 b2 h 2 =

619

Mensuration II (Surface Area and Volume)


Rise in height after spread of the soil will be
equal to the volume of soil taken out from the
pit
So,
Remaining area Raised height = Volume of
pit
1048 raised height = 360

360
= 0.34 metre or 34 cm
1048
The required height is 34 cm.
Let the edge of the cube = a cm
Raised height =

8.

Diagonal of cube =
or,

17 .32
3

or,

(2.5)3
h
0.25 1.25

2.5 2.5 2.5


= 50 cm
0.25 1.25
Surface area of cube
= 6a2 = 6 (2.5)2 = 6 6.25 = 37.50 cm2
Surface area of cuboid = 2 (lb + bh + hl)
= 2(1.25 0.25 + 0.25 50 + 50 1.25)
= 2(0.3125 + 12.50 + 62.50)
= 2 75.3125 = 150.625 cm 2
Increase in Surface Area
= (150.625 37.50 =) 113.125 cm 2
13. Given length of swimming pool, l = 32 m
Width of swimming pool, W = 9.5 m
Depth of the swimming pool at one end,
h1 = 1.5 m
Depth of the swimming pool at the other end
h2 = 4.5 m
The cross section of swimming pool is in the
shape of trapezium
or, h =

3 a = 17.32

or, a =

9.

3 (edge)

This becomes the volume of the recasted cube.


Now, the volume of new recasted cube.
L3 = 216
or, L = 6 cm
The required answer is 6 cm.
12. As the metallic cube is melted and recasted
into the form of a cuboid,
The volume of metallic cube = volume of cuboid
(Side) 3 = length breadth height
(2.5)3 = 1.25 0.25 h

17 .32
= 10
1.732

Volume = a3 = (10 10 10 =) 1000 cm3.


Volume of the water that flows in 1 second
= (10 0.05 0.03) m3

10 5 3 3
15 3
m =
m
=
100

100

1000
Volume of the water that flows in 40 minutes
15

60 40 m3 = 36 m3
=
1000

Area of the base of cuboid = (7.2 2.5) m2

K KUNDAN
36
m = 2m
Height of water level =
7.2 25

10. Let the length and breadth of the new bar be


x metres.
Volume of the bar = 4 x x m3
Volume of cubical bar
= length breadth height
Volume of the metal cube = 1 m3
Volume of metal cube melted = Volume of bar
formed
4 x x = 1
1
2
r, x
4
or, x

1
m
2

The edges of cube =

1
m
2
3

1
1
volume of cube = m3
8
2
The weight of 1 m3 volume of cube = 480 kg

1 480
60 kg
The weight of the cube =
8

11. Volume of the rod = (19 4 2 =) 152 cm3


Volume of the cube = (43 =) 64 cm3
Total volume of rod and cube
= (152 + 64) cm3 = 216 cm3

1
l h1 h 2
2
Volume of water in swimming pool
= Area of cross section width
Area of trapezium =

= l h1 h 2 w
2

= 32 (1.5 4.5 ) 9.5


2

= 16 6 9.5 = 912 cu m
14. Let the volume of aquarium be x litres.
According to the question,

2
x
3
Quantity of water left after tilting the aquarium
the quantity of water in it =

x
2

620

Concept of Arithmetic
According to the question,

2
x
x 6 litres
3
2
4x 3x
6
6
or, x = 36 litres.
The volume of tank is 36 litres.
15. Let the height of the room be a m.
The breadth will be 2a m.
The length of the room is 18 m (given)
Area of the four walls
= 2(length + breadth) height
Area of the four walls of the room
= 2a(18 + 2a) = (36a + 4a2) m2
Area of 70 square papers
= (70 2 2 =) 280 m2
As the wall ar e to be covered f rom these
papers.
Area of four walls = Area of wall paper
or,

17. Internal length = 85 cm


Internal breadth = 65 cm
Internal height = 72.5 cm
Thickness of wood = 25 mm = 2.5 cm
External length = (85 + 2 2.5 =) 90 cm
External breadth = (65 + 2 2.5 =) 70 cm
External height = (72.5 + 2.5 =) 75 cm.
Area which is to be painted
= Area of the external walls of the box
= 2 (l + b) h = 2 (90 + 70) 75
= 24000 sq cm

24000
sq dm = 240 sq dm
100
Cost of painting at 15 per sq dm
= Rs (240 15 =) Rs 3600.
18. ABCD is the square court having each side
equal to 17 metres.
=

or, 36a 4a 2 280


or, 4a 2 36a 280 0
or, a 2 9a 70 0

(on dividing by 4)

or, a 2 14a 5a 70 0
or, a (a 14) 5 (a 14 ) 0

Walls are built along AB, BC, CD, DA. Note


that wall also exists at the four corners A, B,

K KUNDAN

or, (a 5) (a 14) 0
or, a = 5 or 14
Negative height is not possible
a = 5 metres
When height = a = 5 m
Breadth = 2a = (2 5 =) 10 m
Area of floor = l b = (18 10 =) 180 m2
Cost of flooring at the rate of Rs 20 per m2
= (180 20 =) Rs 3600
16. Let the height of the hall be h m.
Then, sum of the areas of four walls
= 2(l + b) h m2
= 2(20 + 16)h m2
= 72h m2
Sum of the areas of the floor and the flat roof
= (20 16 + 20 16) m2
= 640 m2
It is given that the sum of the areas of four
walls is equal to the sum of the areas of the
floor and roof.
72 h = 640

640
80
m =
m = 8.88 m
72
9
So, height of the hall = 8.88 m
or, h =

80

m3
Volume of the hall = 20 16
9

25600
m3 = 2844.4 m3
9

C, D. There is a gate of 1

1
metres width on
2

each side of the square.


Length of wall along AB

= 17 metres 1

1
metres
2

1
metres
2
length of wall along AB, BC, CD, DA
= 15

31
4 = 62 metres
2
Length of wall at the four corners A, B, C, D
=

1
= 1 metre.
4
length of the whole wall
= 62 metres + 1 metre = 63 metres
Volume of the whole wall
= 4

= 63 3

1
cub metres
4

189
cub metres
4
189
2500
4
= Rs 118125.

cost of wall = Rs

621

Mensuration II (Surface Area and Volume)


19. Internal length = (100 5 =) 95 cm
Internal breadth = (85 5 =) 80 cm
Internal height = (65 5 =) 60 cm
External volume of box = 100 85 65
= 552500 cub cm
Internal volume of box = 95 80 60
= 456000 cub cm
volume of wood
= (552500 456000 =) 96500 cub cm
= .0965 cub m
2.5

sq m
area of planking = .0965
100

100
sq m = 3.86 sq m.
2.5
2 0 . Length = 2.5 km = 2500 m
Width = 20 m, Depth = 1.2 m
The volume of water passing the ferry per
hour is that of a cuboid whose dimensions
are given above. Hence,
Volume of water passing in 1 hour
= 2500 20 1.2 cub m
The number of litres of water passing in 1
minute
= .0965

2500 20 1.2 1000


= 1000000
60
3
128
21. Length = 25 metres =
metres
5
5
=

Thickness of iron = 4 cm
Internal dimensions
l 292 cm

b 200 cm
h 176 cm (because the cistern is open)
External volume = l b h
= (300 208 180 =) 11232000 cu cm.
Internal volume = l b h
= (292 200 176 =) 10278400 cu cm.
( i ) Volume of the metal of cistern
= External volume Internal volume
= (11232000 10278400 =) 953600 cu cm.
Weight of the cistern = Volume Density
= 953600 7.2 = 6865920 gm
= 6865.92 kg
(ii) Volume of water
= Internal volume of the cistern
= 10278400 cu cm
Weight of the water = 10278400 1
= 10278400 gm = 10278.4 kg
Total weight
= Weight of the cistern + Weight of water
= (6865.92 + 10278.4 =) 17144.32 kg
Hence weight of cistern full of water
= 17144.32 kg
23. Rate of flow of water = 5 km/hr
55
25
= m/sec =
m/sec
18
18
Volume of water flowing in 1 second

K KUNDAN
1
96
metres =
metres
5
5

Breadth = 19

128
96


metres
5
5

diagonal =

16384 9216
metres

25
25

256000
metres = 32 metres
25

22
7
7
25
154

m3
7 100 100 18 7200
Level of water in the tank = 14 cm = 0.14 m
Volume of water in the tank
= 100 44 0.14 m3
required time
=

Amount of water

128 96

14 1000 kilograms
5
5
= 6881.28 metric tonnes
2 2 . External dimensions
l = 300 cm
b = 208 cm
h = 180 cm

2 25
(0.07 ) 18 m3

[ Volume of the cylinder = r2h]

100 44 0.14 7200


seconds
154
= 28800 seconds
=

28800
hours = 8 hours
3600
24. Volume of water in the first cistern
= (5 3 2.5 =) 36 cu m
Volume of water in the second cistern
= (4 3.6 2 =) 28.8 cu m
A pipe of radius of 3 cm gives, 36 cu m
water in 25 minutes.
A pipe of radius of 5 cm will give water
28.8 cu m
=

25 3 28.8
= 12 minutes
36 5

622

Concept of Arithmetic
Exercise2

1.

Let the sides be a = 17 cm, b = 25 cm, c = 28 cm

Volume of embankment

Area of a triangle =

12.25 h

= (196 12 .25 ) h
= 183 .75 h m3

s (s a ) (s b ) (s c )
Clearly, 183.75 h

Hence, area of the base = 35 18 10 7 sq cm


= 210 sq cm
Volume of the prism = Area of the base Height

2.

14

= 10.5 3.5 2 3.52 h

a b c 17 25 28

= 35 cm
Then, s =
2
2

(s a) = (35 17 =) 18 cm
(s b) = (35 25 =) 10 cm
(s c) = (35 28 =) 7 cm

4200
20 cm
Height of the prism =
210
Lateral Area = Perimeter of the base Height
= (17 + 25 + 28) 20
= 1400 sq cm
Let the height of each prism be h units and
the length of each side of equilateral triangle
at the base of first prism be a units and that
the second prism having regular hexagon as
base be b units. (See the figures given below)

49
22.5
4

49 22.5
3
= 1.5
183.75 4 2
Hence, the required height = 1.5 metres.
Here, the diameter of the cylindrical tank
= 35 cm
Therefore, the base area of the tank
or, h =

4.

22 35

7 2

Now, suppose the drop in the water level is x


cm. Then
2

22 35

x 11 1000 cm 3
7 2

K KUNDAN
[ 1 litre = 10003]

11000 7 2 2 308000

11.428 cm
22 35 35
26950
Hence, required drop in the water level
= 11.428 cm
Given
Speed of the water = 7 m/s = 700 cm/s
Radius of pipe = 1 cm
Area of cross section of pipe = (1)2 = cm2
Volume of water falling from pipe per second
= 700 sq cm
Time = (30 60 =) 1800 seconds
Volume of water falling in half an hour
= 700 30 60 cu cm
This v olume and v olume of wat er in
cylindrical tank should be equal. So taking
the height of water in cylindrical tank as h
cm,
40 40 h 700 30 60
x=

5.

II

According to the question,


Volume of first prism = Volume of second prism
3 2
3 3 2
a h
b h
4
2

or,

1 2 3 2
a b
4
2

or, a 6b

or, a 2 6b 2
or,

a
6

b
1

700 30 60
= 787.5 cms
40 40
or, 7.875 metres
Here, r = 4.2 m, h = 3.5 m.
Volume of the drum
= (r2h) m3 = (3.14 (4.2)2 3.5) m3
Volume of each bag of wheat = 2.1 cubic metres
or, h =

a :b 6 :1
3.

Volume of earth taken out from the well


2

= r h
2

49
7

22.5 m3
= 22 .5
4
2

Let the height of embankment be h metres.

6.

Volume of drum
Number of bags = Volume of a bag

623

Mensuration II (Surface Area and Volume)


3.14 4.2 4.2 3.5
2.1
= 3.14 8.4 3.5
= 92.316 = 92 bags nearly.
Volume of the earth to be dugout = Volume of
the well
=

7.

22 7 7

22.5 m3 = 866.25 m3
=
7 2 2

Area of the inner curved surface = 2rh

8.

22 7

22 .5 m2 = 495 m2
= 2
7 2

Cost of plastering the inner curved surface


= Rs (495 3) = Rs 1485.
External radius of the spherical shell (R)

1
10 cm = 5 cm
2
Internal radius of the spherical shell (r)
=

1
6 cm = 3 cm
2
Volume of the hollow spherical shell
=

4
4
R3 r 3 =
5 3 33
3
3

cm

Let the tank be filled in x seconds


Volume of water flowing per second
2

7
= 200 cm3
4
[Here h = 2 m = 200 cm]
Volume of water flowing in x seconds
2

7
= x 200 cm3
4
Again volume of cylindrical tank
= (70 )2 210 cm3
According to the question,
Volume of water filled by the pipe in x seconds
= volume of the tank
2

7
2
x 200 (70 ) 210
4

K KUNDAN
=

4 22

125 27 cm3
3 7

4 22

98 cm3
3 7
Let the radius of the solid cylinder be r cm
Height of the solid cylinder (h)
=

2
cm = 8 cm
3
3
Volume of the solid cylinder
= Volume of the hollow spherical shell
= 2

or, r 2h

or,

4 22

98
3 7

22 2 8 4 22
r
98
7
3 3 7

or, r 2

4 22
7 3

98
49
3 7
22 8

r=

9.

49 = 7
Hence, diameter of the solid cylinder
= 2 radius = (2 7 =) 14 cm
Diameter = 3.5 cm

3.5

2
Radius = 0.7 m =
Height = 2.1 m =
Radius =

35 7
cm

20 4
70 cm
210 cm

or, x =

70 70 210
7 7
200
4 4

70 70 210 4 4
7 7 200
= 1680 seconds = 28 minutes
1680

60 28

Hence it takes 28 minutes to fill the cylindrical


water tank.
10. Volume of the cylinder = r2h
=

2
= (4) 10 160 cm3

Volume of one cone =


=

1 2
r h
3
1
32 4 12 cm3
3

624

Concept of Arithmetic

Volume of third cylinder =

163350
cm3
7

Its height = 30 cm
Let its radius be x cm

22
x 2 30 cm3
7

Its volume = r 2h

Volume of two cones = (2 12 =) 24 cm3


Hence volume of the remaining solid
= (160 24 )136 cm3
11. Diameter of the milk tanker = 2 m
Radius of the tanker = 1 m
Length of milk tanker = 4.2 m
2
Volume of the tanker = r h

22 x 2 30 163350
7
7
or, x 2

163350 7
1
495

7
22 30
2
12

x =

495 495

2
2

15.73 cm

13. Taking first of all general cases,

22
12 4.2 cu m = 13.2 cu m
7
Ratio of the volume of milk in rectangular and
cylindrical vessels = V1 : V2 = 3 : 2
or, V =

V1 =

3
13.2 m3 = 7.92 m3
5

2
13.2 m3 = 5.28 m3
5
Area of the base of rectangular vessel = 3.96 m2
Let h1 be the height of milk in this vessel
3.96h1 = 7.92
and V1 =

K KUNDAN
7.92
2 m
or, h1 =
3.96
Radius of cylindrical vessel = 1 m
Let h2 be the height of milk in this cylinder

5.28 7
1.68 m
h2 =
22

Height of milk in rectangular vessel = 2 m


and height of milk in cylindr ical v essel
= 1.68 m.
12. Volume of first cylinder = r12h1
=

22
123750
cm 3
15 15 25 cm3 =
7
7

Let a cylinder with base radius r and altitude


h be inscribed in t he cone. From the
similarity of the triangle AOS and BO 1S, it
follows that

123750 39600 163350

=
cm3
7
7
7

Since the third cylinder contains the volume


of two given vessels.

....(i)

The lateral surface area of cylinder S = 2rh ,

2R
h H h {substituting (1) for r}.
H
From geometric consideration, h varies in the
interval 0 < h < H.
or S =

S = 2Rh

2
Volume of second cylinder = r2 h 2

22
39600
=
cm 3
10 10 18 cm3 =
7
7
Combined volume of f ir st and second
cylinder

R (H h )
H

r=

2Rh 2
H

RH 2R
H

h
2
H
2

(isolating the perfect square)


S is maximum when 2nd term is zero,
ie h

H
O
2

ie h

H
.
2

625

Mensuration II (Surface Area and Volume)


Maximum value of lateral surface area

1
RH
2
Radius = 14 cm
Height = 20 cm
Maximum value of lateral surface area
=

1 22

14 20 440 cm2.
=
2 7

14. We have,
inner diameter of the tube = 12 cm
r = Inner radius of the tube = 6 cm.
Thickness of the tube = 1 cm.
R = Outer radius of the tube
= (6 + 1) cm = 7 cm.
h = Length of the tube = 1 m = 100 cm
Volume of the metal in the tube
2
2
= (R r )h

22
(7 2 62 ) 100 cm3
7

22
13 100 cm3
7
Density of the metal = 7.8 gm/cm3
Weight of the tube = Volume Density
=

16. We have
R = external radius of pipe = 9 cm
h = length of the pipe = 14 cm
V = volume of the pipe = 748 cm3
Let r be the internal radius in centimetres.
Th en ,
volume = 748 cm3
or, (R 2 r 2 )h 748 cm3
or,

22 2
9 r 2 14 748
7

748
44
or, 81 r2 = 17
or, r2 = 64
or, r = 8 cm
Hence, thickness of the pipe
= (R r) cm = (9 8) cm = 1 cm.
17. Let the original radius of cylinder be x units.
Given, height of cylinder = 2 units
Volume of cylinder = r2h
2
or, 81 r

Volume = x 2 2 2x 2
When the height of cylinder increases by 6
2
2
units, the new volume is = x 2 6 8 x
Increase in volume due to increase in height

K KUNDAN
22
13 100 7.8 gm
7
= 31868.57 gm
=

31868.57
kg = 31.86857 kg
1000
15. The width of the road roller is 1 m ie 100 cm.
So, height (length) of the cylinder = 100 cm
Inner radius of the cylinder = r
=

54
=
cm = 27 cm.
2
Thickness of the iron sheet = 9 cm.
Outer radius of the cylinder = R
= (27 + 9) cm = 36 cm.
Thus, volume of the iron sheet used

= 8x 2 2x 2 y units .... (i)

New volume due to increase in radius by 6


units
2
= x 6 2

Increase in volume due to increase in radius


by 6 units = x 62 2 2x 2 y units .... (ii)
Computing the equations (i) and (ii) above

x 62 2 2x 2 8x 2 2x 2
or, x 62 2 8 x 2
or, x 2 36 12x 4x 2
or, 36 12x 3x 2

= (R 2h r 2h ) cm3

or, 3x 2 12x 36 0

2
2
= (R r )h cm3

or, x 2 4x 12 0

= (R r ) R r h cm3
= [3.14 (36 + 27) (36 - 27) 100] cm3

or, x 2 6x 2x 12 0
or, x x 6 2x 6 0

314
63 9 100 cm3
100
= 178038 cm 3
Weight of the roller = 178038 8 gm
=

= 178038

8
kg = 1424.304 kg.
1000

or, x 2x 6 0
or, x + 2 = 0 or, x 6 = 0
x = 2 or x = 6
Ignoring the negative value, the radius
= 6 units.
18. The cross-section of swimming pool is of the
shape of trapezium
Sum of parallel sides = (1 + 3.5 =) 4.5 m

626

Concept of Arithmetic
Area of trapezium

or, a 4 3

1
=
length sum of parallel sides
2

Volume of the prism


= Area of the base Height

1
25 4.5 sq m
2
Volume = Area of cross section width
1
25 4.5 12
2
= 675 cm3
=

2
Volume of cylinder = r h
Volume of water which flows out from pipe
in one second
= 3.142 9 9 300 cm3
Volume of wat er which f lows out in 60
seconds
= 3.142 9 9 300 60 cm3
= 4581036 cm 3
= 4581.036 litres
[ 1 litre = 1000 cm3]
19. Let each side of the base of the prism be a
cm. Then,

4 cm

3
4 3
4

Area of cross-section =

= 48 3 cm3
2 0 . Let the radius of the base of the right circular
cylinder be r cm and its height be h cm.
Now, according to the question,
h + r = 37
......(i)
Total surface area of the cylinder = 1628 cm2
or, 2rh 2r 2 1628
or, 2r (h r ) 1628
or, 2r 37 1628
or, 2

(Using equation (i))

22
37 r 1628
7

1628 7
= 7
2 22 37
r = 7 cm and h = (37 7 =) 30 cm
or, r =

Total surface area = 72 3 cm2

Volume of the cylinder = r 2h

or, Perimeter of the base height + 2(Area of

22

7 30 cm3 = 4620 cm3


=
7

21. Let the level of water in the tank be rise 7 cm


in x hours.
Rate of flowing water = 5 km/hr

the base) = 72 3

K KUNDAN
3 2
or, 3a 4 2
a 72 3
4

3a 2 24a 144 3 0

or,

5 25

= 5
m/sec
18 18

Volume of water flowed through pipe in x


hours
= Volume of water in the tank

or, a 2 8 3a 144 0

or, a 2 12 3a 4 3a 144 0

3 0

or, a a 12 3 4 3 a 12 3 0

or, a 4 3 a 12

or, a 4 3 0

22
7
7
25
7

x 60 60 50 44
7 100 100 18
100

50 44 7 7 100 100 18
= 2
22 7 7 25 60 60 100
required time = 2 hours.
= x =

[ a 12 3 0 as a 0]

Exercise3
1.

We have,
a = length of each side of the base = 4 cm and
slant height = 5 cm.
Let h be the height of the pyramid. Then,
Slant height =

or, 5 h 2

16
12

h2

a
12
or, 25 h 2

4
3

or, h 2 25

4 71

3
3

71
cm
3

Now, lateral surface area


=

1
(Perimeter of the base Slant height)
2

1
4 4 4 5 cm2 = 30 cm2
2

627

Mensuration II (Surface Area and Volume)


Whole surface area
= Lateral surface area + Area of the base

30 3 42
2

cm
4

= 30 4 3

cm

4.

1
(Area of the base height)
3

cm2

Let a be the length of each side of the base, h


be the height and l be the slant height of the
pyramid.
We have, a = 4

4
71 cm3
3
Clearly, the pyramid is a tetrahedron whose
edge is of length 10 cm.
2
edge 2
12

Volume of the pyramid =

2
3
10 3 cm =
12

Lateral surface area =

h2

Slant height =

50
3

1 3
71
2
= 3 4 4 3 cm3

2.

1 150
cm2

3
3

Volume of the pyramid


=

or, l =

h2

16
12

or, l =

h2

4
3

a2
12

It is given that the number of square units of


whole surface area of the pyramid is three
times the number of cubic units of its volume.
Lateral surface area + Area of the base
= 3(Volume)

2
250 cm3
3

3 3
edge 2
4

or

1
4 4 4 h 2 4 3 42
2
3
4

K KUNDAN
=

3 edge 2

Total surface area =


=

3.

1 3
2

= 3 3 4 4 h

3 3
10 2 cm2 = 75 3 cm2
4

2 cm2

3 10

or, 6 h 2 4 4 3 4 3h
3

= 100 3 cm2

( i ) We have,
a = length of each side of the base = 10 cm
h = height of the pyramid = 5 cm

h2

Slant height =

or, 6 h 2 4 4 3 h 1
3

a2
12

25

2 4
2
or, 36 h 48 h 1
3

2 4
2
or, 3 h 4h 1
3

100
cm
12

or, h 2 8h 0

( i i) Lateral surface area


=

1
(Perimeter of the base Slant height)
2

1
10 10 10 10 cm2 = 150 cm2
2
3
3

or, 3h 2 4 4 h 2 2h 1

10
25
cm =
cm
25
3
3

or, h h 8 0

5.

or, h 8 0
h 0
or, h = 8
Hence, the height of the pyramid is 8 units.
We have, a = 2 m and lateral height = 3 m.
Let h be the height of the pyramid. Then,

Area of one side face

1
=
(Lateral surface area)
3

Lateral height =

h2

a2
3

628

Concept of Arithmetic

2
or, 3 h

2
or, 9 h

2
or, h 9

4
3

Now, lateral edge =

4
3

4 23

3
3

Slant height =

23 4

8 m2 2 m
3 12

7.

1
(Perimeter of the base Slant height)
2

cm

52
2 13
cm
cm
9
3

1
(Area of the base Height)
3

1
2 2 2 2 2 m2 = 6 2 m2
2
Volume of the pyramid

= 8 3 cm 3
9
Let r be the radius, h be the height and l be
the slant height of the cone.
Then, r = 21 cm (given)
Now, Volume = 12936 cm 3
or,

1 2
r h = 12936
3

or,

1 22

21 21 h = 12936
3 7

1
(Area of the base height)
3

4 16

9 3

= 1 3 42 2 cm3
3
4
3

a2
h2
12

Now, lateral surface area


=

a2 =
3

Volume of the pyramid

23
m
3

or, h

h2

3 7 12936
or, h =
28 cm
22 21 21

K KUNDAN
1
3
23

22
m3 =
3 4
3

6.

Now, l2 = r2 + h2

23
m3
3

or, l = Slant height =

We have,
a = length of each side of the base = 4 cm.
Let h be the height of the pyramid and l be
slant height. Then,

l
h
2
or, h

2
2
or, 4h h

8.

a2
12

16
12

or, l

a 4 cm

1225 = 35

or l h 2 r 2

28 2 212

784 441 1225 35 cm


Now, curved surface area of the cone = rl cm2

4
or, h
9

4
3

784 441

Here r = 21 cm and h = 28 cm.


Let the slant height of the cone be l cm. Then,
l2 h2 r 2

282 212 =

22

=
21 35 2310 cm2
7

a2
2
or, 3h
12

or, h

Curved surface area of the cone = rl

1
a2
h2
2
12

2
or, 3h

r2 h2

22

21 35 cm2 2310 cm2


7

2
cm
3

Total surface area of the cone = rl r 2 cm2

h 2

= r (l r ) cm
=

22
21 35 21 cm2 = 3696 cm2
7

629

Mensuration II (Surface Area and Volume)

Volume of the cone =

1 2
r h cm3
3

1 2

=
21 21 28 cm3
3 7

9.

= 12936 cm 3
Let the radius r and slant height h of the
cone be 3x cm and 4x cm respectively. Then,
Volume = 301.44 cm 3
or,

or, l r 2 h 2

1
3.14 3x 3x 4x 301.44
or,
3

or, l 49 100 149 12.2 m


Now, surface area of the tent = rl m 2

301.44
3 4 3.14

22
7 12.2 m2
7
= 268.4 m2
Area of the canvas used = 268.4 m2
It is given that the width of the canvas is 2 m.
Length of the canvas used
=

301.44
8
37.68
x= 2
r = radius = 3x = 6 cm and
h = height = 4x = 8 cm.

3
or, x

Now, slant height =

22
r 44
7
r = 7 metres.
It is given that h = 10 metres.
or, 2

l2 r 2 h2

1 2
r h 301.44 cm3
3

3
or, x

12. Let r m be the radius of the base, h m be the


height and l m be the slant height of the cone.
Th en ,
Circumference = 44 metres
or, 2r 44

r2 h2

cm

36 64 = 100 cm
= 10 cm
10. Let r = radius of the base, h = height and
l = slant height. Then,
=

Area
268.4
=
134.2 m
Width
2

13. Let r m be the radius, h m be the height and l


m be the slant height of the tent.
Then, r = 7 m, h = 24 m.

K KUNDAN
70
cm 35 cm
2
Now, curved surface area = 4070 cm2
r

l r2 h2

or, l 49 576 625 25

Now, curved surface = rl m 2

or, rl = 4070 cm2


or,

22
35 l 4070
7

4070
= 37
110
l = 37 cm
11. Let r cm be the radius and l cm be the slant
height of the cone. Then,
r: l = 4 : 7
or, r = 4x cm, l = 7x cm
Now, curved surface area = 792 cm2
or, l =

or, rl = 792
22
4x 7 x 792
or,
7
2
or, x

792
9
88

or, x = 3
x = 4x cm = 12 cm.
Hence, the radius of the base of the cone is
12 cm.

22

=
7 25 m2
7

= 550 m2
Area of the canvas used = 550 m2
It is given that the width of the canvas is 5 m.
Length of the canvas used

Area
550
=
m = 110 m
Width
5
14. For cylinder
Height of cylinder h1 = 9 cm
=

Radius of base r1 =

60
cm = 30 cm
2

For cone
height of cone h2 = 108 cm
Radius r2 = r1
Volume of cylinder = r 2h
Volume of cone =

1 2
r h
3

630

Concept of Arithmetic
Volume of cylinder = Volume of cone
1
r1 2h1 r2 2 h
3

17. Volume of the glass piston =

h 2
R r 2 Rr
3

(frustum of a cone)

r2 30 30 9 3 225 152
108
or, r2 = 15 cm
The radius of cone is 15 cm.
The diameter of cone is 30 cm.
15. For conical vessel, internal radius = 10 cm
Height = 48 cm
1 2
r h
Volume of water = Volume of cone =
3
2

1
10 10 48
3
= 1600 cm 3
This water is poured into a cylindrical vessel.
Let the required height be h cm.
=

Substituting h

7
cm = 3.5 cm
2

3
cm = 1.5 cm
2
We get, V = 82.76 cm3
Weight of the glass piston
= (82.76 2.1 =) 173.8 gram
18. Let ABC be a cone which is divided into three
parts R, K and T each of height H.
From the figure it can be seen that x is radius
of part R, y is radius of part K and r is radius
of full cone.
Since the triangles AME and ANF are similar
r =

r 2h 1600

= 4 cm, R =

AM ME

AN
NF

or

H
x

2H y

or, 20 20 h 1600

1600

4 cm
h =
20 20
16. Given that R = 28 cm, r = 7 cm, h = 45 cm
Volume of frustum of a cone =

1
h R 2 r 2 Rr
3

K KUNDAN
The capacity of the bucket

22 45

28 2 72 28 7
7
3

22 45 (784 49 196)
21

22 45 1029
=
= 48510 cm3
21
Surface area of frustum of a cone
= [R 2 r 2 (R r )l ]
Slant height of frustum of a cone
=

h 2 R r 2

l = 452 + (28 7)2 = 2025 + 212


= 2025 + 441 = 2466
l =

2466 = 49.66

The surface area of the bucket


22
282 72 (28 7) 49.66
=
7

or, y = 2x .... (i)


Similarly, triangles ANF and AOC are similar

AN NF

AO OC
r=

or

2H y

3H r

3
y
2

1 2
x H
3
Volume of the part K
= Volume of cone AGF Volume of cone ADE
Volume of part R =

1
1
y 2 2H x 2H
3
3

22
784 49 35 49.66
=
7

1
y 2 2H x 2H
3

22
784 49 1738.1
7

1
2
2x 2H x 2H
3

= 22 2571.1 = 8080.6 cm2


7

1
7x 2h
3

631

Mensuration II (Surface Area and Volume)


Volume of part T
= Volume of cone ABC Volume of AGF
=

1 3

2
= 3 2 2x 3 2x 2H

102 242
676

Curved Surface area of the cone = rl


= (3.14 10 26 =) 816.4 cm2

1
27x 2H 8x 2H
3

r2 h2 =

= 100 576 =
= 26 cm

1 2
1
r 3H y 2 2H
3
3

1 3y
2
= 3 2 3H y 2H

Slant height (l) =

1
19x 2H
3
The required ratio is
=

1 2
1
1
x H :
7x 2H :
19x 2H
3
3
3
= 1 : 7 : 19
19. Let the radius and height of the cone be 5x
cm and 12x cm respectively. Then,
Volume = 2512 cm3
1 2
r h = 2512
or,
3
1
3.14 5x 5x 12x = 2512
or,
3
2512
or, x3 =
= 8 = 23
3.14 5 5 4
or, x = 2
radius = 5x = (5 2 =) 10 cm and
height = 12x = (12 2 =) 24 cm

2
Total surface area = rl r
= 816.4 + 3.14 10 10 cm2
= 816.4 + 314 cm2
= 1130.4 cm2
1
2 0 . Radius of cylindrical pipe =
cm
2
Rate of flow of water = 10 m/minute
= 1000 cm/minute
Volume of water coming out per minute from
cylindrical pipe
= r2v
2

1
= 1000
2
= 250cm 3
For conical vessel, r = 20 cm, h = 21 cm
Volume =

1
(20)2 21
3

400 21
3
= 2800cm3
=

K KUNDAN
2800
250
= 11.2 minutes
= 11 minutes 12 second

Time taken =

Exercise4
1.

We have
r = radius of the sphere = 4.2 cm.
Volume of the sphere =

4 3
r
3

4 22

4.2 4.2 4.2 cm3


=
3 7

= 310.464 cm 3
Surface area of the sphere = 4r 2
22

4.2 4.2 cm2


= 4
7

2.

= 221.76 cm 2
We have
r = radius of the hemisphere = 3.5 cm
Volume of the hemisphere =

2 3
r
3

2 22

3.5 3.5 3.5 cm3


=
3 7

2 22 7 7 7
cm3
=
3 7 2 2 2
11 49
cm3 = 89.83 cm3
=
3 2
2
Total surface area of the hemisphere = 3r

22

3.5 3.5 cm2


= 3
7

22 7 7

cm2
= 3
7 2 2

231
cm2 = 115.5 cm2
2

632
3.

Concept of Arithmetic
Let the external and internal radii of the
hemispher ical v essel be R cm and r cm
respectively. Then, R = 12.5 cm and r = 12
cm
Now, Area of outer surface = 2R 2
Area of the inner surface = 2r2
Area of the ring at the top = R2 r2
Total area to be painted

= 2R 2 2r 2 R 2 r 2

= 3R r
=

5.

22
3 12.5 2 122 cm2
7

22 25

122 cm2

= 7
2

Hence weight of the shell = 14942 gm


= 14.94 kg
Let the radius of the sphere be r and the edge
of the cube be x.
Whole surface area of sphere = 4r2
and whole surface area of cube = 6x 2
According to the question,
4r2 = 6x2
or,

r2
6
3

4 2
x2

or,

4 3
r
Volume of sphere
3 3
Volume of cube
x

22
468.75 144 cm2
7

22
13480.5
cm2
612.75 cm2 =
7
7
= 1925.78 cm 2
Cost of painting = Rs (1925.78 0.05)
= Rs 96.28
Volume of the shell V
= Volume of the external sphere of radius R
Volume of the internal sphere of radius r
4
R3 r 3
V =
3

4 r
4 r
r

3 x
3 x
x

4
3
3
2 3

3
2
2
2

4.

3
2

2 3

Hence ratio of the volume of sphere to that of


cube =
6.

6: .

1
2 cm = 1 cm
2

Radius of each ball =

K KUNDAN

4
R r R 2 R r r 2
3

Volume of each ball =

4
3.1416 12 10 122 12 10 10 2 cm3
3

4
3.1416 2144 120 100 cm3
3
4
=
3.1416 2 364 cm3
3
Weight of 1 cm3 of metal = 4.9 gm
W = Weight of the shell of volume V
=

4
3.1416 2 364 4.9 gm
3
8 3.1416 364 4.9
=
= 14942 gm
3

4
4
1 1 cm3 = cm3
3
3

4
16
=
cm3
3
3
Volume of water raised in the jar
= Volume of 4 balls
16

or, Area of the base h =


3
Volume of 4 balls = 4

or, 5 5 h
or, h

16

16
16

cm
3 25 75

Hence rise in the level of water =

16
cm
75

633

Mensuration II (Surface Area and Volume)


7.

Internal volume of rectangular box


= 16 cm 8 cm 8 cm = 1024 cm3
Radius of each metal sphere (r) = 2 cm
Volume of each metal sphere

4 3 4
32
cm3
r 23
3
3
3
Volume of 16 metal spheres
=

= 16
=

8.

32 669
cm3

3 213

512 223
cm3
213

Total surface area = Curved surface area of


the cylinder + Surface areas of hemispherical
ends

= 536.04 cm3 536 cm3


Volume of preservative liquid filled in the
box = Internal volume of box Volume of 16
metal spheres
= 1024 cm3 536 cm3
= 488 cm3
We have, r = 5 cm, h = 8 cm
Volume of cone =

cm
2rh 4r cm

= 2rh 2 2r 2
=

= 2r h 2r cm2
= 2

1 2
r h
3

22
18 72 36 cm2
7

1
200
cm2
5 2 8
3
3
The cone is filled to the brim. When lead
shots are dropped, one fourth of the water
flown out.
The volume of water flown out
=

18 cm, h 72 cm

22
18 108 cm2
7
= 12219.42 cm 2
Rate of polishing = 7 paise per sq cm.
= 2

K KUNDAN
7

Cost of polishing = Rs 12219 .42


100

= Rs 855.36.

1 200 50

=
cm3
4
3
3

10.

The volume of a lead shot


3

4 1
4
1
cm3

3 2
3
8 6

Number of lead shots dropped into the


vessel

9.

50
3 50 6
= 100

We have
r = r adius of t he cylinder = r adius of
hemispherical ends = 18 cm
h = Height of the cylinder = 72 cm

To avoid using r, assume that the radii of the


spheres and the can are 1. Then the volume
of each ball is

4
4
13 =
and the total
3
3

4
volume of 3 balls is 3 = 4 . Since, the
3
height of the can is 6 (the diameter of each

634

Concept of Arithmetic

spher e is 2), the volume of t he can is


4 2

of
12 6 6. So the balls take up
6 3
the can.
11. Volume of the solid hemisphere of radius

[where r is the radius of t he base of t he


circular cylinder and R is the radius of the
sphere.]

2 3
r .
3
Radius of the biggest possible sphere that can
r
be cut out from a solid hemisphere =
.
2
Volume of solid sphere
r

or,

4 r
4 r
1

r 3
3 2
3 8
6

or,

or,

Volume of hemisphere
2r 6
4

Volume of solid sphere 3 1 r 3 1

2r

3r 2h
8r 3

3
2

3
2

3h
3

8r
2

h
4

r
1
Now, we have to find,

Volume of right circular cylinder


3

Volume of sphere
2

3r 2h

or,

Ratio = 4 : 1
12.

3r 2h

2
R3
But it is given that R = 2r
or,

Curved surface area of the cylinder


2rh

Curved surface area of the sphere


4R 2

r h
3

4
2
3
R
3

rh
22r 2

h
4 1

8r
8 2

K KUNDAN
Exercise5

1.

The base of the largest right circular cone


will be the circle inscribed in a face of the
cube and its height will be equal to an edge of
the cube.

2.

Let V 1 and V 2 be the volumes of the right


circular cylinder and cone respectively. Then,

22

6 6 10 cm3
V1 =
7

2
[Using V1 = r h ]

1 22

6 6 10 cm3
and, V2 =
3 7

1 2

Using : V2 3 r h

Volume of the remaining solid = V1 V2

1 22
22

6 6 10
6 6 10 cm3
=
3 7
7

9
cm
2
[ edge = 9 cm]

r = radius of the base of the cone =

Hence, volume of the cone =

1 2
r h
3

1 22 9 9

2673
9 cm3 =
=
cm3
14
3 7 2 2

= 190.93 cm 3

22
2
cm3
6 6 10
7
3
= 754.28 cm 3.
Let r be the radius of the hemisphere and h
be the height of the conical part of the toy.
Th en ,
r = OA = 3 cm
h = 4 cm
=

h = height of cone = 9 cm.


3.

22
1

6 6 10 1 cm3
7
3

635

Mensuration II (Surface Area and Volume)

Slant height of conical part l =


=

42 32 =

16 9 =

This gives the surface area of the top as

h2 r 2

22 3.5 3.5

22 3.5

3.7 cm 2
2
cm2 +
7
2
2
2

25 = 5 cm

22 3.5

3.5 3.7 cm2


7
2

11
3.5 3.7 cm2 = 39.6 cm2 (approx.)
2
Total surface area to be polished = Curved
surface area of cylinder + curved surface area
of the hemisphere = 2rh + 2r2
[Curved surface area of cylinder = 2rh;
Where r and h are radius and height of the
cylinder respectively.]
Curved surface area of the hemisphere = 2r2
where r is the radius of the hemisphere]
=

5.

Curved surface area of cone = rl

22
330
3 5
=
cm2
7
7
2
Curved surface area of hemisphere = 2r

22

396
3 3
= 2
cm2
7

7
Total curved surface area of toy
330 396 330 396 726

=
cm2
7
7
7
7
Cost of painting an area of 100 cm2 = Rs 7.

= 2

22
21 40 21
7

K KUNDAN
Cost of painting an area of

4.

A = 2r(r + h)

726
cm2
7

22

21 61 8052 cm2
= 2
7

Cost of polishing 1000 cm2 area = Rs 10

726 7
Rs 7.26
=
7 100
Total surface area of the top
= Curved surface area of hemisphere +
Curved surface area of cone
Now, the curved surface area of hemisphere

Cost of polishing 1 cm2 area = Rs


Cost of polishing 8052 cm2 area

10 8052
= Rs 80.52
1000
Radius of hemisphere = Radius of cylinder
= Radius of cone = 3.5 cm
Height of cylinder (H) = 6 cm
Height of cone (h) = {15.5 (6 + 3.5) =} 6 cm
= Rs

1
4r 2 2r 2
=
2

6.

22 3.5 3.5

cm2
= 2
7
2
2

Also, the height of the cone


= Height of the top Height (radius) of the
hemispherical part

Volume of toy =

3.5

cm = 3.25 cm.
= 5
2

So, the slant heigh of the cone (l) =

10
1000

2 3
1
r r 2H r 2h
3
3

h
2 2
= r r H
3
3

r2 h2

22
6
2
3.5 3.5 3.5 6
7
3
3

3.5
2

3.25 cm = 3.7 cm (approx.)


2

Therefore, curved surface area of cone = rl

22 3.5

3.7 cm2
=
2
7

7.

31
7

397 .83 cm3


= 38.5 8 38.5
3
3

Required volume = 397.83 cm3


Diameter of bigger sphere = Diagonal of bigger
cube = 500 3 cm

636

Concept of Arithmetic

Side of cube =

500 3
3

Lateral surface area of the conical part = rl


= 500 cm

22
1430
5 13
=
m2
7
7
Total surface area of the tent
= Lateral surface area of cylindrical portion
+ Lateral surface area of conical portion

Volume of bigger cube = (500)3 cm3


Diameter of smaller sphere = 500 cm
Diameter of smaller sphere
= Diagonal of smaller cube = 500 cm
Side of smaller cube =

500
3

1430 1540 1430

= 220
7
7

500 3
cm
3

2970
m2
7
Cost of 1 m2 canvassing = Rs 7.70

Volume of bigger cube


500

Volume of smaller cube


500 3
3

27

3 3

9
3

2970
7.70
7
= Rs 3267.
Let r cm and h cm be the radius and the height
of the solid toy.
Cost of 2970 m2 canvassing =

9 3
3 3
3

9.

Ratio = 3 3 : 1
8.

K KUNDAN
From the question,

2r

1
h
4

h
8
From the figure, r + h + r = 35
or, r

Diameter of the base, d = 10 m


Height of the cylindrical portion, h = 7 m
Height of the conical portion H = Height of
the tent height of the cylindrical portion
H = (19 7) m = 12 m
Lateral surface area of cylindrical portion
= 2rh or dh

or, l

52 122

or, l 25 144
or, l =

169 = 13 m

h 2
35
8

or, h

h
35
4

5h
35
4
or, h = 28 cm
or,

22

10 7 220 sq m
=
7

Slant height of the conical part (l) =

or, h

r 2 H2

h 28 7

cm
8
8
2
Volume of the solid toy
= volume of cylinder + 2(volume of hemisphere)
or, r

2
= r h

4 3
r
3

637

Mensuration II (Surface Area and Volume)


4 22 7 7 7
22 7 7

28

cm3
=
3 7 2 22
7 2 2
= (1078 + 282.32 =) 1360.32 cm3
Total surface area of the solid
= curved surface area of cylinder + surface area
of the sphere
= 2rh 4r 2 2r (r 2h )

22 7 7

56
= 2
7 2 2

1309 cm2

10. Total surface area of the cube = 6 (edge)2


= (6 5 5 =) 150 cm2
Note that the part of the cube where the
hemisphere is attached is not included in
the surface area.
So, the surface area of the block
= Total surface area of cube Base area of
the hemisphere + Curved surface area of
the hemisphere
= 150 r 2 2r 2 = (150 2r 2 ) cm2

22 4.2 4.2

cm2
= 150 cm2 +
2
2
7
= (150 + 13.86) cm2 = 163.86 cm2
11. Denote radius of cone by r, slant height of
cone by l, height of cone by h, r adius of
cylinder by r and height of cylinder by h .

12. Let h be the height of the cylinder, and r the


common r adius of the cylinder and
hemisphere. Then the total surface area of
the bird bath
= Curved Surface area of cylinder + Curved
surface area of hemisphere
2
= 2rh 2r 2r (h r )

22
30 (145 30) cm2
7
= 33000 cm2 = 3.3 m2
13. The volume of the air inside the shed (when
three are no people or machinery) is given by
the volume of air inside the cuboid and inside
the half cylinder, taken together. Now, the
length, breadth and height of the cuboid are
15 m, 7 m and 8 m respectively.
Also, the diameter of the half cylinder is 7 m
and its height is 15 m.
So, the required volume
= 2

= Volume of the cuboid +

1
Volume of the
2

cylinder

1 22 7 7

15 m3
= 15 7 8
2 7 2 2

= 1128.75 m 3
Next , t he t ot al space occupied by t he
machinery
= 300 m3
And the total space occupied by the workers
= 20 0.08 m3 = 1.6 m3
Therefore, the volume of the air, when there
are machinery and workers
= {1128.75 (300.00 + 1.60)} m3
= 827.15 m 3
14. Since the inner diameter of the glass = 5 cm
and height = 10 cm

K KUNDAN
Then, r = 2.5 cm, h = 6 cm, r = 1.5 cm
h = (26 6 =) 20 cm

2
2
r 2 h 2 = 2.5 6 6.5 cm

and l =

Here, the conical portion has its circular base


resting on the base of the cylinder, but the
base of the cone is larger than the base of the
cylinder. So, a part of the base of the cone (a
ring) is to be painted.
So, the area of the painted orange
= Curved Surface area + Base area of the
cone Base area of the cylinder
2
2
= rl r (r )

The apparent capacity of the glass = r 2h


= (3.14 2.5 2.5 10) cm3
= 196.25 cm 3
But the acutal capacity of the glass is less by
the volume of the hemisphere at the base of
the glass.

= 2.5 6.5 2.52 1.52 cm2


=
=
Now,
=

(20 .25 ) cm2 = (3.14 20.25) cm2


63.585 cm 2
the area to be painted yellow
Curved Surface area of the cylinder + Area
of one base of the cylinder

= 2r h (r )2
=
=
=
=

r (2h r )
(3.14 1.5) (2 20 + 1.5) cm2
(4.71 41.5) cm2
195.465 cm 2

ie, it is less by

2 3
r
3

= 3.14 2.5 2.5 2.5 cm3


3

= 32.71 cm 3
So the actual capacity of the glass
= Apparent capacity of glass Volume of
the hemisphere
= (196.25 32.71) cm3
= 163.54 cm 3

638

Concept of Arithmetic
Exercise6

1.

The volume remains same in both the cases


The volume of cube
= Volume of spherical balls
The volume of cube = Number of balls Volume
of 1 spherical ball
or, Number of balls =

2.

4.

16
8 cm
Radius of cylinder =
2
Height of cylinder = 2 cm
Volume of cylinder = r 2h (8)2 2 cm 3

Volume of cube
Volume of one ball

When the cylinder is melted and cast into a


circular cone, then
height of the cone = (3 2 =) 6 cm
Let r1 cm be the radius of cone. Then volume

22 22 22
4 22

13
3 7

1
r12 h1
3
Volume of both must be the same
of cone =

22 22 22 3 7
=
4 22 1
= 2541
The volume of metallic sphere

Diameter of cylinder = 16 cm


r 6

ie (8)2 2

4 22

1.5 1.5 1.5 cm3



3 7

or, 6 (8)2

1
r12 6
3
2
1

or, r1 = 8 cm
Curved surface area of the cone

1 22

.1 .1 L cm 3
The volume of wire =
3 7

The volume will remain same in both the


cases.

= r1l r1 h 2 r12
= 3.14 8 36 64 = 3.14 8 100

1 22
4 22

.1 .1 L =

1.5 1.5 1.5


3 7
3 7

= 3.14 8 10
= 251.20 cm 2

K KUNDAN
4 1.5 1.5 1.5
.1 .1
L = 1350 cm = 13.5 m
Let the total number of balls be x.
Volume of the solid sphere
or, L =

3.

5.

4 3
r
3
Since, the volume of clay in the form of the
cone and the sphere remains the same, we
have
its volume is

4 3 4
r 33 cm3 = 36 cm3
3
3
0.6
cm
2
= 0.3 cm.

Radius of spherical ball =

Volume of
=

a spherical ball =

4
(0.3)3 cm3
3

4
3
3
3
36
cm3 =
cm3

3
10 10 10
1000

36
x cm3
1000
Clearly, volume of the solid sphere = Volume
of x spherical balls.
Volume of x spherical balls =

36
x
1000
or, x = 1000
Hence, 1000 spherical balls are obtained by
melting the given solid sphere.
or, 36

Volume of cone = 6 6 24 cm 3
3

If r is the radius of the sphere, then

6.

4
1
r 3 6 6 24
3
3
or, r3 = 3 3 24 = 33 23
r = 3 2 = 6
Therefore, the radius of the sphere is 6 cm.
The volume of water in the overhead tank
equals the volume of the water removed from
the sump.
Now, the volume of water in overhead tank
(cylinder)
2
= r h = (3.14 0.6 0.6 0.95) m3
The volume of water in the sump when full
= l b h = (1.57 1.44 0.95) m3
The volume of water left in the sump after
filling the tank
= {[(1.571.440.95) (3.140.60.60.95)} m3
= (1.57 0.6 0.6 0.95 2) m3

639

Mensuration II (Surface Area and Volume)


So, the height of the water left in the sump

99

1000 litres
=
28

Volume of water left in the sump


=
l b

99000
litres
=
28

1.57 0.6 0.6 0.95 2


m
=
1.57 1.44

= 0.475 m = 45.7 cm

Since,

Capacity of tank
Also, Capacity of sump

litres of water is emptied in 1

second.

99000
litres of water will be emptied in

28

3.14 0.6 0.6 0.95 1

=
1.57 1.44 0.95
2
Therefore, the capacity of the tank is half the
capacity of the sump.
9.

7.

25
7

1
The volume of the rod = 8 cm 3
2
= 2 cm3
The length of the new wire of the same volume
= 18 m = 1800 cm
If r is the radius (in cm) of cross-section of
the wire,
its volume = r 2 1800 cm3

7
99000

seconds, ie in 16.5 minutes.


25
28
Let the height of the cone be h cm.
Now, according to the question, since volume
of the cone is equal to the volume of the solid
cylinder.

1 2
r h r 2 3
3
h = 3 3 = 9 cm
10. Volume of the cylinder = r2h = (8)2 2
= 128 cm3 and

1 2
r 6
3
Since, according to the question, volume of
the cone is equal to the volume of cylinder.

Therefore, r 2 1800 2

Volume of the cone=

K KUNDAN
or, r2 =

1
900

1
30
So, the diameter of the cross-section ie the
or, r =

thickness of the wire is

1
cm ie 0.67 mm
15

1 2
r 6 128
3
or, r2 = 64

or, r =

64 = 8 cm

(approx.).
8.

Radius of the hemispherical tank =

3
m
2

Volume of the tank


3

99 3
2 22 3
m

m3 =
14
3 7 2

So the volume of the water to be emptied

1 99 3
m
=
2 14

The slant height of cone l =

62 8 2 = 10 cm

The curved surface area of the cone = rl


= 3.14 8 10 = 251.2 cm2

You might also like